You are on page 1of 121

 

 
             
 
 "!# $ %  & '(*)+( , .-/0123+-/., 4 !5 # %   4'6)+( &7/(8# !+8# % 4& $ '*!5( , #$ +% #  4&  9
 : ;8# % +4& $ +'.!5( ,7 # <=* 4+& +*4=*(86('(()& 4>/(8?% 4?!+8# % 4& * (#64.@A+ )
4=*(8+ (B=*(+ /

C DE+4 $ & # 4 $ (F(B +>4#  4*(B4*% $ # % G  (6 >4#  4*("4.$ & % # $ -5 )& H84#  9
 34F%  #  4$ * (( -I4G GIG  4'8 + (+G /F,74 /+& (6& % (#  !5($  &4# 7 (>J$ % J*4>@I G )3'(4GI (#7
!5($  & (#>& % (# 64F (8% )+( , ? (#>KF!5($  &  L( ,M=*4/.!5(& $  $ N6& % (#  & % 4+(!5(& & $ -+G /
-5>(-+ 4$ + )3-/.4F  4=O$ 3 $ &7'4=* 9
P Q   # $ 4'G R SFTF7UVG $  &*(QRTF74+)QWXG $  &3(QS6TF7, $  QU3WY!I4# 4 G G  7 G  (ZR S3[ 
RT  IU*T
 C +4+).SFW + + 3, +4 $ & +>G  ' .("W6T69
  
K7L( ,\=*4/0# $ '> # $ 4'G  &>4 NF4G G& $ )+FG  +' +&>$   ' # & , $  -5( ?G  .
' G  ' &!+# $ =*
8+=>-5 # & 9
]7^7+6% # (& & _ &  %  $ (4G"4#  4*(4* #  >$ & 43G $  4# 8+%  $ ((" $ =*7^7># 4)$ 8+& $ &*   $ 
` 1*4).*    $ ? `]"DE+4 $ &7$  &# 4)$ 8+&7$ 1119
`7aB >b" c  cBd c  4).e5 c c c c   fB c $ &74F!5(G /+(=*$ 4G

"


& 8% . +4b" c e5 c f5 c   ., 4 $ & N4G 8>("fB 9
"
17^7+F  =F!5 # 4 8# *(?4& 8=*=* #F N $ +'% 4?-5*)+   # =*$  )?-/Z% (8 $ ' +*8=6-5 #
(% $ # !+&>4.% # $ % J 6=*4J &$ Z3&  % (+)&64)?4))+$ '.]F^7*& !5  )?(4./!5(   $ % 4G
4N4# $  &, $  * +   =F!5 # 4 8#  $ 3& 8% .4>,74 /* +4 + 8+=>-5 # (5$ +% + & +4 4
=*( N &7$ 06=*$ 8  &7$ &7]FG  & &7 +43 +   =F!5 # 4 8# L ( ,g=*4/3 $ =* &)+( & 4F% # $ % J 
% $ # !., +$ G   >47 # 4 N G & 1F$ +% + & 9
7hg'$ # GI'( &8!46& J$IG $  74 C =*$ G  &!5 # (8# +4).% (=* &)( , * & J+$I& G (!5 4  C =*$ G  &
!5 #6+(8# 6  *& J+$& G (!5F$ & +F& 4=*FG  +' ?4&  *& J+$"G $   B4+)0,F$ '+(# F4/0 $ =*
& !5  47  (!5+, 4 $ &7+ # 4 N # 4'>& !5  ) (#7 # (8+)3 # $ !B+$ .=*$ G  &!5 # (8# 9
 i N *, $  7& (% J&4)3 (8#-+G 4% J*& (% J&4#  $ *4>-+4'^,(6& (% J+&4#  )# 4 , 34# 4+)(=.
, $  (8 #  !IG 4%  =*   DE+4$ &7 >!+# (-I4-+$ G $  /. 4   /+4 N> 6& 4=*>% (G (# 9?D?# $  
4& 46#  )8+%  ) # 4%  $ (I
7hOG (+'0 & % 4G 4 (#F'( &6 # (=X40& 8-,4 /Z # 4% J? (. 3&  #   F4)j=*( N &68!j4F% (+&  4
# 4 Fk6!5 # & (0#  H8$ #  & C 13&  % ()+& (3,4G J08!?, $ G 6 +6 A+% 4G 4 (#>$ &=*( N+$ '54+)
 4J & C 1&   !I&$ F +!+# (%  & & h(  #!5 # & (6#  H8+$ #  &1&  % (+)& ( ,74G JF8!F, +$ G  +
 & % 4G 4 (#$ &"=*( N$ +'4+)> 4J &1 &   !I&"$ > !+# (%  & & L ( ,j=*4/6&   !+&( + & % 4G 4 (#
4# #  H8$ #  )3 (*'(F # (=l +>& 8-,4 /3 (6 >&  #    $ B + & % 4G 4 (# $ &7+( =*( N+$ '9
C  i8!+!5(& F 4> *8=6-5 # &*" C "] PB"P C ]B4+)ZP.4# F!+G 4%  )Z$  (3 *`
& H8+4#  &F$ Z4_ -/_ '# $ )Z$ Z& 8+% j4.,74 /? 4F 4% j( 3`08=6-5 # &64!+!5 4# &F A4%  G /
(+% "4)0 6!+# ()8+% 6( +F8+=>-5 # &>4!!5 4# $ '.$ ?4/0# ( ,\(#>% (G 8+=*0$ &  *& 4=*
DE+4 $ & N4G 8>(5 !+# ()8+%  $ . 4% 3# ( ,m4)% (G 8=*I9
nop7qrs t u t v t wx0y\s t z {rx?|rx+}w ~{ B€"+z { }+w ~+{ "wx}0‚Zz }x+z { }+w ~+{>wx+}u z s s { u ƒ+z6u „ u ƒ0rx0u ƒ+z
ru ƒz „>}+w ~+{ r…"u ƒz6†z z ‡5pq"rs t u t v t wxˆgs t z {rx€7ƒ„ { }w ~{ B‰„ t }w ~{ 5wx}?Šwu „ }+w ~+{ 5wx+}
u z s s {7u ƒ+z>u „ u ƒrxu ƒ+z6ru ƒ+z „}w ~{r…"u ƒ+z>†z z ‡5p‹xzF}w ~.Œ5ru ƒ0r…"u ƒ+z Ž{ w ~[ z { u z „ }+w ~
†7w{7rx+zr…"6~3s ~+t x‘F}+w ~+{ p ’F‚Eƒwu}w ~.t {7t u7† ƒz x.u ƒ+z ~.{ w ~3u ƒ+t { “
n ”p‚Eƒ+wuFt {>u ƒz3{ *ws s z { u6•5r{ t u t –z*t xu z ‘z „F† ƒt v ƒZv wx+xru*rv v „Fw{>u ƒz3}+t —5z „ z x+v z*Œ5z u †7z z x
u †7r6•5r{ t u t –z•„ t *zx+>Œ5z „ { “
n˜p‚?„ t u z+t x*t xu z „ –wsI… r„ .u ƒ+z { rs u t rx*{ z u7r…Iu ƒ+z t x+z ™+ws t u ~0š ›BœB0ž ›60ž+šŸmš ›BœB0ž ›> ?ž+š p
n ¡p‚Eƒ+wu*t {Fu ƒ+z.¢£ v rF•5rx+z xu*r…7u ƒ+z.v z xu z „3r…7u ƒ+z.v t „ v s z3† ƒ+t v ƒj•Iw{ { z {Fu ƒ„ r+‘ƒE¤ n¥ ž¦ 
¤ §¥ ž¦ +wx}?¤ o¥ ¨¦ “
n ©p‚?„ t u z*¤ žªªn «Qn ©¡”«Qno« ¦ ¬¤ žªªn­n ©¡”­no¦w{7wx.t xu z ‘z „ r„7„ z }+v z }… „ wv u t rx5p
žªp Šz *t v t „ v s z {7w„ z}„ w † x3† t u ƒ.z wv ƒ.{ t }zr…Bw6„ t ‘ƒuu „ t wx+‘s z w{7}+t w*z u z „ { +w{t x3u ƒ+z ®I‘„ zp
¯ z u.°>7±3wx+}Q²X}+z xru z.u ƒ+zt x}+t v wu z }[w„ z w{ pE€7ƒ{3²Xt {Fu ƒzw„ z wZr… u ƒ+z.u „ t wx‘s z
† ƒ+t s z°mwx+}.±lw„ zu ƒ+zw„ z w{7t x{ t }zu ƒz>{ *ws s z „ { z *t v t „ v s z {† ƒ+t v ƒs t zru { t }zu ƒ+zs w„ ‘z
{ z *t v t „ v s zp³´•„ z { { ²Mt x3u z „ *{7r…"°mwx}±*p

µ µ µ µ µ µ µ µ µ µ µ µ µ°µ µ µ µ µ µ µ µ µ µ µ µ µ µ µ µ µ µ µ µ µ µ µ
µµ µ µ µ µ µ µ µ µµ µµ µ µ µ µ µ µ µ µ µ µ µ µ µ µ µ µ µ µ µ µ µ µ µ µ µ µ µ µ µ µ µ µ µ µ µ µ µ µ µ µ µ µ µ µ
µµµµ µµµµµµ µµµ µ µµµµµµ µ µµµµ
µµµµµµ µµµ
µµ µ µ µµ µ µ µ µ µ µ µµµµ µµ
µ µ µ µ µ µ µµ
µµ µ µ± µ µ µ µ µ µ µ µ µµ µ µ µ µ µ µ µ µ µ µ µ µ µ µ µ µ µ µ µ µ µ µ µ µ µ µ µ µ µ µ µ µ µ µµ
µµµ µ µ µµ µ µ ² µµµµµµµµµµµµµµµµµ µ µ µ µµ
µ µ µ µ µ µµµ µ µµµµµµµµµ µµµµ µµµ µµ
µ µ µ
µ µ µ µ µ µ µ µ µ µ µ µ µ µ µ µ µ µ µ µ µ µ µ µ µ µ µ µ µ µ µ µ µ µ µ µ µ µ µ µ µ µ µ µ µ µ µ µ µ µ µ µ µ µ µ µ µ µ µ µ µ µ µ µ µ µ µ µ µ µ µ µ µ µµ µµ µµ µµ µµ µµ µµ µµ µ µ µ µ µ µµ µ
µ µ

žnp¶ B… { t x7· { t x œ · 0{ t x « · ?­ ­ ­¸E¨u ƒ+z x*† ƒwu7t {u ƒzs w„ ‘z „r…5u ƒ+z7u †r>•5r{ { t ŒIs z7–ws z {… r„
v r{+·  jv r{ œI·  jv r{ «5·  [­ ­ ­ “
žžp ¹rx+v z xu „ t v v t „ v s z {r…I„ w}+t {7˜wx}nn w„ z }„ w † x*t x*w>•+s wx+zpyEv ƒ+r„ }3r…+u ƒz s w„ ‘z „v t „ v s z
t {7u „ t { z v u z }.wut u {•5rt xu {7r…"t xu z „ { z v u t rx.† t u ƒ3u ƒ+z>{ *ws s z „ v t „ v s zp‚Eƒwut {u ƒz>s z x+‘u ƒr…
u ƒz>v ƒr„ }+“
ž§p¶ .x ƒ+r †m*wx~*†w ~{7v wx3~r3†ws ‡3•wF{ u wt „ †7w ~3† ƒt v ƒ.ƒw{ ˜6{ u z •I{7t …5~r.v wx3u w ‡z6n>r„
ž*{ u z •I{wuw*u t *z “[¤ ‰+r„z ´+wF•+s zI~rv w x†ws ‡•?w3{ u wt „ †7w ~.
† ƒ+t v ƒ0ƒ+w{ §3{ u z •+{t x0§
}+t —5z „ z xu †7w ~+{ º7n £ n £ n5n £ ž+r„ž £ np ¦
ž¨p‚Eƒ+wu t {7u ƒ+zrx+s ~3•5r{ t u t –zt xu z ‘z „7† ƒ+t v ƒt {7}+t –t { t Œ+s zŒ~˜žFwx}ƒw{ }t ‘t u 7{ w{ v z x+}t x+‘*t x
v rx+{ z v u t –z6r„ }+z „ “Z¤ €7ƒ+{ z wv ƒ}t ‘t u 6+{ u7Œ5zFn6‘„ z wu z „7u ƒ+wx.t u {•+„ z }+z v z { { r„ p ¦
žop7ymv t „ v s zt {7t x{ v „ t Œ5z }.t x.wF{ ™w„ zp¶ x3wx~.rxz>v r„ x+z „7r…Bu ƒ+z{ ™w„ zu ƒ+z „ z>t {wx.t { r{ v z s z {
„ t ‘ƒu>u „ t wx‘s z6† ƒt v ƒ?{ ƒw„ z {>w3–z „ u z ´0† t u ƒ0u ƒ+zF{ ™+w„ z*wx}?ƒ+w{>ƒ~•5ru z x{ zFu wx+‘z xuu r
u ƒz>v t „ v s zp‚Eƒwu t {7u ƒ+z „ wu t r*r…5u ƒz>w„ z wFr…Bu ƒ+t {u „ t wx‘s zu rFu ƒ+zw„ z wFr…Bu ƒz>{ ™+w„ z “
ž”p ¯ z u»?z ™wsu ƒ+z7s w„ ‘z { ur…Iž« ¼o½ ¼ 
 wx}*”½ ¾wx+}*¿u ƒz7{ *ws s z { ur…+u  ƒ z { zu ƒ„ z z7x+>Œ5z „ { p
‚?„ t u z>u ƒ+z6r„ }+z „ z }•+wt „F¤ »7¥ ¿¦ I+{ t +x ‘*u ƒ+z>‘t –z xz ´•5rxz xu t wsB… r„ Ž… r „ u ƒz>u †7r*x+>Œ5z „ {
u ƒwu7~r3†7„ t u zp
ž˜p7ˆ7z ‘t xx+t x‘F† t u ƒ0nIws sIu + ƒ z•5r{ t u t –zt xu z ‘z „ {7w„ 
z †7„ t u u z x.{ v v z { { t –z s ~Œ5z ‘t x+xt x+‘
nž§¨o”˜¡©n ªnnnžn §n ¨À À À p‚Eƒwu }+t ‘t u w••5z w„ 7 { t x3u ƒz6žªªn Á  •5r{ t u t rxI“
ÃÄÅ7ÆmÇ5È É.Ê Ë+ÈFÌ ÍÎ+ÏÄ*Ð*Ñ Ò Ó Ï7Ô5Ó ÌË+ÈÍÌ>Ñ Ï ÕÖÄ*È×"Ø ËÓ>Ê7Ù ÉÙÚ Ì ÈÏ ÏÙFÌ ÙÑ Ò Ì ÈÙÛ.Ç+Ì Ñ Û+ÜÓ>Ê ËÓ Î0ËÓ
Ë+Ó ÙÌ ÏÙ>Ø Ì ÙÑ Î*Ê ËÑ Ï Ø Ò Ó7Ç5Ó Ë+Ñ ÎÛ3Ë+Ñ Ð.Å"Ý ×5ËÓ Ì Í+ÎÏÇIÙÚ Þ5ßË+Ó7Ê Ñ Ò ÒIÒ Ó Ù àÓ7Ø ËÓ7Ç+Ì Ñ Û+ÜÓ ÙØÔÌ Ó Ú Ñ Ï Ó Ò É
Ø ËÓFÐ*ÈÐ*Ó ÎØ Ø Ë+Ó>Ø Ì ÙÑ Î0Ó ÎØ Ó Ì ÏÑ Ø Å Ý ×Ñ Î+Ï Ø Ó ÙÛ0ËÓ6ÞÓ Ó ÔIÏÈÎÌ Í+ÎÎ+Ñ ÎÜ3Ø È*Ø ËÓF× ÙÌÓ Î+Û0È×Ø Ë+Ó
Ç+Ì Ñ Û+ÜÓßØ Ë+Ó Ø Ì ÙÑ Î*Ê Ñ Ò Ò+Ì Ó ÙÚ Ë.ËÑ Ðmá ÍÏ Ø7ÙÏË+ÓÒ Ó Ù àÓ ÏØ Ë+Ó ÇÌ Ñ ÛÜÓÅâEË+ÙØ7Ñ ÏØ ËÓÏ Ô5Ó Ó Û.È×5Ø Ë+Ó
Ø Ì ÙÑ Î.Ñ Î.Ð*Ñ Ò Ó ÏÔ5Ó Ì Ë+ÈÍÌ ã
ÃäÅâEË+ÙØ Ñ ÏØ ËÓ>Ï Ð*ÙÒ Ò Ó Ï ØÔ5ÈÏ Ñ Ø Ñ àÓ Ñ ÎØ Ó ÜÓ Ì åægçèéFÏ Í+Ú Ë.Ø Ë+ÙØ*êë Ñ Ï7Û+Ñ àÑ Ï Ñ Ç+Ò Ó ÇÉQêë ÇIÍØ
ìí ìî ìíïî
Î+ÈØ Ó ðÍ+ÙÒIØ ÈFÑ Ø ã
ÕéÅ7ñ7ËÌ Ó Ó àÓ Ì Ø Ñ Ú Ó ÏÈ×5Ù>Ú ÍÇ5Ó Ñ Î*Ï Ô+ÙÚ Ó Ë+Ù àÓ Ú ÈÈÌ Û+Ñ ÎÙØ Ó ÏòFó Õô õô çö ß÷.ó èô Ãô äö ßÙÎ+Û*ø*ó çô ùô èö Å
âEË+ÙØ ÙÌ Ó Ø ËÓ>Ú ÈÈÌ Û+Ñ ÎÙØ Ó Ï7È×BØ Ë+ÓÚ Ó ÎØ Ó Ì È×BØ ËÓ>Ú ÍÇ5Ó ã
ÕçÅ7úBÓ Ø7ògÙÎÛ3÷\ÛÓ Î+ÈØ Ó Ø 
Ë Ó Ô5ÈÑ ÎØ ÏÈ×5Ñ ÎØ Ó Ì Ï Ó Ú Ø Ñ ÈÎ3È×5Ø Ë+Ó Ú Ñ Ì Ú Ò Ó Ïû5üBý0þü"ÿùû>ý0õþ [Õ6ÙÎ+Û 
 
ûBüýjþ+üýjõû*ÿZõþ gç ÅâEË+ÙØ Ñ ÏØ Ë+Ó>Ï Ò ÈÔ5Ó>È×BØ Ë+ÓÏ Ó ÜÐ*Ó ÎØ ò÷Fã
ÕÃÅ7úBÓ Ø7û ì  û
ü  û     Ç5Ó>ÙFÒ Ñ Ï Ø Ñ Î+ÜFÈ×BÙÒ ÒIØ Ë+ÓÎÍ+Ð>Ç5Ó Ì ÏÊ ËÑ Ú Ë.Ú ÙÎ3Ç5ÓÊ7Ì Ñ Ø Ø Ó Î.ÙÏ7ÙFÏ Í+Ð


Ø Ô5È Ê7Ó Ì ÏÈ×ÕÅ IÈÌÓ ÙÐFÔIÒ ÓßIû ì mÕ ï \çßIû mÕß5ÙÎ+Ûû mõÅ
È×ÈÎÓ6ÈÌ>Ð*ÈÌ Ó6Û+Ñ Ï Ø Ñ ÎÚ 
ü 
âEË+ÙØ Ñ ÏØ ËÓàÙÒ ÍÓ>È×5û ì ï ï ã


ÕÕÅ Ñ ÎÛÙ6Ô5ÈÒ É+ÎÈÐ*Ñ ÙÒ5È×BÛ+Ó ÜÌ Ó Ó>õFÊ Ñ Ø Ë.Ñ ÎØ Ó ÜÓ Ì Ú ÈÓ *Ú Ñ Ó ÎØ ÏÊ Ë+Ñ Ú ËËÙÏ Ã7ý
èFÙÏ7ÙFÌ ÈÈØ Å  
ÕõÅ7ÆgÒ Ñ ÎÓ Ï Ó ÜÐ*Ó Î ØÔIÙÌ ÙÒ Ò Ó Ò+Ø È6ÙÎ3Ó ÛÜÓÈ×5Ù>Ì Ó ÜÍ+Ò ÙÌË+Ó ÙÜÈÎ3È×5Ï Ñ Û+Ó>çÛÑ à+Ñ ÛÓ ÏØ Ë+ÙØË+Ó ÙÜÈÎ
Ñ ÎØ È6Ø Ê7È6ÔIÑ Ó Ú Ó Ï7Ê ËÈÏ ÓÌ ÙØ Ñ ÈFÈ×BÙÌ Ó ÙÏ7Ñ Ïç  
à ÅâEËÙØÑ ÏØ Ë+Ó>Ò Ó ÎÜØ Ë.È×5Ø ËÙØ Ò Ñ Î+Ó>Ï Ó ÜÐ*Ó ÎØ ã

ÕèÅ7úBÓ Ø.Ø Ê7ÈZÄ EçÃ?Ì Ó Ú Ø ÙÎÜÒ Ó Ï.Ï ËÙÌ Ó0ÙjÚ ÈÐ*Ð*ÈÎ[Ú ÈÌ Î+Ó ÌÙÎÛEÈ àÓ Ì Ò ÙÔEÙÏ.Ñ ÎQØ ËÓ0Û+Ñ ÙÜÌ ÙÐ
Ç5Ó Ò È Ê>ßIÏ ÈFØ Ë+ÙØ7Ø ËÓ6ÛÑ Ï Ø ÙÎ+Ú Ó>ò÷l× Ì ÈÐlØ ËÓ>Ç5ÈØ Ø ÈÐlÌ Ñ ÜËØÚ ÈÌ ÎÓ ÌÈ×"ÈÎ+ÓÌ Ó Ú Ø ÙÎ+ÜÒ ÓØ ÈFØ Ë+Ó
Ñ ÎØ Ó Ì Ï Ó Ú Ø Ñ ÈÎ0Ô5ÈÑ ÎØ>òMÙÒ ÈÎ+Ü*Ø ËÓ6Ì Ñ ÜËØ>Ó Û+ÜÓFÈ×"Ø ËÙØÌ Ó Ú Ø ÙÎÜÒ ÓFÑ Ï ÅâEË+ÙØ>Ñ Ï Ø ËÓFÙÌ Ó Ù.È× 
Ø ËÓÌ Ó ÜÑ ÈÎ.Ú ÈÐ*Ð*ÈÎ*Ø ÈFØ Ë+ÓØ ÊÈFÌ Ó Ú Ø ÙÎÜÒ Ó Ï ã
 
        Ä

 
çÃ

       


                                                ò       

    
         
Ä     
         
      
 ø    
                    ç  Ã                      ÷
ÕùÅÝ 
× Ø Ë+Ó6Ò È ÊÓ Ì>Ò Ó × Ø>Ú ÈÌ Î+Ó Ì>È×"Ø +
Ë Ó6ÙÇ5È àÓ6ÛÑ ÙÜÌ ÙÐŽÑ ÏÙØFó éß éö ßIÊ Ë+ÙØÙÌ Ó>Ø ËÓ6Ú ÈÈÌ Û+Ñ ÎÙØ Ó ÏÈ×
Ø 
Ë ÓÔ5ÈÑ ÎØ øMÊ Ë+Ñ Ú ËÑ ÏØ Ë+ÓÒ È ÊÓ Ï Ø7àÓ Ì Ø Ó .È×BØ Ë+Ó>Ï Ò ÙÎØ Ó Û3Ì Ó Ú Ø ÙÎ+ÜÒ Ó ã

Õ ÅâEË+ÙØ7Ñ ÏØ ËÓ àÙÒ ÍÓÈ×5Ø  Ë Ó Ô5ÈÏ Ñ Ø Ñ àÓ Ñ ÎØ Ó Ü Ó Ì7å0× ÈÌÊ Ë+Ñ Ú Ë3Ø Ë+ÓÒ Ó ÙÏ Ø7Ú ÈÐ*Ð*ÈÎ3Ð6Í+Ò Ø Ñ ÔIÒ Ó È×BÕù
ÙÎ+Û.å?Ñ Ï èééFÜÌ Ó ÙØ Ó Ì7Ø ËÙ Î3Ø ËÓ>ÜÌ Ó ÙØ Ó Ï Ø Ú ÈÐ*Ð*ÈÎ3Û+Ñ àÑ Ï ÈÌ7È×"Õù*ÙÎÛ.åBã

ÕÄÅ ÍÔ+Ô5ÈÏ Ó6Ø Ë+ÙØ Ø Ë+Ó6Ï Í+ЎÈ×"Ø + Ë Ó>Ï ðÍ+ÙÌ Ó ÏÈ×"Ø ÊÈ3Ú ÈÐFÔ+Ò Ó ÎÍÐ6Ç5Ó Ì Ï MÙÎÛ 3Ñ Ï ß+Ø ËÓ6Ï Í+Ð   
È×Ø Ë+Ó Ñ Ì>Ú ÍÇ5Ó Ï>Ñ ÏFç éß"ÙÎÛ0Ø Ë Ó Ñ Ì6Ï Í+ÐYÑ Ï>Ù3Ì Ó ÙÒÎÍ+Ð>Ç5Ó Ì ÅFE
â Ë+ÙØ>Ñ ÏØ ËÓ*Ò ÙÌ ÜÓ Ï ØÌ Ó ÙÒ"àÙÒ Í+Ó
 
Ø ËÙØ jý *Ú ÙÎ.Ë+Ù àÓ ã
 "!"#$&%(' )* '+(, -('. /10 #$ -2 * .'2 2 '0 %(* 3&2 #4'+$2 2 #-5767'0 %&2 , 8&*/#$&9 . * : :72 % *+($ 2 2 #-; 2 %(*
0 #$(-2 * ., -(0 . * ': * :+/=<;5* > 9 . * : : , - ?12 %(*4-$ 84+5* ., -@+(, -('. /- #2 '2 , #-567'0 %2 , 8&*'13(, ?, 2
A , 9(:, -2 % *10 #$ -2 * . ;B/#$=9(' /C<D% *10 #$ -2 * ., :4, - , 2 , 'E E /='24F @GH%('2, :2 %(*&0 #: 2#I
9(. * : : , -(?12 % *4+$2 2 #-=JFF <2 , 8&* : K L JFF <, -+, - '. /, :4<<<<< F < FFF <M 4N(#.* > '89E *;52 %(*
0 #: 2#I59 . * : : , - ?2 %(*+$2 2 #-@42 , 8&* :", :CO ;(: , -(0 * , 2"P, E E5?#I . #8QFF42 #F < 2 #&< F42 #1<<; '2
'&0 #: 2 #IRC<SCJS C<
OF "GH%('295#: , 2 , )* -$(8+5* .T: '2 , : U* :W V T&SX YZW V T&YX4[\ 
(] ^_a` b"c ^d]@b^fegg hiBjkl
k m n o p n q o4r sp p o r ntu v w"o p xay uv z{(tp o|(p tr }o n vwo4~ y v nn q(o4u{(|5o psmR€5o s€~ o4s{nsm"h‚
wq s4tu v w"o p o ƒ&y n"r sp p o r n ~ „xm s~ ~ s wo ƒ|„n q(ou{(|5o p"s{ n7smn q(o…… €5o s€~ o"wq(sv r sp o ƒ&tn"~ o tv n
h †4wq(stu(v wo p o ƒy nr sp p o r n ~ „‡
h‡† ‡ˆ hg x …e ‰5c ny v"n q(o p o r y €(p s r t~5sm Š‹Œf‹4 Ž‹( ‡
e‡‡ˆ h ‘ h‡ x(……‰R’ Š Ž “‡7bq{(v’Ž ‡
… ‡e‡1ˆ ‚” x …e ‰Bbq ou(o w•€(p y r oy v g ‡ ”1n y &o vn q(o4s~ ƒ€(p y r o‡bq{ v n q(o4t&s{(un n q(tnt1ƒ(s~ ~ tp
wy ~ ~€{p r q(tv o4y v h–g — ”Ž•h— en y &o v7wq(tny n{ v o ƒ1n s4|5o‡
‘ ‡ˆ ‚” x …e ‰a˜R–e‡Rc man q o p tƒ y {(vy v™x(n q(o u1n q oy u(v r p y |5o ƒ@v z{(tp oy vr s€5sv o ƒ@smRm s{p y v sv r o ~ o v
p y šqnn p y tu š~ o v"wy n q1|5sn q@~ o švo z{(t~n s&™‡7bq o p tn y sy v˜a™  – › ‘™  –eœ ‡
‡ˆ ” hx e‚‰† ‡Bbq(oy €5sv v y |(~ ov r sp o v7tp o hx(ex ‘ x(x” x tu(ƒ@hh‡Bbq(ov r sp o vBn q tn"r tu&|5o&tƒ o
wy n q1g4n s{(r q(ƒ s wu vtp o g x(… x † x(‚ x5— — — ‡7bq o v r sp o v"n q tnr tu1|5o &tƒ owy n qh n s{(r q(ƒ s wu
tp o1x7h g x"h … x7— — — ‡@bq o&v r sp o vn q tnr tu|5o1&tƒ owy n qe&n s{ r q ƒ(s wu(v4tp o@h ‘ x7hx"— — — ‡
bq p o osp&sp on s{(r q(ƒ s wu vwy ~ ~5ƒ({€~ y r tn osu(osm5n q o v ov r sp o v ‡7c ny vo tv „1n so u{(&o p tn o
n q o&y v v y u(šv r sp o v ‡
† ‡” ‡Zˆ hh † x ……‰bap y tu(š~ o v@žŸ& ¡tu ƒ ž¢£¡tp o=v y &y ~ tp ‡¤bq{(v¥"i¦ ¥"§Ži7¨ ¦ k§4‡© o u r o
¥"i¦ ‘Že¦ h‡
‡g ‡ˆ ‚ x e†‰¨"sn q&~ o šv7r tu(u sn"|5os ƒ ƒ5x v y u(r o4› eª« ¬=hœ  ¬X› e­ « ¬=hœ  Ž\‘¢¬ex wq y r q&y v"u sn
t1v z { tp o‡®Xo44{(v n q(t ¯o4°  Ž•ª  ¬e  wy n q°tu ƒª@sƒ(ƒ@€ p y &o v ‡bq(o u‘&Ž °  Œª  Ž
› °7ŒXª(œ › °B¬ª œ7wy n q1°"ŒXª&±He4tu(ƒ@°7¬Xª1²\ex(wq y r q@y vy €5sv v y |~ o‡
” ‡³ h g ‡Xˆ hx h g‰"™  y u r p o tv o ƒX|„@y uXt ´ „o tp4€5o p y s ƒ‡c n4wy ~ ~7y u r p o tv o&|„hxRn s=h g xBy u
tu(sn q(o ph4„o tp v ‡
‚ ‡‡ˆ e‚ x he ‰µ(tr n sp › ¶a·aŒhœBŽf› ¶Œhœ › ¶ “ ¬@¶ŠR¬@¶5a¬@¶ ¬=hœ7tu(ƒ› ¶5aŒhœ7Ž•› ¶Œhœ › ¶ ¬=hœ
tu(ƒr tu r o ~"n q(o› ¶Œ hœm tr n sp vy uXn q(o1o z { tn y su‡X^| n ty u4¸ eŽe4¸(h¸ ¹a› hœ ‡X^p&šo n
¹a› ¶aœBŽ\¶“"¬=¶ Š ¬X¶  ¬X¶¬\h‡
h g ‡‘”g ‡1ˆ †‚ x e ‰Bbq oy u r q o v&s ¯o ƒy uXhe1&y u{n o v o z{(t~ vn q(or q(y p €v y uXhº‘@&y u{n o‡bq{(v
y u@tu„@€5o p y s ƒ@sm5n y &ox n q(ou{ 4|5o psmRr q(y p €vy v‘”n y &o v"n q(ou{ 4|5o psmRy u r q o v&s ¯o ƒ‡
hh‡‘”º‡Hˆ ‡ x …g‰7c mn q o@~ o u šn qsmn q(o@v ~ s€5o@y vŸ»&y ~ o v x7q o p1v €5o o ƒy v&eŸ1– › ¼ “ ¬½¼ “ œ&Ž
e– ¾ “ Ž\‘”–‡
he‡eº‡ˆ ‚ ‡ x e‡  ‰c ny v› ¿ ¾ À ¬f¿ “ À œ –5¿   À sp• ¾  ¸  ‹ Á ¬  “  ¸  Š Á ‡7i7y n q o pšy ¯o v‘ºhh g ‡
h … ‡ heg ‡ˆ †g x e”‰(c un q o o Ân p t&h g4v o r su(ƒ v x n q(oo v r t~ tn sp"&s ¯o ƒ1eg4v n o €(v ‡7bq{ v7n q oo v r t~ tn sp
šs o v7n w"sv n o €(v7€5o pv o r su ƒ5x(tu ƒ@v s4y u&n q o ‘g4v o r su(ƒ vy n"w"o un”gv n o €(v ‡7”g7¬‘g4Ž heg4Ž
h gg¬eg ‡
h ‘ ‡‘g‚† ‡ˆ ‡ x e”‰5bq o o  €5su o un vm p sQgn s”4tp o|5o y u š4€~ tr o ƒ@y u&n q o v z{(tp o vv s4n q tnn q(o
v { y uo tr q@p s wftu ƒr s~ { &uy vn q o4v t&o‡bq o4r s&&su@v { y v4› h"¬H¸ ¸ ¸¬”œ –…Žfhe‡
bq{(vn q(o r s&&su1€ p sƒ({ r n y ve   Ž\‘g‚† ‡
ÃÄÅÆÇÈ É Ê Ë Ì ÍÅÎ Ã ÏÐ Ñ ÐÐÒBÆÇ(ÓÔÕ(Ö ÍË(Ì Í(Ê × Ç(Ì×ØBÔÖ Ù × Ù Ú Ù ÌÕÛÝÜ"ÔÈ(Ö Ë=Þ&ÌßÓ4× Ç(Ì×Ê × Ì× Ó Þ&Ó Õ×ÌÉ Ó
à ÔÕ Ë(Ì Í4ÌÕ(ËÆÇÈ É Ê Ë Ì ÍÑÌÕ Ë× Ç(Ó7ÔÕ(Ö ÍË Ì Í Êa× Ç(Ì×RØBÔÖ Ù × Ù Ú Ù ÌÕáÜÔÈ Ö Ë4Þ&ÌßÓB× Ç(Ì×RÊ × Ì× Ó Þ&Ó Õ×
ÌÉ ÓÆÇÈÉ Ê Ë(Ì ÍÌÕ ËâÈ(Õ Ë(Ì ÍÅ
à ã ÅäÅ1Î ÄåÑ åÄ ÒRÆÇ(ÓÊ Þ&ÌÖ Ö Ó ÉÕÈ Þ4æ5Ó É Ê Ú ÌÕæ5Ó4Ôæ(× ÌÙ Õ(Ó ËÌÊ ÐçåÑRÄ çÐ ÑRÄ çåÑRä çÐ ÑRä çåÑ
ÌÕ(Ë=ÃÃçÄÅÆÇ(Ó4Ë Ù è5Ó É Ó Õ Ú ÓÔéBÔË(Ë@ê É Ù Þ&Ó ÊÙ ÊÓ ëÓ Õ5Ñ5ÌÕ ËÊ Ù Õ(Ú Ó4ì&Ù ÊÕ(Ô×ê(É Ù Þ&ÓÑaäÚ ÌÕ(Õ Ô×
Ô Ú Ú ÈÉ ÌÊÌÕ@Ô Ë Ë1ê(É Ù Þ&ÓÞ&Ù ÕÈ ÊÌÕ@Ó ëÓ ÕÔÕ ÓÅ
ÃäÅí î ï åð Å7Î ÐåÑ Ãà Òñ ×RÜÙ Ö Öæ5ÓB× Ç(Ó7Ôê5Ó Õ æ5ÔÈ(Õ Ë(Ó Ë4Ù Õ× Ó É ëÌÖÜÇ(ÔÊ Ó7Ó Õ(Ëê5ÔÙ Õ× ÊRÊ Ì× Ù Ê é Íò5óçå òRô4åõ
ç4í ò ó çå ò1çXåð Ñ(Ê Ô4ò ó çå ò@õHî Å
à ö Åã ÅQÎ ÄäÑ Ã ìÒ÷XÓÜ"ÌÕ×øÊ È(Ú Ç\× Ç(Ì×=í Ãï ø ð&Ù Ê1Ó ùÈ(Ù Ë Ù Ê × ÌÕ×@é É ÔÞúí Ð ï åðÌÕ(˕í Äï Ïð ŕÆÇÈ(Ê
å ó ôHí øçåð ó õHÏ ó ôHí øçXÏð ó Å7ÆÇ(Ù Ê"É Ó Ë(È Ú Ó Ê× ÔÏøõ åÏ Å
à ì ÅÐ ÅÎ ÃÄÑ ìÒ5÷HÙ × Ç@ûõ•Ã ìöãÌÕ Ëüõ ÃÄÑÙ ×Ù Ê4í í û ôü ð ý"çXûý"çXü ý ð þ í í û ôü ð û ü ðBõ\Ð Å
åî Åÿ• ô 1ŤΠÐÄÑ ÃÄ Ò aÓ ×  Ë(Ó Õ Ô× Ó× Ç(ÓÙ Õ× Ó É Þ&Ó Ë(Ù Ì× ÓÌÉ Ó Ì ÑÌÕ(Ë\Ö Ó  ×  Ñ Ñ ÌÕ Ë
HË Ó Õ(Ô× Ó
× Ç Ó&× Ç É Ó Ó1Ê Ù Ë Ó Ê4Ôé"× Ç(Ó&× É Ù Ì Õ Ö ÓÑRÜÙ ×  Ç
=× Ç Ó1ÇÍê5Ô× Ó ÕÈ Ê ÓÅÆÇ Ó  Õ   ô   õ 
ó ÑRÜÇ(Ù Ö Ó
ÿ ô Hôú õ    óB ô  óÅBá"Í1× Ç(Ó ØBÍ × Ç(Ì ÔÉ Ó ÌÕ@ÆÇ(Ó ÔÉ Ó Þ@Ñ •õ\ÿ ô 1Å
åÃÅÐ åÅÝÎ äÑ ä ÒÊ Ù Õ þ í Ã4çHÊ Ù Õ ð1õZÏXÌÕ(ËÇ(Ó Õ Ú ÓÊ Ù Õ \õZÏþÄÅHÆÇÈ(Ê1Ú ÔÊ õZÐþÄ=ÌÕ Ë\Ê Ô
Ú ÔÊ þ í Ã"çXÚ ÔÊ ðBõHÐþåÅ
ååÅ Ã ö ÅÎ Ã Ï Ñ öÒñ é× Ç(Ó7Ú Ù É Ú Ö Ó ÊRÌÉ Ó7Ú Ó Õ× Ó É Ó ËÌ×"í î Ñ îð5ÌÕ(Ë × Ç Ó7Ú Ç ÔÉ Ë4Ù ÊaÌ4í ëÓ É × Ù Ú ÌÖ ð5Ö Ù Õ(ÓBÔé Ú ÔÕ(Ê × ÌÕ×
òRÑ × Ç Ó ! Õ ÃåÃç=ò ó õ\Ð Ïìç=ò ó Å7ÆÇ(Ó Õ1ò ó õ\Ïî ÑÌÕ Ë@Ê Ô× Ç(Ó Ö Ó Õ × ÇÙ Êå ÃåÃçXÏîÅ
åÐ ÅåÃÅÎ ÄäÅ ÄÑ åÐ#Ò "Õ(ÓÜ"Ì ÍÜÙ × Ç&ÌÖ Ö5à $ Ê %'& () *Ü"Ì Í ÊBÜÙ × Ç&ÔÕ Ó å %'& +ó *ÜÌ Í(ÊBÜÙ × Ç× Ü"Ôå $ Ê %ÌÕ( Ë &,*
ÜÌ Í(ÊÜÙ × Ç× ÇÉ Ó Ó&å$ Ê Å@í ñ Õé ÌÚ × ÑaÌÊ × Ç(ÓÕÈ Þ4æ5Ó ÉÔé7Ê × Ó êÊÙ Õ Ú É Ó ÌÊ Ó Ê Ña× Ç(ÓÕÈ Þ4æ5Ó É4ÔéBÜ"Ì Í ý Ê
× Ç Ì×ÍÔÈ@Ú ÌÕË Ô× Ç(Ù Êé ÔÉ Þ&Ê7× Ç .Ó -BÙ æ5ÔÕ ÌÚ Ú ÙaÊ Ó ù È Ó Õ(Ú Ó4åÑÐ ÑÄÑ(ö Ñaà РÑåÃ#Ñ / / / Å ð
åÏ ÅÐÏÄã Å Î Ïã Ñ åà ÒaÆÇ(ÓÕÈ Þ4æ5Ó É é ÔÉ Þ&Ó Ë@æÍ1× Ç ÓÖ ÌÊ ×× ÜÔ&Ë Ù Ù × ÊÞ4È(Ê ×æ5ÓË Ù ë(Ù Ê Ù æÖ ÓæÍÏ ÅâÔ1Ù ×
ÞÈ Ê ×4Ó Õ Ë=ÜÙ × ÇXÄã Åí Ãå1Ù Ê × ÔÔ@Ê Þ&ÌÖ Ö Å ðXÆÇ Ó&Ê È ÞZÔéB× Ç Ó&Ë Ù Ù × ÊÞ4È(Ê ×æ5Ó&Ë Ù ë(Ù Ê Ù æÖ Ó4æÍ=ì Å
÷XÔÉ ß Ù Õ 4æÌÚ ß Ü"ÌÉ Ë Êé É ÔÞ¤× Ç ÓãÍ Ù Ó Ö Ë(Ê"× Ç(Ó É Ó Ê È Ö × Å
åÄÅí Ð çå åð þÏ ÅÎ Ã Ð Ñ Ãå 0Ò aÓ ×× Ç(ÓÊ Ù Ë(ÓÖ Ó Õ × Ç@Ôéa× Ç ÓÊ ùÈ(ÌÉ Ó æ5ÓÃÅ7ÆÇ(ÓÌÖ × Ù × È Ë(ÓÔé5× Ç ÓÖ Ù × × Ö Ó
× É Ù ÌÕ Ö Ó Ù Ê4í åçÃð þåÅ7ÆÇ(ÓÌÉ Ó Ì&ÔéRÌÕ@Ù Ê ÔÊ Ú Ó Ö Ó ÊÉ Ù Ç×× É Ù Ì Õ Ö ÓÓ ùÈ(ÌÖ Ê"× Ç ÓÊ ùÈ(ÌÉ ÓÔéRÙ × Ê
ÌÖ × Ù × È(Ë ÓÅí ÆÇ(Ù Ê"Ù Ê"× É È Óæ5Ó Ú ÌÈ(Ê ÓÙ éaÊ È Ú Ç@Ì4× É Ù Ì Õ Ö Ó Ç ÌÊÖ Ó Ê"ÔéRÖ Ó Õ × 2
Ç 1Ñ× Ç Ó Õ@Ù × Ê"ÌÖ × Ù × È Ë(Ó
Ù Ê 1þ åÅ ð4ÆÇÈ(Ê× Ç(Ó ÌÕ(Ê ÜÓ É Ù Êí åçÃð ó þÏ Å
åã Åí ã ) , ï Ä ) + ð ÅÎ ÐÐ Ñ Ã ÐÒRâ Ù Õ Ú ÓÐ4å 3Ðã ÑRí å + ð 5 3•í ã ó ð 5 Å"âÙ Õ(Ú ÓÃå.Ä 3 Ãåö Ñaí Äý ð + 3•í å 5 ð + Å
åäÅÐ ÅÎ Ïö Ñ åå Ò(ÆÇ(ÓÕÈ(Þæ5Ó É ÊB× ÇÉ Ô È Ç1ìì É Ó ù È Ù É ÓìBô=à öîË Ù Ù × Ê Å"âÙ Õ(Ú Óí åîî ÃRçà öìð þÐõ\ãîÏ Ñ
Ù ×ÜÙ Ö Öæ5Ó × Ç Ó × Ç(Ù É ËË Ù Ù ×ÔéBäîÐ Å
åö ÅÐåŤΠÄåÑ åÏ Ò aÓ 2 × 6¤Ë Ó Õ(Ô× Ó× Ç(ÓÖ Ó Õ × Ç\Ôé× Ç(Ó@æ(É Ù Ë ÓÑ 7¡× Ç ÓË(Ù Ê × ÌÕ Ú Óé É ÔÞ»× Ç Ó@× É ÌÙ Õ
× Ô@× Ç Ó&æ(É Ù Ë ÓÜÇ(Ó ÕX× Ç(Óæ5Ô ÍXÇ Ó ÌÉ Ê× Ç Ó&ÜÇ(Ù Ê × Ö ÓÑBÌÕ( Ë 8H× Ç Ó1Ê ê5Ó Ó ËXÔé"× Ç Ó&× É ÌÙ ÕÅÆÇ(Ó Õ
7@þ 8Ýõ ý 6"þö=ÌÕ Ë í 7Q ô 6"ð þ 8fõ + 6"þö ÅÆÇÈ Êí 7Qô 6"ð 9þ 7 õ½ÄþÐ Ñ7Ê !Ô 7 õ ýó 6fÌÕ(Ë
8õ ÐåÅÛ Ö × Ó É Õ Ì× Ù ëÓ Ö ÍÑ(ÜÇ(Ó Õ@× Ç Óæ5Ô ÍÙ ÊÐ Ï1ÔéR× Ç(ÓÜ"Ì Í@× Ç É ÔÈ Ç@× Ç Ó× È Õ(Õ Ó Ö Ñ× Ç Ó× É ÌÙ Õ
ÜÙ Ö Ö : È(Ê ×æ5ÓÓ Õ× Ó É Ù Õ &Ù × 'Å ; Ó Õ Ú ÓÙ ×ÞÈ Ê ×æ5Ó ÔÙ Õ Ï× Ù Þ&Ó ÊÌÊé ÌÊ × ÌÊ"× Ç(Ó æ5Ô Í@Ù Õ@ÔÉ Ë Ó É× Ô
Ú Ì× Ú ÇÇ Ù ÞQÌ×× Ç Óé ÌÉÓ Õ(ËÅ
åì ÅÏÄåÅÎ ã Ñ Ä Ò &=<) + ) * þ &=<) + > * õfí ?1çÃÄîð þÃÄÃÅB÷XÓ ÞÈ Ê ×Ç Ì ë.Ó ?1çÃÄî4õHÐîåÅ
@A B.C D E @ E FG BH I E J9KLMN2O N PQ9R MSUTVR M N2R M W N NS N QXUN PR SUY9W NZ []\_^ `aEZ b\_^ a9D EYPc
[db\ ^ DI B'LMeSZ[X4eS Rf#NYUc g YQTPYO#TV#R M N.h ef#N9EYPc2g R SXUg c i#Tg PRR M N.h N PR N WTV
R M N.h e f#N9B
@ jB Fk9D BH ajB FE jjB F K=l Rdg S.R M NP N QY9R g mNUW N h g iW T h YO=TVR M NS O T9i#NUTVR M NO g PNUh TPP N h R g P Q!R MN
h N PR N W S B LMN4h N PR N W SY9W N4Y9RdC @ n o aGYP cC o an aG B LM N4S O T9i#N4TVpR MN4O g P N4h TP PN h R g PQUR MN
h N PR N W S4g SdoDqFBr O R N W PY9R g mN O sE0S e f R W Yh R4R MN4R tT2N u e Y9R g TP S.YP cwvPc!R MY9R4YPs!i#Tg PR S
O sg P QUTPf#T9R M2Xde S R O g NTPR M N.O g PN4j A9xowIyd\j D E t Mg h MM YS S O T9i#N4Fk9D B
@aBzI jBH @ E @9K{=Yh Mdi#TS g R g mN g PR N QN Wh YPdf#NtW g R R N PUe Pg ueN O sUYS=Y4S eX|TV#cg S R g P h R=i#T tN W STV
aB=LM N T9W c N W g P Q4TVR MNS e XUSTVcg S R g P h Ri#T tN W STV#@t g O O f#NR M N S YXUN YS'R MN T9W cN W g PQ4TV
R M N S e XUSTV#c g S R g Ph R=i#T tN W STV0at g R MUh T9W W N S i#TP cg P QdN }i#TPN PR S B=LMe S=x~dt g O OM Y mNR MN
S YXUN4N }i#TPN PR S.TV'@2YSTh h e W.g PR MN4cN h TXdi#TS g R g TPTV=YSY2S eX€TV'cg S R g P h Ri#T tN W S
TV'aB=T9W N } YXdiO N x‚ \ @ ƒp„@…h T9W W N S i#TP cSR Td@4\ aƒp„a9…9B†g Ph N4j AA4\a9‡'„aˆp„a‰ E
x ƒ … … \Š@‡„@ˆ=„@‰ B
@@ Bx#‹ oŠj D9x#‰„z BH aE A9K=† N Rx\ ^ a„ ^ FB4LMN PC x2o ^ aG ‰.\|FB4LMeSx#‰o@\|a ^ a xpB
† ueY9W g PQ4f#T9R M2S g c N S YP c2N } iYPc g PQds g N O cSR MN.c N S g W N c2W N S e O R B
@D B ^ @ B€H D E j K ŒNwcW Y t€R M N!R T9iM YO V4TVR M NwM N }YQTPf#N O T t.B|ŒNt g S MR Th MTTS N!xS T
R M Y9R4R M NY9W N Y!Y9f#T mNUR MNO g P Ng Sdak9@TVR MNUY9W N YTVR MNUR W Y9i#N ‚ Tg cB!LM NMN g QMRdTVR MN
R W Y9i#N  Tg cg S ‰ƒ ^ @EYP c2g R S=fYS N SY9W N.j.YP c2aB=LMe Sg R SY9W N Ydg S ‹ ^ @ BpŒN tYPRx2S T.R M Y9R
x ‰ƒ C a„  ‰ Ž ‚ G'\ ‰ƒ ^ @ BpLM N P4Te Wpc N S g W N c4O N PQ9R Mt g O O9f#Nj„  ‰ Ž ‚ B0ŒN=M Y mNa x#‰9„.a x0^ @o @4\ A
YPc2MN P h N.x2\C @o ^ @G qaB'LMeSR M N.cN S g W N c2O N PQ9R Mg S ^ @ B
         x                    

@FBDaB H j A E a K#‘0N R’“cN P T9R N R M Ne i i#N W O N V RmN W R N }TV#R MNO T tN WW N h R YPQO N9B'LMNc N S g W N c2Y9W N Y
g S R M NdS e X”TVpR tTUW g QMRR W g YP QO N S t g R M!h TXUXUTPMsi#T9R N Pe S NdZ’2Bd•P NdMYSO N QS4jdYPc
j9aE=S TR MNMsi#T9R N Pe S N2g S ^ j DFBLM NT9R M N WUM YSdTPNO N Q!I E=S TR M NT9R M N WUO N Q!X4eS Rdf#N
^ j DF owJD4\ z B=LMe SR MN.Y9W N YUg S ‰ƒ C j9a„`aG B
@J B.C D – I n j– JG B2H A E A9K=‘0N R4’—c N PT9R NdR M NdmN W R N }wY9R.t M g h M!R M NdR tT2W N h R YP QO N S4g PR N W S N h R BLM N
R YP QN PRTV#R M N.YPQO NY9R ’“f#N R tN N P2R MNMT9W g  TPR YO#YPc2cT t PtY9W cS O YPR g PQdO g PN Sg S
R YP0C Y9W h R YP ƒ ƒ ‰ „Y9W h R YPd™ ˜ G'\ jƒ ƒ o‰ „ ƒ ˜™ ™˜ \ D@ –
ƒ‰
LMeSR MNO g PN’b|QTN ST mN W ‚ˆ IdYPc2cT t P ‹ˆ I B
@`B F9J BH J E J9K0LMN.O h X“Xde S Rf#N.N mN P#EM N Ph N.R MN.Qh c2Xde S Rf#N.N mN P#BLM N4TP O si#TS S g fO N.N mN P
mYO e N S.V T9WR MN4Qh cwY9W Nd@J Epj I E'‚ j9aE#J E0D E0YP cwaB.Œ M N PF9AAg SYcc N c!R TR MN S N9E#R M Nd‚ TPO s
S e Xcg m g S g fO Nfs@J4g‚ SF9AD4\a @‰ `B†g Ph N Qh c0C a‰ @‰9n 0G'\a‰YP cO h XC a‰ @‰9n 0G'\ a @‰ `E
tN.X4eS R MY mN.\ a `B
š› œ œž › Ÿ   ¡=¢ £¤9¥ ¦§ ¥4¨ © ª9¨U«Š¬ ­!ª£¦«® ¬Š­®ª9¥ §U¯#¤9¨ ©w¥ § ª° Ÿp±§²4³´ ¨U© ª µ§«·¶¸¬Š¹ º
ª£¦­¶“¸»Š¹ ºœw¼©§ £¸ ® »Šº ® ¶¾½ ª£¦¸¿.»š9¸#º ® ¶€Àœ¼© Á ´4 Á § ° ¦´4¨ ©§§ óª9¨ Á ¤£
¸'Ä Å  »9¸ ® Æ ¶  ÇU¤9¥ Çd¶9¸¿»Å  ¸d¬ ®   Ç œ¼©Á ´ § óª9¨ Á ¤£© ª´ ¤£§¥ § ª°0´ ¤° ³¨ Á ¤£2È9¥ § ª9¨ § ¥
¨ © ª£! œ'¼0¥ ÂU¸2¶Å.ª£¦UÉ£ ¦¨ ©ª9¨Á ¨±¤9¥ Ê ´ œ=¼© §¦§ ´ Á ¥ § ¦´ ³ ²“Á ´Å ¸2¶  œ=¼© § µª° ³ § ´¤Ë#­
ª£¦2«ª9¥ §4ÅÌÎÏ Í ® œ
šÐ œ Ñ9šÐÐÀœž Ò œ ÀŸ 9¡¼© § Ó ´'¦ Á ÈÁ ¨'± Á ° ° Ô © ª£È§§ µ§ ¥ Â4¨ Á ²U§9Ÿ9¨ © § ÅÓ ´'¦Á ÈÁ ¨'§ µ§ ¥ ÂUÅ9£ ¦d¨ Á ²U§9Ÿ¨ ©§ Ó ´
¦Á ÈÁ ¨§ µ§ ¥ 9¨ ©2¨ Á ²U§9Ÿ§ ¨ Ô9œ¼©§4£³ ²4¯#§ ¥¤Ë'¦ ¤° ° ª9¥ ´ÕªÁ ¦¦³ §.¨ ¤UÔ © ª£È§ ´ Á £2¨ © §U Ó ´ ¦Á ÈÁ ¨
©ª´¯Á £ª9¥ § ÖÕª£ ´ Á ¤£w      Ç   ÇÇÇ  Ÿ ¨ ©§.£³².¯#§ ¥¤Ëp¦¤° ° ª9¥ ´ÕªÁ ¦¦³ §.¨ ¤UÔ © ª£È§ ´ Á £2¨ ©§
ÅÓ ´¦Á ÈÁ ¨©ª´=¯Á £ª9¥ § Ö Õª£´ Á ¤£      Ç   ÇÇÇ Ÿ¨ ©§ £³².¯#§ ¥ ¤Ë#¦¤° ° ª9¥ ´=ÕªÁ ¦¦³ § ¨ ¤4Ô © ª£È§ ´
Á £d¨ © §  Ó ´=¦ Á ÈÁ ¨© ª´'¯Á £ ª9¥ ÂU§ ÖÕª£ ´ Á ¤£2      Ç   ÇÇ Ÿ§ ¨ Ô9œ'¼©³´'¨ © §¨ ¤9¨ ª°£³ ²4¯#§ ¥¤Ë¦¤° ° ª9¥ ´
ÕªÁ ¦± Á ° °¯#§¨ ©§£³².¯#§ ¥Ô ¤9¥ ¥ § ´ Õ#¤£¦ Á £Èd¨ ¤d¨ ©§´ ³ ²·¤Ë#¯Á £ ª9¥ Â2£³ ²4¯#§ ¥ ´¯#§ ÈÁ £ £Á £ Èd± Á ¨ ©
      Ç   ÇÇÇ  Õ° ³ ´¨ © Á ´£³².¯#§ ¥ ± Á ¨ ©2ª£Â2£³².¯#§ ¥¤Ëp¦Á ÈÁ ¨ ´Ô ³ ¨ ¤9×!¨ © §¥ Á È©¨ ´ Á ¦ §9œØ=ªÔ ©
 ± © Á Ô ©ª9ÕÕ#§ ª9¥ ´Á £¨ ©§Ù¨ ©Õ#¤´ Á ¨ Á ¤£Á £       Ç   ÇÇÇ  2± Á ° °ª° ´ ¤ª9Õ Õ#§ ª9¥2¤£ Ô §Á £§ ªÔ ©
Õ#¤´ Á ¨ Á ¤£¨ ¤4¨ ©§ ¥ Á È©¨ ¤Ë#¨ ©§Ù¨ ©Á £¤£§¤Ë#¨ © §Ë ¤° ° ¤ ± Á £ Èd£³ ²4¯#§ ¥ ´Á £¨ ©§´ ³ ²2œ=¼©³ ´ ŸË ¤9¥
§ Öª²dÕ° §9Ÿ¨ © §Á £ Á ¨ Á ª°#  ± Á ° °ª9ÕÕ#§ ª9¥¤£ Ô §Á £U§ ªÔ ©UÕ#¤´ Á ¨ Á ¤£Ë ¥ ¤²|¨ © §Å9Ú Õ#¤´ Á ¨ Á ¤£U³ Õ¨ ¤.¨ ©§
ÅÛ ÚÕ#¤´ Á ¨ Á ¤£#œ=¼©§.£³ ²4¯#§ ¥ Ô ¤£¨ ªÁ £ Á £ÈU´ ³Ô ©w Ó ´ Á £2Á ¨ ´¯Á £ª9¥ Â2§ Ö Õª£´ Á ¤£Á ´ ÅÛ Û'» œ¼© §
´ ³ ²Ë ¤9¥'ª° °¨ © §  Ó ´pÁ ´Ä Å Û Û »d  Æ ¬Ä Å Û Ú »d  Æ ¬Ä Å9Ü»d  Æ ¬Ä Å9Ý »U  Æ ¬2Ä Å½ »d  Æ ¬Ä ÅÞ»U  Æ ¬2Ä Å Û »d  Æ œ
¼©§.´ ³²“¤Ë0¨ ©§4Å ß Õ#¤ ±§ ¥ ´©§ ¥ §.Á ´ Å à9Å9ÇÇ  4ª£ ¦ádÁ ´´ ³¯¨ ¥ ªÔ ¨ § ¦Ë ¥ ¤²·Á ¨ œ
Ç œ ⠛Ç œž  Ÿ   ¡ã³¯#§¯#¤9¨ ©´ Á ¦ § ´4¨ ¤!ȧ ¨4¸2¬ Ð4»ŠšÄ ¸2¬ŠÐ Æ ® ä ¿Ä ¸!»Ð Æ Û ä ¿ ¬ŠšÄ ¸2¬ŠÐ Æ Û ä ¿Ä ¸»
Ð Æ ® ä ¿»Ä ¸!»Ð Æ ¶€Åáœ!¼©Á ´4´ Á ²dÕ° Á ɧ ´.¨ ¤Ä ¸ ® »Š›   Æ Û ä ¿Ä Ä ¸»Ð Æ Û ä ¿»Ä ¸¬ŠÐ Æ Û ä ¿ Æ ¶·š œ
å Âd¨ © §¤9¥ Á ÈÁ £ ª°§ à ³ ª9¨ Á ¤£Ÿ¨ © §§ Ö Õ ¥ § ´ ´ Á ¤£Á £UÕª9¥ § £¨ © § ´ § ´§ óª° ´ »š œ=¼©³´=¨ © §§ óª9¨ Á ¤£
¥ § ¦ ³Ô § ´¨ ¤d¸ ® »w›  ¶» œ
  
 

 !"# $%!& ')(*+ "$,,-/.01 "2!"-3,
45,-76 .98:;#<,-=$,)" 2!& -3,
> *' > )",()?$<'!@47A
B9&')() > '!(C $D!DE
,-E 89)"FE

G IHJ-3K?#<':;!-3<'C ,-/. ,- > <'3L,!")M,"N!1O!?9'()!" '?)"NI-3P@ $,$,*'Q-3K?#<')-)SRT#<,-
U<'CV!D
()L'()$,:;,(+ >  -3-3)-;9'!?F5())"N*'-!D
O!?9'-3?9W#-3K?#<')-+A
6IXD*
Y  [ZZZ 8\ B] ZZ 8 "_^ /` ZZ H'3'+ "1Ya8\78b "_^
,"c,"(*'C -3,"2! '*'Cda! '

fe# P > ,$  8, gY@hi\hj^78:I'L5Yak+\7k+^7
.9Il*m#"Vn"*:oW#,"#<'3p! > *'+<,!"b8q(C $,$,)_rs8q!"U'C $&"s?P W%*'-WspF O! 'P1?$VE@r=t

E t 6SRT#<I'C $"s?P W%*'vuU-<,-wm#)-IExr=u EyO! 'v $,$%EbA
 
z RT#<,-{$<')-w=,"N)*';|V-3?(+M#<=} }~,-QL9,-3LW#$,;Wsp2|VO! 'I $,$ > !-3LLN;,"N)*'-

}DA
€9RT)" z `@,-L9,)yWsp~V!c"s?P W%*'5}‚8b1K?! ,)"N5,-5 > !-3LLN > ',Pn "s?P W%*'
 "F5')P@ ,"*';,-;`9SRT#<;,-I}DA
09IXD*IE 7ƒ2„…,"M'+  "-+†fSRT#<I,-=  $,?!D$,! „‡+ "=Eb† $,! „…-3,"Eb† $,! „…()!-9Eb†3A

/`9Rˆ'LU "~)K?#<,!"Ffe > ')-3-3,"2Y@,"*'Pn-I!a‰w?-w\5 "~^5,    8#E t \78a "
Š  ‹ Œ 
E  t  ^  

IH3' "$,v# -=N*'3feMŽ<
„`9k+BN†f8NN*'3fe2‘’<
„ G k+`N†f89 "MN*'3fe2“”<
„‡E&kf6†•O! '-3!PnE
W%*[:=))"y`x " G ‚&U<'CV!D53' "$,,-€989)"F:;#<;,-5  $,?5!&EbA
7Id‚,"F5!"$LpM'C $-3!$,?9,!"F!DE €<E  77E B€N6 `9
 
/B9;–*'x<'U[:=!2"!"()!" '?)"N5')(*+ "$,)-:;Ly,"N)*'5-3,)-O! '5C (+_!S:;,(+yV<'C
)K?# $,- > *',Pn**'C‚RT#<,-
-3?P—!DU<'C -I!D5[:=!V')(*+ "$,)-+A
}
G ’™ ˜ „|9† 8)"F:I'L
V-3,P > $,fe > ')-3-3,!"O! ' „|9†f
B } 
šC›  œ œ
76;–"s?P W%*'-='!P—!1 ``N5<'$,,-w)n,"nO!$,$,!/:;,"
! '*'CžSd‚L'-w= $,$a"s?P W%*'-{:;,(+
<'M"! VL9,-3LW#$,nWspŸBy<'M$,,-w)," „…,"(*'C -3,"s†U! '*'C_–)"¡ $,$Q"s?P W%*'- :;,(+¡<'
L9,-3LW#$,WspB1W#?9"! ;WspFB  <'
$,,-w)F,"! '*'CQ–)"~ $,$"s?P W%*'-;:;,(+~<'5L9,-3LW#$,
WspŸB  W#?9V"! 1Wsp¢B <'M$,,-w)Ÿ,"Ÿ! '*'C8{*( cRT#<x,- M$ -wV"s?P W%*'V,",-V$,,-wfA
„¤£LN
)"NL'
"s?P W%*'C8#"! ‚‰w?-wL-I$ -w;,LC¥†
/.9IH¦$,! 3*'3pŸLN)- > ',§))-x:;,(+i<' > !/:=*'-@!U¨–s?-n > !-3-3LW#$, > ' ,§))-@<'_8v8
785/BB98 G . G 8I "/.9/`N6~!$,$<'-)–*p¡:;,$,$;LN~"!Pn! '# "J/`C:<'-M!
 "Np > <'3,()?$<' Pn! ?"NC–*pLNU!?9vfe# (*$Lp~©8,8,ªv!/:oP@ "Np > ',§))-!n*p
C:<'aA
z IHi()L'()$, >  -3-3)-‚9'!? [:=!5 7‰3 ())"N‚N*'3,())-‚!av-3K?#<'= "V,-&+ ")"N‚!=! >> !-3L
-3,I!aI-3K?#<' ‚aI-3,I$,)" x!aI-3K?#<'I,-{8N:;#<•,-S'+ ,?-•!aI()L'()$,/A
/€9;–=:=!x"s?P W%*'-<'5-3?(+F#<)L'; *')"() 8#)L';-3?P8# "F)L' > '!?(*v<'!VC (+
! *'v -5ž z ž¥ G •–)L' > '!?(*P1?-w;)K?# $b:;#<;"s?P W%*'fA
/09IHo9'+C:=*'()!"N+ ,"-;}_-3!(9-)SRT)"F[:=!n<'59'+C:;"F'+ "!Pn$Lp@:;L!?9I' > $  ())Pn)"NC8
> '! Wa<W#,$,L[pM#<IW%! <''),-6 G ‚RT#<,-
-3P@ $,$,)-w > !-3-3LW#$,  $,?

 !&}DA
 `9IH1!/: P@ "NpˆPn,"s?9)- >  -w1€9ž ``F:;,$,$•nPn,"s?9M "!?9'1# "-1O! 'P )K?# $Q "$,)-
:;L25N*'3,(C $bO! ';m'-w;,Pn/A
Iªv!/:JP@ "Np > !-3LLN5,"N)*'-<'5L9,-3! '-I!S ``NA
Rˆ'L2cK?# 9'+<,( > !$Lp9"!Pn ${#C9,"_ -1L- [:=!c'!! -U2"s?P W%*'-V! W+ ,")ŸWspŸ,"
(*'C -3,"nC (+F'!! ;!DE  7E 6 `1WspM5')() > '!(C $!D
! *'C

 B9"2'+ (),"V!/N*'1LN)"2,-w+ "()
<I?",O! 'P - > ))b8aHv$b(C "MW%C<=! W2WspF `1Pn**'-)8 =! W
(C "2W%C< 
=9',-Wspy/`1Pn**'-)8# "2Hv$b(C "2W%C< 
=9',-WspF €1Pn**'-)Sªv!/:P@ "Np@Pn**'-
,-
,-w+ "()
!/N*'I:;,(+F*p<''+ (),"sA
 G ;–"s?P W%*'S}M# -• ``N;,L-)8 $,$!:;,(+x<'I&RT#<S,-g= 'C<)-wS()!PnPn!"UL9,-3! '
!g}ˆ "_7A
6IH’')?$<' > !$Lp9!"_!{}¡-3,)-
,"-3(*'LW%)ˆ," 2()L'()$,x!{'+ ,?- M# -U<'CFB  VRT#<U,-
5  $,?5!g}DA
 .9RT#<,-
$<')-w;"s?P W%*'v!g` -I#<;(C "!()()?9'v<I5)"~!Q  B G O! '
 "Np > !-3LLN5,"N)*'}DA ˜  ˜  ˜  ˜

 z Q (+~-3, !g3' "$,


Ž‘1“ # -$,)" cH ()L'()$,
:;L())"N*'5<Ž  "~'+ ,?-UU()?9-
Ž‘¦<  VH + ")"N
!2V()L'()$,x'!P ‘¦ "ˆ$Lp9,"F!?9-3,VV3' "$,1Pn)*-5
()L'()$, <
 xQRT#<,-I <'Cx!&5'),!"FW%!?")~Wsp‘x8q‘ 8a "FUPn,"! '<'(
 A

 €9RT#<x,-12$<')-wV"s?P W%*'x!;,-w,"(* > !-3LLNM,"N)*'-V-3?(+¢#<x"!cO!?9'n!I)P


#CN
-3?P—L9,-3LW#$,Wsp G A
 09
=9',-5 "yl,(+NU?9'"-)8%:;L
=9',-v!,"nm'-wC
=9',-!-3-3)-5@‡ L'()!,"c<C (+c!
*';?9'"-)8:;,$,5l,(@'!$,$,-IV‡ L';,
<IC (+F!&,-I?9'"-){–5N Pn)"-I:;)")L*'

=9',-!-3-3)-
MªvC b8D,"y:;,(+_(C -3 
=9',-:;,"-)8D! '5l,( ~'!$,$,-52B98,"c:;,(+_(C -3Vl,( 
:;,"-)SRT#<,- > '! Wa<W#,$,L[p@#<;l,( M:;,"-+A
B`9? >> !-3@Y k%k+Y   , -
M-3)K?)"()n!= ``NM"s?P W%*'-)8DC +( ˆ!{:;,(+_)K?# $,-Vx! ' 
 Z Z
RT#<F,-@c-3P@ $,$,)w-  > ! -3-3LW#$,~  $,?c!c-3?P ™ Y  Y A „–,-2,-@y-3?P !U $,$
!
> '!?(*-;! >  L'-I!g,-w,"(*;"s?P W%*'-;,"2
-3)K?)"() ¥†
B9•Y 8%Y B98%Y  z 8% "cY BY  BY Y O! 'v}%$`98a)"~:;#<v,-v
Z   
)"*'+ $DO! 'P1?$1O! 'vY  A „'&Q!?9' "˜#-w" :=*';:;,$,˜#$%" W%5,"F*˜#'" Pn-!g}‚
˜ ¥†
BNRT#<,-Q$<')-w=,"N˜ )*'}yO! '=:;,(+M*'fe,-w > !,"N-( k%k ,"@ > $ " "

> !-3LLNU'C $g"s?P W%*')- k %k -3?(+c#<v ,-w+ "() W%*[:=))

"   ˜  "*
  ,+
- 


O! 'v $,-
$ ,. A ˜
/
0
BB9RT#<@,-x"s?P W%*'@!v-3!$,?9,!"-y„‡E&kt†f8•:;L¡ET "itŸ"!"")N<LN,"N)*'-)8!
)K?#<,!" GŠ 0 ‹ 66A

B G   ? >> !-3 > <'+<W%!$5t
@'+Cp9-1t Ei "Ÿt

E 

E

,-{-3L?#<)@-3!
#<QLQ# -•[:=! <'()-Q$Lp9,"
W%*[:=))"
7E&8‚E $ `9_–)-3M[:=!_<'()-V<' > '!<‰w)(*)¢!"N!y@E [7e,-)8


 
p9,)$,," -3)Pn)"N-4  "F4 Q8s:;L24 _!U;',N=!&4 &‚RT#<=,-{v *')"()v!b
   
$,)" - N„¤4 ‚†

N„¤4 %†3A

BN6"x3' "$,Ž‘1“V8

,-S;Pn, > !,"NS!%Ž‘@8:;,$, $,,)-•!"n‘1“-<,-3p9,"


‘

 1“V
 
 Ž M“ 

‘ Ž @8)"!/:P@ "NpM) '))-v<'5,"  ‘ Ž“UA


B.9S ‚„‡Eb†  
‚„

Eb†


E  O! ' $,$%E&8)" ‚„‡Eb†

B z RT#<v<'5'C $b'!! -!&5)K?#<,!"2E 

/€<E

B` 

 E 

/€<E

G 6A
B€9IXD*  „‡Eb† 
„‡E z †  „‡E  7† „‡E 
G †  gda! 'C (+U} 87$,*  
„‡Eb† „‡E 7†  „‡Eb†f
  
RT#<@,-VF-3P@ $,$,)-w > !-3LLN2,"N)*'@} -3?(+¡#<  „7† ˜   

„¤˜ †
 
„BN† ˜

   
 „¤ G †3A ˜

˜

˜
Q

B09IH ˜ 3'+CN)$S )"(*py-3)$,$,-v,(N*-v!c/``M)-w,"#<,!"-)5;/€`` > )! > $,1W#?9p~,(N*-)8b:;#<5,-


! "!
x-3P@ $,$,)-w5"s?P W%*' -3?(+_#<5*'xP1?-w5W%x<
$,C -w )-w,"#<,!"-5')())L9,"2
- Pn5"s?P W%*'!D)-353'+CN)$,*'-+A
G `9Id‚,"¢2-3P@ $,$,)-w > !-3-3LW#$,2  $,?2! „‡E  t  †3ƒ„‡E t†UO! 'n $,$Q'C $"s?P W%*'-1Ej "¡t
-<,-3p9,"E ’t#  `~ " Eqt

%$
B9 „ v"@Pn*!ˆ!!,",-U?-3)- nH'LPn*,( 
'&
£)!Pn*3',( yC "1")K?# $,L[pN8<:;,(+1-w+<)-g#<S,aYak+\$i`98 )" Œ
" 
$ Y\{ "1)K?# $,L[p
( 
!$,-;,g "!"$Lp2,gY \7¥†

 $vX–   –
= $ $
 ``9 H &
–"s?P W%*'I! > )! > $,x„…!?9=!•/€97†S:;!1! = > '! W#$,)P ',NC8 "@"s?P W%*'!
! > m#",-3*'-I:;!x! = > '! W#$,)P ',NC8#<'$,,-w)F,"MW'+ (N*-; *'I5()! '3')(*; "-w:=*'C

67 z 68¥b[;,-U„. B G †3ƒ7



;.N6  /`9¥68,/€
%[;,-5/B  /B 

7 

/B
 6
B9    009¥68, G ¥6 g7ƒ„‡E



E

{–s?-IE 

{–s?-;E 


G 
S /.98, z {$ ()S{N*'3,())-‚!9QW#,v-3K?#<'=<I„ 5k 5†fg–)" =()L'()$,Q,-g())"N*')
< „`98 `N†= " >  -3-3)-I9'!?„ k 7†f•–s?-;'+ ,?-;!D
()L'()$,5,- s 
6;\7k+^7k+Ya .N68, z D
 Ya8#\78a "~^<'x/``< > !/:=*'-;!{/`N G 8 z  098a "_/```98')- > )(*LN)$LpN
.9 6
  /.N8¥u
P1?-w;-<,-3pME u 6 E&
 
z ;.9 .098, G  ,"()x  .98%:=1P1?-w#CN1}.9 ,"()1} } „…} 7†}•„…} 7†;,-

 > '!?(*I!9')()!"-3)()?9LN5,"N)*'-)8!:;,(+<$,C -w!"P1?-w=W%*N)"F  " F!"
VP1?$L > $,!‚B98#}

}_,-; $L:Cp9-IL9,-3LW#$,Wsp B9

€95/BN6  /B`98,/0
 ,"()  z `
 
 B 685!"$Lp@:CpM!1:I'LL -= > '!?(*;!g > ',Pn
,Pn)-I "!F"s?P W%*',-; -v /BN6

09;`9  z B98,/€
b[I,-;$,! „‡+ "=E ()!-Ebƒ‚-3,"=Eb†

$,! „7†

`9
/`9;Y „\  ^  †3ƒ„¤ \)†f B098,/.
•\  „‡E t†  ^  7EqtqV–s?-
Eqt „\  ^  †3ƒv!/:
   
Y Eqtƒ„‡E t† „\ ^ †3ƒ„¤ \)†f

;# €  B9   z 8
€ %"M5  '+ P W%)$,!/: 5<'C1!Dv3' "$,,-Ž ŽU‘ Ž M“
 
“ 1‘

€

.

E

 „G

Eb†fSD,-;)K?# $,-;€98)" E

G 


E  

“
 

             
          
Ž                                      
    

            B
          
G
      ‘
76 /`98,/€
DXD!! Fm'-wO! ',"N)*'-3!$,?9,!"-)v–*pP1?-wW% L9,-3! '-!•B€N6
d‚,"y#<
6x:=! ' 9-)
–&!M-3) #<v,-,-1!"$Lp~-3!$,?9,!"b8%?-3VL9,-3,!"~! >


! L
$ 9
p
 
6 z 
"!Pn $,-
!1-3)#<=E €<E  77E B€N6 „‡E 6†)„‡E  /B<E zz †f8 "2E  /B<E zz ,-="*N*'
     
`9
/B9;B G  0.98, z D–*p<'UB~.n " G  G Q–*pF(C "FW%
O!?"Wsp2m#","@,"N)*'-3!$,?9,!"-
!  Ÿƒ„ †f

      
G !  8, ƒ„3„Bs| †)„Bs| 6†3†fv! •#< „…}D†  "L # " $# "L  #
˜  " ˜ $# U ˜ ˜ "L  # ˜ "L #˜
  


ƒ„3„B }

†)„B }

6†3†f•–
)-3L')~ œ "-w:=*'v,-I˜ ! W+ ,")˜ 2Wsp2' > $ (),"x˜ }cWspy|a˜  
765 G 6 €9  G 098,7 g– > !/:=*'-5!•BM<'VB98b098D z 8b€98D G B98 z  098/€ z 8%*( 
–1$ -wv"s?P W%*'

,"U$,,-w;:;,$,$bW%1 z  098a-3,"()1 z 0x,-IU,)-wB  > !/:=*'$,)-3-;# "! 'v)K?# $!M ``N8

 "y  z 01,-I5,)-wP1?$L > $,! z  01$,)-3-I# "F! ';)K?# $b!@ ``N
/.9 G  B z 8 €
% . /.9/`N6
 GG €`N6Q GG €`N6 0 G . G

/B`B.9{/B`B. 0 /BB9
 
/`N6 z {/`N6 z € 7
 €09g€0 € 

g–s?-&•"s?P W%*'g!9C:<'-&,-&. 0 0 € € 
    
z 6  G   8,/B
{–x',NU3' "$,x,"ˆn  '+ P W%)$,!/: -3!/:;-
#<  ¤„  †  8

:;,(+,P > $,,)- G 6


                                            
     
                                         
      
    
                                        
 
/€9 G €9  0€98,/.
d#'!P—Y \ Œ 8:=U! W+ ,".Y €\7(?9W#-wL?9,"n,-;,"~ G „Y \)† Y\
p9,)$,-U„BN

 G †w\

(
Y\7{–s?-Y "

€x "2)"~\


.9

/09;€9   098,‚ 2,-5n"s?P W%*'U!Q')ˆ-3!(9-)8&)"ˆ > '! Wa<W#,$,L[py#<


W%!  <'V') ,-
„ 7†3ƒ„…}•„…} 7†3†fQda! ';,-I!n)K?# $g 6  G 8#)L*'}ˆ! '} UP1?-w;W%
L9,-3LW#$,5Wsp z 

=} z 8D)" „ 7† 768:;,(+ ,-
"! U-<,-wm#)ˆWsp  "ˆ,"N)*' nQ} €98D)"

„ 7†  `98-3! 6 :=! ' 9-)

 `95/€ ] ! '2/` [Z    `98,7 Q;L1,- EiPn,"s?9)- W%)O! 'F€9ž¥ `98‚)"Ÿ:=2P1?-wV#CNME
   
„¤ ` Eb†3ƒ7~ " )"()ME  `Nƒ/B9y–M! *'V-3!$,?9,!" $,$,!/:;-UM# "- !~W%M,"Ÿ

-w3'+ ,N;$,," {–)"F
"s?P W%*'!&Pn,"s?9)-; *'v€9ž,/`1-<,-wm#)-IE „/` Eb†3ƒ7

5/.9   B098,/B
D ``N
  
 z  /B9 Q (+F‡ (*! ';(C "F#CN
fe > !")"N`V! '
5,"VL9,-3! 'C8-3!
U "-w:=*',-   
  
E   E  ]   €98 B
g=Y~ " \U:=*'V1'!! -!{E 
  

7E
> !$Lp9"!Pn $S„‡E Y  †)„‡E \ Œ  † E „Y  \  Œ    †[E  „Y\    Œ  †fSR 
#CN

6

`98)"_:=1: "N


Y

\

V "~Y\
(
6(,"()
Π

Œ"
Π8:=
! W+ ,"2E 

„¤
( (

 †[E „ 6
 


 †f (  ( 
! (
 B95/``9  B`98,•=@,-w+ "()@,- #8S " )L'1- > ))-V<'M4Q8•4I8• "¢4=8‚')- > )(*LN)$LpN8
) "   8*(  ,"()    
  8%:=1! W+ ," 
 "())$ F " !


Z

(*'!-3- P1?$L > $LpM!V! W+ ," q„

! !



 
 €N†

 
„

/`N†)„

!
 Z [Z  
 `N†f89:;,(+-3,P > $,Lm#)-!@

 ``9 !    !
 G 5 G .98, G S

 /`98a "ˆ/`9
,- > ',Pn {–
$<'
"s?P W%*'}_)K?# $,-v 5,Pn)-
2"s?P W%*'5:;L_ $L*'"#<,"c -5 " ` -)=?9
}Ÿ,-5"! 5L9,-3LW#$, WspŸ8-3,"()V}Dƒ

W%),"-v ``VO!$,$,!/:=)2Wsp G 00V`N `` -:;L1')P@ ,"*';!‚
657  78 z R  #CN B 
! ]

-3,"D„B.`Nƒ<}D†f
=$,C<'$Lp2}
 
}   -3,"D„B.`Nƒ<}D†f8#:;*'
 <'?Pn)"Nv!‚-3,"~,-;,"~) '))-)
7 :=! ' 9-)

 .9   ˜ G B98, G {  B G /``9;da! 'v}%$B98q Pn!~€x  $,? !  G ,- 8
   ˜  ˜  ˜
:;,$,#<!DB ,-v! 'IB9•–s?-  B G (C ""! W%vL9,-3LW#$,WspM€98 "M)"()
(C ""! ;W%5L9,-3˜ LW#$,Wsp_/```9 ˜  ˜  ˜  ˜
 z  B ƒ
N

&ƒ .9  BN8,7 "U  '+ P W%)$,!/:
8a:=
: "N <'Cx!g3' "$,
Ž‘’Pn,"s?-
<'C !%:=)Ž+xS–D' "$,Ž‘,-
',N=3' "$,;:;L@Np > ! )"s?-3
U "
!" $,)~8%-3!@# -<'C 
BNƒ–,-3' "$,
-3!/:;-v#< "$, 
Ž‘ ,-.`n) '))-)8b "
-3!x:=)
# -<'C &ƒ .9

“

        
   
                       

  Ž                                         ‘
           
   
                         
 €9;.9  /`98 .
SR ‰w?-w :=! ' c:;L_nPn! G   $,?)-)8‚-3,"()n,-w,"(* ,"N)*'-
:;L "Np > ' 
-3(*'LW%)-3*1!=Pn! G   $,?)-U(C "Ÿ $L:Cp9-5W%@O!?"b `9k+`9k+`9kCkCkC :=! ' 9-) ? >> !-3M
-3*;!&-3*N)"~ `  -)8 -)8a -)8# "~B -# -I"!xO!?9'-3?PnPn," !x`VPn! G ‚‚<I$,C -wIO!?9'v<'
*N)"b8%)"y*p~P1?-wW%1`98 `98 `98¥V! '5`98¥8¥8¥8% "~*'1(C ""! vW%UmN *N)"b5–U9')
!-Q(C ""! =#CN;W%! @xv "MUB98O! '{)"~8 B98 `98 `5! '8 B98¥8¥-3?P’! `
Pn! G 8-3!
*p
:=!?$,2#CNv! W%18,8,v! ';B98 B98 B98sW#?9)"c8,8¥8 `U! 'IB98 B98¥8 `U-3?P !x` Pn! G •HJ-3,Pn,$<'
<'?Pn)"NC8:;L $,$bPn! G   $,?)-;,"(*'C -3)FWsp_8## "$,)-I5(C -35:;)"O!?9'v<'
!b
 095  z  /B98,/`
qXD*3," )"! vv*N)"N=#<U"s?P W%*' *')"NI'!P BU,-Q'!$,$,)b8s)"
l,(n:;,"-Q!"@vO!$,$,!/:;," -3)K?)"())-)ž{–IB98––IB989–––IB98 f{– > '! Wa<W#,$,L,)-Q!
)-3U<' 8')- > )(*LN)$LpN8  8  
    

8  „ †  k f{–5-3?P—,-  ƒ„
 
† 7ƒ„7

6†f 
/``9  BB98¥ 6 ;„Y
B`9

Y 

  
Y   †
ZZ 
Y 
 ``Ny–s?-1!?9'1)-3L')¢-3?P )K?# $,-  „


 ! 
Y Y 1[5:;,$,$• $L:Cp9-W%V3'?1#<
Y † /``9[- -3P@ $,$,)-w > !-3-3LW#$,
  
  $,?5:;,$,$b!()()?9';:;)" Y  `9

B9I}  }   76¥68 €9¥ 6 g{pN!?ˆ()!P > ?91x"fe9UO*: *'Pn-)8pN!?ˆ(C "ˆ! W#-3*'3NV#<

 
-3?()())-3-3LNV *')"())-5,"(*'C -3 $,,"C<'$LpN1'„ &Q!?~(C "~)" > '!/NU,-)8%,‚pN!?~:;,-3b¥†~–s?-
1O! 'P1?$x:;,$,$DW%1K?# 9'+<,( •Y Žv}  ‘ } “V8q)"_“ 8%Ž ‘ “ B98
   
 " G Ž  ‘ “ z 8'!P :;,(+˜ !"5)?())-vŽ ‘ “ 
 
BN G   G 8,
=L'()$,)-;())"N*')c<    :;LF'+ ,? -  :=!?$,W% >  L'3:;,-3
fe9*'"# $,$Lp2+ ")"N;!
C (+! *'C8a "F,-I(C "!"$Lp2# >> )"F!n<;Pn!-w G ()L'()$,)-)
BB95  .98,/B
;„¤ Š B ‹ †)„¤ Š B ‹ † 662,P > $,,)-5)L*'1 Š B ‹ 6F "Ÿ Š B ‹ 8&! '
 
 Š B ‹  "F Š B ‹ 66•–m'-w(C -3v-Cp9- Š €  "2B ‹
B98:;,$,;v-3)()!"

(C -3  -Cp9-v Š  €n "cB ‹  z I–
m'-wp9,)$,-1„‡E&kt† „ B9kC7†f8:;,$,5U-3)()!"y!)-

"! ; LN
 ","N)*'I  $,?5O! ';E&
B G 5 G ¥8  ;XD*MYh \MW%'!! -n!E E  E 8I "j^ h W%'!! -n!
 !
7E

E 

E
 SR )-3L'1„

!
\)† „Y


^)†


„


^)† „ Y

\)†

„¤

† „

†

 !
BN6;0`9  098,=XD* )"! M > !,"Nx!;,"N*'-3)(*,!"¢!;Ž  "¨“ ~8Q "¢$,* W%M 


P , > !,"Nn!‘ @j–)"¨Ž 


n  ~T–D' "$,)-
 1“  " U“ <'-3,Pn,$<'C8{)"()

“ 

 
Ž V –s?-


“5Ž 
5Ž“V8I "¨-3! @Ž“

 @Ž 

5Ž“

  
  @ Ž  
“5Ž 8‚  " ˆV[:=!~-3,)-U!Q,-
)K?#<,!"ˆ-3?P !ˆ/€`98&-3,"()n*pˆ<'x

 "$,)-;!g13' "$, • – s?- @Ž“
0`9
B.9U„‡E  7E 7†3ƒ B9
  68 G   > $ ()EFWsp~ 
EF! *; ‚„ Eb† 
‚„‡Eb† „ Eb†   ?9W3'+ (*

,-;)K?#<,!"2'!P¦ ,Pn)-=
! ',,"# $b)K?#<,!"2!V)?() B ‚„‡Eb†



7E


„

Eb†  
B z  

 0 .9   8,DXD*t

 E 

/€<E

G 6s–)"yt 

76

<tq8q-3!@t

6x! '

B98aW#?9t
(C ""! ;W%5")N<LN {–s?-;E  /€<E G 6 689p9,)$,,"V5()$ ,Pn)2')-3?$LC
 

B€95/09  `98
` Nd‚L'-w&"! S#<D,9 > !$Lp9"!Pn $ ‚„‡Eb†%# -D$,C ,"I*'PY š E š 87)" ‚„‡E 7† ‚„‡Eb†  
# -v$,C ,"n*'P |Y š E š  )  ,"()  # -v) ')@/€98  :;,$,$D#CN ) ')@/0



 vfe9
"! #<=   
,"nv()!"()$,?-3,!"2P1?-w-<,-3px#<=˜ > !$Lp9"!Pn $ 
„‡Eb†


 „7†{# -
<V$,C -wn G '!! ˜ -)8{ "Ÿ-3!_P1?-w W%2,)"N,(C $,$Lp`98•-3,"()2L :;,$,$Q"! V#C˜N2) ')˜  $— G 
–s?-;:=5P1?-w;#CN
}

/0V-3!x#<  ,-I()!"-w+ "NC
B09;B9  €98¥ +? >> !-3cY   Y [ZZ '˜  > ')-3)"N-@~"s?P W%*'M!v3'+CN)$,*'-n!,"ˆ!ˆ
 
 <',!?-)-w,"#<,!"-)•"!M9')VY  <' )K?# $8%)"y1-3)K?)"()~„Y k %k+Y †;,-)8b,"
 [ZZ
C (+ > !-3L,!"b8 $i-3)K?)"()x„`9k+`9kCkCk#k G 09k G 0N†f8 :;,(+M# - G 6 `53'+CN)$,*'-){–s?-=L
,-,P > !-3-3LW#$,v#<"!V9'))-w,"#<,!"-I#CNv5- Pnv"s?P W%*';!3'+CN)$,*'-)•ªv!/:=*N*'C8
•-3)K?)"()v„`9k+`9k+`9kCkCkCk  k+BNk+BNk+BNkf/.N†f8f:;,(+
-3?Pn-b!/€``987-3!/:;-DL,- > !-3-3LW#$,
#<;"!xPn! '# "29')
)-w,"#<,!"-;#CN
- Pn5"s?P W%*';!&3'+CN)$,*'-)
G `9  .9 `98 `
%Rˆ'L Š ‹
Š " ‹
E

t
 Š ‹
 ŠC‹ •H >> $LpnH£ ~{:;LFY

E

&
t@ "F\
Š ‹
 ŠC‹
!n)?() Š ‹ $  Y\
Š " ‹
  .9I  
 –&!n-3)U#<)K?# $,L[p2(C "~W%U! W+ ,")b8a:=UP1?-w;m#" 

Ey "2t@ -I<W%!/N-3!1#<IY

\7• – s?-:=v:;,-3M!1-3!$LNvE

t  ŠC‹  "MEqt
Š ‹
B9{–,-
(C "MW%')?())2! E  78 "M)"()E  .  z 
=!!-3E  .  z ,"  
! '*';!x#CNE ¨t i`9 
 Š

 

      "!#%$ $ 
&(' )+*
,-/./ % %0 1
2 34-562 78 195 1 
2:34  <;= 1 134-5 1?>@ 5ACBD 5E F 1
G>H I7JLK6MN,POQSRT%UV 5W D X5E F 1 YZ>H [AJLK6M
OQSR\%
]D^ + ( D 5E F 1 ?_7` YAUQ ba6 1 15cd Le VXf

g6/h^iDS;jY k2lk 1m 1n0 o 1nQp5- 1n D 1 ( ba6E V / e m V;D 5 1
D^ %0 l 1m D:,q$ UQr-UQ 5sptf
&
u /vq  /wLxy"!-"r-UzxySgk!-S{6U 5:|xySKk!,g6n
9 D    1w^UQzSUQ 5:|
s1 % m5 1}#Y  1E ( e m 1E 
*
r->( /DS;~Y k2X4k 5- 1 D m D

€ x#‚p &„ƒ r"p ' { * #‚p &„ƒ K+p ' { * #‚p &„ƒ†… p ' { *

k 1 1  D ?p‡ˆ "a6Ff

K6/./S;~Y k2X^ l#‚pŠ‰ €  + E‚2s34 Dsp x~,L 5p & x u f


* ' € ' €
… /./S;~Y k2Xk 1m 134  <;= 1 1WSRR 5ISRRR  Po
 1= … f

‹6]
D D oX5 1 
2S0 1D % 5CU l;=Y YD^ 5 1=D 1 1
q;
DX l^  
4 5oV,/ 1  „ ŒD (;( ŽV 9m E2Le %; o5 911 (4 5oŒK
 1  %BD„m „PoE „34 /1E F k =omDk %U- 5Y} k2 1 ;DFD
0 1 134 1 % + D D L^  5 19E F k 
2}./S;Y k2:Žo^  L 1  
5 % 5dm ;t 1o
bf

{6]D^ + / D l4 1  1 mDk  e m 9;D l3^  }D^  1m Dcr 5
;D Dk2-4 1Žo }/,Lm % + 1( D^  D ? D55 Sf

SR6/‘V FDXC’}UkpŠU € U- 5X“} 1Žo^ =Rl”• =„ŽS;Y D^ + ?#– 


tpx—R6UŽ D 1
*
€ xy˜i#3 * 
 € x™R6U D 1d’™x™“6˜4 5W# * 
n“Px™R6U D 1:’—xy(š-o4
€ x™R6]D^ + /( D ?0 o ?n’ ' p ' “kf
>™o 15s% =  1Y X5 <;=L%  5X 11 1
0 15:›‹ u R„ % FD% 1Vœ9
 D 1 l    2- 15 15 u RkHž / D }5 %  1P#1^  15d 
DoFD^  }1 U^;D 1 % / D l D 1 ok 15 X Y,rŸ1=]D^ +
*
( D 5 %  11    /ž }# s D ? <;=    Ff
*
",->y34S;i1k F 9rR1 15:3^  Sgm 1 1CUiSR 15CUt{3o U4‹2 1S;UtK
3e FQUt 5 u ;D
”•2o  }3565 15d /2ocFs3^ o6  D
34S; UC;D^ + l9 D Y  1E ŽoP34 1=3^ 2o[PoE lFcI5 1? 
mo^  k 1 ? D^ + 2odD^ %0 } + / % E … 3^ (i D   1Ff


Sg6”•V7=q 5 : E2d % %0 l D 1Do 1? +  D }    U;( Žm5 1 
2
S;( 5 % FD D 11UŽ % FD +  D 1ŒS;1E F k V + U D 2P;- 1  „ +‚ 1

rdŽo6 11:”•7=Œ % %0 1g:Žo6 1}e + 1 D^  : E2Un D 2W 1  L +‚ 1
D D^ ;(  15dgŽo6 11„./S;~Y k2sŽo6 1(;=o5d
 F  lDŸ 

;( X t D ?;( %2sHDDo   : E24  Ff


u B(;=– n1V  ^4 15Po
F  1o
2iBDŒŒ5  14 % + 15
2Lok 
+ } % E l Y( D 1}1 1loWD % 51UΠ+ ;DFDW4k l D -1 1
E 1]D^ +  D }3^ 3
<2Y D^ + 934 D:1/%  locD % 59d D

e E ^Qf

"r-/vq  i #‚p * x—wp & ' z p



| (1 1E F 1- 1 1k /w^U^zSUQ 5d|"Œ”• i#ˆ xyU
/
 # , * x™,-UQ 5 i#–g * x
i
'

u U;D^ + / i# r f
*
*

 
SK6]D^ + ( D e m 1E 0 o ? p & ƒ u k 4pc + E‚2-m p  '
, 
R6M
"f


…  ? 1m   (% (m  gP  1„4 1mk 5omLDmDk;( %2X5
0-m} D^ 

P5- 1P5om 1
<25
0-m[œ9 c 11 1k L CUŒD X5S0 I",d 1}
D IDmDk;( %2— 5y"rR  1s D I1
<2] 5—o 15~ ba  
28SR mk 
mk   ./S;HY k2 1l4 1Pmk [5- 1LD}% Pm  L5omc1
<2
5
0-mŽf

S‹6/hn
0 ( 1n^   1k 9 D 3^  (t  e m „5
0-5 ( % FD^ D  D 15 1
k } aX 1Žo^ t 1m 1k  5X D 9  % }k P aY5E  (^ E 1Œ”•q D ?  %
Š D e m 1E Š D 1 l^ E gg6U- D 1;D^ + ( D   % LŠ D m^ 
e m Sf


S{6>Y 1Žo^ tŽoP34 1= o= 5Y 1= 145 15X l D Žo 1E  9 
  
2os Y + DQfU P % FD 145m};
D „ 1 /Œ”• … Rk q D ;D
  
 5 „ 1=;= 1 ? 1 5s‹Rk q D ;DL 5 /P;= 1 o1U- D 1
!
;D^ + /4 11 1k F m i D  14 15s 5 „ 1Ff

,R6" 1% ΠD^ + l D Y


1 + 154S;= 1Pw$#&%l % Pw(')#&%+*<d”•(p†} : % VŽoP34 1
;DFDd + Ež^ 1, &+, u &+, x~rK6U6 D 1;D^ + /( D ?0 o ?Œ, &.- , f
'

,
,-BD Y  1E ?4

0 k 1m 1 ;
DI D 4 1E <2 D^ + }g5
0-5 1 U u
5
0-5 1 U6r5
0-5 1 ,-U6Kl5
0-5 1 g6U … 5
0-5 1 u U6‹l5
0-5 1
' ' ' '
r-UC 5 {Y5
0-5 1 K9g6/]D^ + 9 D P ba- ?e m 1E k 1m 1/;
D
' '
D4 1E <2f

,,-BD }5e m^ V Žo^ 5e + 1 q  }4 14 151oe 1(BD 1 P
/5 11U
  5 11UqD^ %0  1m D,-Uig6UŠ 5 u P]D^ + }  P D 43 P0 o 1
( D  1m Di D ?oE D5 Sf

,g6BD }5e m  34 1S;81E / D 1 }Žo^  1? 5: 11 1 mY1 1ˆ‡
F L0 1E 1 1= V5% + 15CnBD 9 m 1 D^ + V D /e k m? 1VY  ;
D
D LD 1k F Π L5 1 153k 2 @  5 YP]D^ + l D  5e   % o
 m
 Lf
                                                                                                                    
'                      
                       
            
                                       
                           
                                                                                                             

, u BD 4e 1-5^ + 1/# "‰


C ?4k „  
Œ5E F 1  D m
*
5L D  m
 D 4

0 (p‡ˆ "a6Œ 
Œ 5o0 1 1Vvq   ~ 5
 D^ %0 /4e 1-5^ + 1#–‹6‰br i!-", 5c#–‹6‰ ƒ g i!-", Uk 5   34  D
* *
54k (i D  1m 1k  ]D^ +   ? D 4e (1-5^ + 1  ~f

,r-/h^ % FD4k #  ‰ P D e  U;


DH 5 k 1m 11UŽ5 %; 911
*
 5ol"!# ,   $ 
g    $ (]D^ +  D l  F Š  %  11 15c3k2d C D 1
*
11 1Ff

,K6A=5 1 D ?S;mr,P1oE0 1

D },-?0 1E % q 1(px"!#!nx ƒ SR6‰ ƒ 6 { ‰%M%M%MC‰%SR6˜


D },-D1k F C 1 € x%$&$Lx ƒ SR6‰ ƒ {6‰%M%M%MC‰%SR6˜
D [SR 11 11 1k 1 15` +  D mW;
D  5o(' $ (' x
' )
R6‰%‰%M%M%MC‰F{6

/L D 1 n D 1oE0 1/k 1 1 d L1s4k %]D^ + /( D ?  F 
ŽoP34 1n4k nk 1 1 di D 1 1oE0 1Ff

$+*,*
, … /‘0 o^ +  / L    -.

u !& ƒ
/ $

g
,‹6/vq  /w x~,R 5dw & x™Rg6Vh^ U5 ž^ }w x—w w Œ]D^ + 
$   &  '   $
D  1Y 5 1=;D 1cw & w && &
OSOSO w & ) 5
0-5 153k2d‹kf
$ ' ' ' *,*

,{6/h^n D  F 3 (34 1S;Uk2o% E F E „ + „ k2L   11U D 1S0 D1k F 
2
i0 1E % 
2l (   1 + V /  Uon5S;n ‚ mDk %n./S;jY k2
5 t 1 1k n^ + Dil D V F 3 Œ;k4 1k D V;=5LvŠ‘V./” .lUS;
Dl D =   1 
-11oml D =1 1 Œ5 1Ffh^i ba  Umm3^ Fk;( 5V m D
34  H1ok  ?;( %2

v ‘ . ”  .
‘ v ‘ . ” 
. ‘ v ‘ . ”
” . ‘ v ‘ .
 ” . ‘ v ‘
.  ” . ‘ v

gR6B(;=cY + FDm5 1FŽPD^ %0 r,s% 5} % FDC:œ9 DŽo 15† 5o6 P
Xq D 9 115C„h^= % FDs% 5sq D  s5 1FQU6;= ?1ok ( D 9ŽoP34 1
4% 5Œ34  <;= 1 1Y
Œ 5L D 1 145m% 5LQ D 34  5 1F#
1o5m} D % 5 D 1 1
0 1 Œ]D^ + /( D o_i D 1 ŽoP34 1Ff
*
g6/vq  34 s c1k0 ba 4
2-mWoFD† D^ + %UŒ
5 1  XŽoP34 1 15 1 1b‡
o6 
0 1
2W‰%M%M%MC‰ Uq5 1 !  5 ! ,Y 0 1k o^ 
2 k 1 1 ;D 1 ba- 15 15
' ƒ
34 25†5 ! j#–h^ !lx s  U ! ,:}k 1  15 ss
' '
,- 7„2mmd % FD ^ l5 1 !9 5 ! ,dok V D 2W 1  %UŒ 
* '
‡<4k 15E F 36 F  15C]D^ + ( D loG#5 1m 1 1 i D l m 1
+ 9 D 4k 9„ D LE F Ff „o? E;= 1Y %2c34  YPoe Yk0
0-m  *
 l o r-#> ba  ?;D 1 x~rL 4 % 34 1S;
*
         
                 
                          
   
                             
 
        
                                                                                      
  
     
   
   
  

u
gk,-/vq  Ÿ5 1 P D L4 1  1= XmDk  e m UC 5  D Lo V D
Žo^  1
P D 1 5 11 ]D^ + P D 5 1 15†  X1- 1 1k 1U

 ' &
# ' ‰ ' & ‰ ' ) U^oFDd D^ +  D }  % i D  e m 9 1Žo^  '
& 
$ * $ ' '
'
)  f

gg6A=1 1/;
D 1 1k 1 + }U YUq 5 H  } ba- 1
2d F m 1k 9 X L -‡
 D 11BD  5o„C D 11 1 1k 1 15 + —=g6U6 5 D^ + =C D /11
1 1k 1 15 + ™r-n”•^l e m& 
 U1" #  xy"!-SK6U";D^ + i1"+#  f
* *
g u /./S; Y k2k 1m 1„o6 #‚pŠ‰ €   D 1 V D S;m 1Žo^ + Qf
*
#‚p ƒ u #‚p ƒ SR x™,
* *

 
gkr-A=DF 1 d /omm  (1 Y^ F  iK6U6{6U-,R6n]D^ + (V D e m 1E
 
ŽoP34 1=iA=DF 1 d /omm  = D^ + „2oX%  (3o623k2Y1P3m?0 ˆ‡
o^ F  Ff

gK6/h^ % FD`5 d j 1Žoe + 1 ( e m s?5 d 1m D ,-U [Žo^  :
5 %;1k F m} D 9 e m  5D^ %0-mP D^ + 5 ?  9i
( 15m 11
]D^ + / D l5  1m Ddn D Y  1E  e m 91k F mL D 1 l D 1
Žo^  1Ff

g … /h^X k2` 16 <2`o3  dl5E  s4



0 ck 1m 11U/5 ž^ 
s  ‡
1^ + mWo †34 : D o 36 F  15 3k2  mmW
XŽoP34 1s
5 11 % m5 1/ 5 D 1CU^34 1mmP;
D D e m 1E %U^ 
1^ + 1
2X 55
5o36   o11 1
0 }ŽoP34 11=h^ ba  U6 D l 
1^ + mLo 
 
‰b,-‰br-‰FK /„K ƒ r   
, ƒ x™,-U6 5 r 9
V oE r-i]D^ + =ΠD /o
'
i D l 
1^ + mPo C 16 <2so3   ‰b,-‰Fg6‰ u ‰br-‰FK f  

g‹6/vq  #‚p
*
x]p )
'
u p &
'
‹+p
'
SK6V‘na6 1


#‚p ' * ' *

… ƒ #‚p K ƒ #‚p r
' *F'

#‚p u ƒ #‚p g
' * ' *F'
#‚p ,

' *F'
#‚p
'



*

ƒ #‚p
*
 d ba61
 94
2-e  

g{6/vq  #‚p x p & 'Žp:;


D ' L % CŽoP34 11Œ]D^ + ( D   /i0 o 1
* '
'/;DFDc D P 1Žo^ +  #‚p
*

xRX 5 # #‚p


**



xyRYD^ %0 L ba  


2d D
  ? % qo6 s  Ff

u R6/hn5 C % CŽoP34 1pcoFDd D^ + px  p ƒ


$ '  ƒ
$ 
r

   !
"
#

/K%$r-”• /?"!# )& && 


' *
,-r-M u {9OkSR\%Œ”• /l#–K6M
 ƒ R6M Kk, SR\%
*
g6r-Vš-1  D ?oHq <;=5 1(PoE 34 9m % + 1= D^ X D  D5CU-pd% s34
g6U u UQr-UQK6U … 

u /zS‰F|"‰Fw^VwLxyM … g6U^zxyM K‹6U^ 5:|xyM K{6


r-/g6 /  D^ + € xZR[
? 5j
2†
9 d9 D d–   1Žo^ YR6]h^
D Y D 1 s–  1UVz & ƒ u wŽ| 1Žo^  ƒ UVR6U„ 5 Sg6UŒ D^ + P D 2WD^ %0
 14 1 
0 1
2R6UqU^ 5c,P- 1

K6,-Vš-o36    D ? 1Žo^ + ( Lm  € & ƒ € x~,-BD 1 € x™,} ƒ U


5 % FD:i D 1 2- 15 LoŽo ?0 o ?ip: D^ + ;=Ž1
… ",‹6(SRkr}( D 9ž^E %SRkr ",‹O … xySRR6?= -Le m UL D 1   L", …
'
  +‚ 1?SRkr-

‹6 u ‹ Œn”• V;634 ( D /  % 5  ?11 t 5oV‹9 1  V;DFD 1Œo6 5
L11 1k 11 ?n 5o u  1  %VBD„#–‹ & ƒ u & 
*
{6 … $  () ' ™”•?p]5 1 1Y D c D 1Y 1mU( D 1 #‚p , & xZp & r & U
& & … x &$ …  ' * '
u p u x~,r-ŒBD l4 1  1(/,"p
' ' &d'
SR6/g6l#–  1(p xJ}#3  1’ x8“6/”•V“x R6U^ D 1c3k2 # U4* ’ x ) “6
* * *
./ 11 }+ “ x™) R6ŒBDŽo“}xyl 5 D 1d’—xy
› u RP1V./(oFD^  l1 9p:s D 9ž^E % PoE  + E‚2cM u px—‹ u R6
BDŽoPp`x KRR6 š-e 
2UŠD}oFD^  s1 X D X 115†% }PoE
D^ %0 :34 1 1 ‹ u Rk!-M … r x ",R6`BDŽoXD :^ 5`›- … ,R[ 5j
2† 11o4 15
›-SK‹R6


",-/gkr- „oYmDk „FD- 9Km 1 11UKl 151UK3o 11UK?2 1S;1U6Kl3e FŽ1U6 5
u ;D
11VBD  ba-  ?;t1 1E F 
2Xm
0 92o … n ?11


Sg6"r-vq  , - 5., / 5 1  D Y4 1 15P7=Π5 : E2Un 1C
UV 510
D ?5E F 1 34  <;= 1 1d D 1Do 11„BD 1cr2, - r2, /yx30Px™g%, - K%, /Y
' '
./ 114
, /yx~2, , -UQ 5I"r2, -c3 x 0^

K

u  $g6PBD  0-o ^   1 0 k %Lœ„ D Lo? 1Žo^ 
2c 1
2:o6 E‡
1 1? s5o3 ^C 
 Ui..lUŠ./B?UqB.lUŠ 5[BB?UC D ž^E  D 1 D^ %0 Y +

 % E  D % 5CUQ 5 oE / i D 1_D^  D  D 11d  LD % 5C

"r-sh^} dŽo^ 5 + Ui5t 1 11 1P34  <;= 1 1W0 o 1} + } 1Žo^ 
2 ^ 1 15  -‡
 
1E0 l1 %  X % 
2:š-1 i# , ƒ i#ˆ x s 5 i#–g ƒ i# , x -  , U 
  *
! *
;= LPoE ?D^ %0 i# u ƒ i#–g xgs 5 i# r ƒ i# u x u UC 5 D 11 i# r x  * *


i#–g g
* * * *
u x d>
1^ + 
0 1
2U
0  D X 1Žo^ + Pw z
*
|Yx U
u w
'
*Œ'
,z
'
'
|x™,-U6 5s{w
'
gz
' *
'
|x u }36 F  D^ + i#‚p x $ p & ƒ $ p
&
 '
& '
5s D 1:o3E 
o6 ?px~r-


SK6/R6 u X / [BDP3 1G;( E F + 15CIBD sk 1k P;( P  D^ %0
 
p , R6M
iBD 1P D  E;= 1n;=o5PD^ %0 (34 1 1R6 u U";
D} D „S;m
'
o6  h^ ƒ ,-M
p  
ƒ M {6U p & ƒ u 6D^ ? YPoH0 o l„RX +
p†x ƒ ,d 51 %  1} 2oWS0 [ 1
D 15 1 CY”• lY "a6Po
;-11oV 1
D 1= + ƒ ,-M
/ ƒ M {6Vš-1 /
V 1Žo^  p    
, p ƒ , NUk + V D 1
'
4k 
;C 1Žo^ nR6M
  1 u 
}/g6 {6(>/E F + 15CUQ;
D p
'
, —R6M
U (
IY "a6Po  FD 0 15CjBD 0 o s/ D o †m   3
 
2
1  XR6 u /7= Dc E;= 19;= 1 L 11 16 15I /1 1 %UC m;
D:oFD
0 e + ( /R6 u R{{{{6M%M%M
… ,r-”•Špd5 1 1 D 9 1Žo 1E 15dŽoP34 11U6 D 1 $<$ & $ ' * xySR6ŒhQ  m
o6 V l,-U D 15o1 1V R}x™r"p & ƒ SK+p ƒ  )V' 
,,r-ŒBDŒ% 34 
0 153k2P D
Žo^ 5 + Poe 6Un 
DomD[ D YŽoP34 1L     <2e m ”• P% W34
–   15X (RlxJ# r"p { #‚p ƒ ,r Œœ9V2oXmDk „mo 1„ D o6 
' * *
-Žml + = D m^  1Žo^ + CU- 14 11e 
2 +‚ 1„
=D^ =34 1 1X5
0-5 15X3k2
,-

S‹6SR‹6ŒBD l4E si D ;D 9 e m 936 F  153k2 ba61o5mL D l34 E‡
 ^ E YY [ e m se }  D d;D U=;
Dj5 1X2 r $K[ 
e m LBDŽol
  % l,2r $gKX  19 D^ + l„ D L;D BDŽol D   %
Š D 34  Ÿ4E s? $gKl  1( D^ + i D ?;D „š-L D   % L
D ? e m gg?O"gKk!-

S{6/‹ u Œvq  $c# 1C , 5 1 9 D 9  s o# 1CV 1 *  %2-m
 *
„ 11BD 1 $x ) ,UQ;D  ƒ $x u #ˆ ƒ ,


*
BDŽo} ƒ , x
) 


u ƒ u ,U4 5
D 11 (& ' ,9x—g6U^L D^ + ,x & $ 
&('


+,
,R6",‹6?”• € xJ, & Ut D 1 €
' €
& xJrK6Ut ;DFDc;= L5 15o1 € x … BD
5 1 15: E;= 1//, 


…
,-,r,g6 ƒ gYPoE 934 P5
0-3 3k2cg6U u Uqr-UtK6U … UC‹6UC 5 {6BDŽo?
PoE
34 ?5
0-3 93k2d‹9O"{?O+r9O … x~,r,R6M

,,- = ,-#–7= DP  = 1Žo 15C   1i  D =5e m ~34 1S;7= 1% o "
D  e m 1/  EmDk %U Yw &  '
 *
| & x~’ &
'
p &
' € '
& “ & x8z &
'
0 & BDŽo
D c43 : 1m D 0W D :oE D 5 : + E‚2 0 x , & & g & ƒ u &
'
#43 /, & u & ƒ g & g & u & ƒ , & 
* ' '

            
                  
    
    
    z  
w    ’ 
     
    
   
                   “                                                              p                                                
  
                   

0               €                             | 

,g6#i! u  F Š#   x #‚ F Š#  F Š#+ !#ˆ ƒ F Š#  F Š#+ x #%$


ƒ ' ' & * '& *„' ** * ** &'
$ !#ˆ $ Ot$ x ! xy
) * & ) *
, u }# u ‰ i!-", i”•  D /mCU+ D 1 e m  Œ1 1 11Uk;
D x 
 *
",R %VBDŽo D  m  D^ 
‚;( %2X34  <;= 1 1 ƒ g i!-",L 5:ri!-",-U 5


D  1m D  ‹Œ1S#–KR 


*

,r-,ri! u gk,-”• [  1

%# $
 '
, #Š$

'
& & #ˆ"!#ˆ
-  $ ƒ '
OSOSO %# $
** \ '
,#Š$ $
\ ' \ '
& #ˆ"!#ˆ ƒ $
-  OSOSO
**
x #%$
' $
&$ %# $ &

$ #%$
** \ ' T $ \ * *
x

#%$
' *
' O ' & x™\ , r!
'


& *
x u kg ,-M
$& )
,K6/‹ u Œ‘V FDsq D ,-0 1E % t 1= 1  ( % FDsq D ,-9D1k F 4 11U
2- 15m/,-6Oe,-/x uu „4k Šk 1 1 CA=5 11U+q D „11 Œ;
D
 5o ' $ US D VŽo^ E 1Š34 1mm D V4

0 np‡ˆ "a6i 51k Žom
' )
od U43o6  1o5mU D l4k /;D 1 }
/ 1  / D }4

0 € ‡ˆ "a61
”•  1  (  ' D 1k F = 1L 5 '0 1E % ( 11[BDŽo D X;D
'
11 D^  u # , ' k 1 1 1dš-oms D}  'Im- 1GR 
' *
{Y2- 15?‹^#ˆ OSOSO { u O6SRXxJ‹}O u r u RXx u RR6lBDŽo? D P  F n
' ' *n' '
uu u RR6
'

‹
, … SRRk!,R6”•  1Žo^ 
$+*,* $+*,* . .
-.
x -.
$ #]$ ƒ $ x $ #ˆ ƒ $ x_$+*,* ‰
# ,! ƒ # ,! & & $ &  $ * & & *$ * & *$
$ * /
' * / $

1  Ž 1Œ34  <;= 1 1d 5 $ -11o;


DP34 DL4

0  5L 1mk + 
0
& *$
m1
,‹6 …   BD }-5c‹0 o lVw & 5 14 1. 59
2: D l-5 u 0 o lVw^/BD
-5 u 0 o 1Š D ?o11 1
0 Pw  R6‰Fg6‰Fg6‰b,-‰%‰Fg} 5s D 1Žo 11 l
 a:ŽoP34 19 D 1  14 % +  0 11?BD P-5 ‹0 o }Œ D Po Œ D
Žo^  1„L DV 14 % + m3-FPVR u (x—R6V,RRg?D^ „ggg
' ' ' ' '
C D 1 3-FŽ( 5Y D 1Y D  ba- rŽoP34 11U-2- 15m}R u 
' ' ' '
,{6/K u cBD sŽoP34 1P/^ + DP 15mI + } D .Ÿ D s34  G ‚ P1 1
1Žo^ ( D ?ŽoP34 1 15mLX D 9 smDk 1 11UQ 5sL;= 1ok  D
e +  1l 5W5o3 
%X Ye 34 1„ d   1;
D D ŽoP34 1} S;( 3
15mI^ + D34 1mmc;
D D^ +    11 BD  4
—#ˆU K  
 
e 34 1 15c;
D Ui }  L D L <;= L  l 5 1 1k l 
%BD ”F  lI4

*
#ˆU u U?# ,-UNr U= 58#–g6U K ;(34 de 34 1 15 ;
D™U,-U( 5—U( 14 1 
0 1
2
 * * *
/  D^ + = D e 34 1Q 1Žo^ „ D oŸC D e 34 1VC D 9   1VS;m


(;DFDc  l 5 1 1k  
%VBD 1d D e 34 1(n D ?o.l    LUQg6UQg6U
5 U6 D ?Š D 9ž0 ‘  U u U^K6U u UCU 5sž^^ 
2 D ?Š D ? asvŒ  
U^r-UtSR6UtSR6Ur-U^ 5 BD(o #ˆ r SR SR r Po
 153k2
' ' ' ' ' *
,-U6  11ok /( D  D 15 1 CUo5 1 15c E;= 11
.

gR6,Kkr,- vŠ 34 1 D ^ 9Y + FDmI% 5d‰b,-‰%M%M%MC‰br,-jvq  Yw 34 s . D

-% + 8 D (34   ^ D ( „^ D „ L% 5Pe 34 1 1. 5 !UŽ . 5L. z D
-% + l^ D (34  y% 5Le 34 1 15 !i (;D} 9ž^5 '<& #–w ƒ z ƒ 
/ $ *

BD 1Žo^ 
'<& . '<& .
-.
w ƒ -.
z ƒ r,lx# rg OSOSO SR u ƒ ˆ# OSOSO r, ƒ r,?x~r, &%ƒ r,lx™,Kkr,-M
' ' * ' ' *
/ $ / $
g6}# ƒ u S‹R6BD 1 ?  e m 1V ba- 1„ l D 4
2-m;
DX l0 1E ba
*
+  }0 1E baŠ D ?E F 1ŒBD ?  Š D ?3^   m 1(n Q D 1 ? e m 1
 C ba- 1 m 1i D 4
2-mCŒBD lo_i D  ba- 1/ m 1
4
2-md/gKR5 1m 1 11U43o6 / D^ + 
2s1ok  }n D  <;=X ba- 1
m 1i + i % FDl0 1E batŒBDŽoi D Vo 6 D „3^  = m 1Šo e m 1
 … ,R6UŽ;DFDX;D 1so36   15s S‹R 2- 15„ D 95 1 15 m od

{
gk,-}#%$ ‰ ƒ $ ,-‰FR n”•ŠwP 5Xz   D / 1m„ 5Y|( D /Dk2-4 1Žo U- D 1Xw
  *
z & x ~ 5CUkom D 2Ž D^ m % BD 1 1@ mk CUŽ,#–w & z & x
' '
z
w &
'
 [BDŽow 
zsx ƒ  !,-Wš-Žo^ mc D}2- 15Lw & z &
' *
,wŽzsx
& ƒ ,  !,
'
'
 !,-ŒBD 9 1w &
'
' '
z & 5  !,l% 1 1  / m} D^ + 
D l  % LY$ wŽzSU^;= 36 F `$ #+ & ƒ
&
,
 
*


gg6 … $‹6vq  5 1  D Y 5ol( D Y11 1 1k 1 15† + dBD 1]#  
r & x ‹ &
* ' ƒ '
#
g & ƒ ,O‹^ # 
* '
g "!-SK6 BDY 1  x  /S;
*
'

u & x—‹ & K & ,9O"K?O"‹Œ1S+#  U6;DFD: 1=1S+#  x … !‹6
' * *
g u ,-ip ƒ u 5Pp ƒ SR?PoE Œ34 ,S‡<4S;= 1n D^ + Œ5t 1n3k2LK6Un D 1n 1mk + 
0 11
BD Y
2IoFD†  s,d 5 ‹6Un ƒ ,: 5 ƒ ‹6dBD2- 15l D Yo6 
€ x u 5p ƒ SRlx ƒ ‹/,-
gkr- u 6 g :>k2Po
 /gd;DFD lm % + 1} D^  g:%  34 Y36 F  15
^ F  }(K 5{6Yš-1 u g 5 u g ƒ ,R   lPo
 1g6UŠ 5
u g ƒ ,XOQ,R X - Y / † FD 0 U u gW%  34 [ FD 0 15C Bi† 1
D^ + ? k2cŽoP34 1?e m 1 D^  u gX%  34 L FD 0 15CUqž^E ? } D^ + ?1
gKlx—{ { { {6Ukg‹lx™,R { {6UŽ 5 u Rlx~,R ,R6UŽ k2 0 1sŽoP34 1
' ' ' ' ' '
gKd% W34 Y FD 0 15CUŒ3k2W 55m:K6  l d D 1 c7„2 55mc  D 1P{6U
k2s-55ŽoP34 1 u rL% 34 l FD 0 15C

"
gK6 u g ƒ ,-   1X W D c5e m  34 1S;U/`;DFD  Y D cm
0 1 
e m PBie m  1/ D^ +  xŸ,! 6g Ui 5ID 11  x 
  
   


, ƒ ,! g6 /S;] e m   1Œ D^ + x g O 
 x™, g ƒ ,-U
5D 11  x™,# , g ƒ ,
*q'
,-

 


                                                                                                         
                   
                                            

                 
              
        
       
                                                       
                      
                  

                    
     
     
     
      
   
 

g … S{k,-sBD 1 s  X, ' x gk,o3  P ‰b,-‰Fg6‰ u ‰br ŽUŠ1o5ms D Y 16 <2  
  % #BiI 1 s D1UV X D^ +  o3  P5 1134 15 3k2 1mdL % FD

SR
 D r: 1 1 1k };D  D 1L} L
P} D X  % h^} % FD† D 1
gk,co3   U 5    *

K :m
0 s5E  o3  Y ‰b,-‰Fg6‰ u ‰br-‰FK ŽU 
 
5† „o3   ‰b,-‰Fg6‰ u ‰br-‰FK s  X36 F  15W Dl;( %2[BD so 
D P 
1^ + mo 5   
K L9K6U41 } D } D 1/ 1/;
4 % 1 ?;
Dd % FDdmCŒBDŽo D l E;= 1/K?O"gk,-

g‹6 u 6 
‹ Ivq  C#‚p x #‚p

ƒ #‚p 5 Š#‚p x C#‚p  


, ƒ C#‚p  BD 1
* ' *
D m
0 1[4
2-e i 1Žo^  C#‚p  *
K ƒ C#‚p  *

'
u ƒ C#‚p
*
,
*
C#‚p x
 Š#‚p u  ƒ  ' * ' * '
Š#‚p  /  D^ + /
 /p   D l % 5mL 1_Π4
2-e 
*n' *
 ' * *
i#‚p UC D 1I D L % 5mY 1  i#‚p  
ƒ i#‚p  'Žp  $ PBDŽo? D

 
'
* ' * *
 % 5ms 1G C#‚p }g+p & Ui D  % 5m 1G Š#‚p ","pŠU 5[ D
* *
 % 5md#– 5D 11 l
2 1 nom
0 1d4
2-e 4 u ‹6
*

g{6 R6‰ u R
*
! '


u Œš-1 # #‚p
**

x #‚p # #‚p 

* *S' *
'

x #‚p #‚p & 'Žp


* ' ' *
' U
;= 9 1 1 D /0 o 1= 'V;DFDXp & 'Žp 'x—R}D^ =P % 4o6 1U
' '
ba61 16 94 1D^ pdx~Rp:x ƒ '4(BD 1 }e +  1( <;=X  lo6 
p & 'Žp 'x—R  'x—R6U";DFD};=Žn n /43 0 o Šp & 'Žp 'x™R
' '

D^ =} % 4o6 „
q 5X
2
 ' & ƒ u ' `R}
q 5X
2
qR   ' '
' u 

u R6}#ˆ r !,-/ } D^ + ? D L 1Žo^ + I 1pyR6}”<e + m  ƒ  $


' *
& ƒ
5Žo^ m
ž^ 1„ :#‚p
*
ƒ
, pn#‚p  &ƒ * '
C
px™R6U6;DFD %2-
# p & ƒ ƒ p & x R6UV 5 D 11 sp x_p & ƒ UG;DFDW D s 1o

*
S;1


1. 5 is 20% of what number?

2. What is the value of 1 − 2 + 3 − 4 − · · · + 99 − 100? (alternating signs)

3. A frog is at the bottom of a well 20 feet deep. It climbs up 3 feet every


day, but slides back 2 feet each night. If it started climbing on January
1, on what date does it get to the top of the well?

4. How many integers between 2004 and 4002 are perfect squares?

5. Let the operation ∗ be defined by a ∗ b = a2 + 3b . What is the value of


(2 ∗ 0) ∗ (0 ∗ 1)?

6. The radius of each small circle is 1/6 of the radius of the large sur-
rounding circle. The radius of the middle-sized circle is twice that of
either small circle. What fraction of the area of the large circle lies
outside the three interior circles?
..........
........ ..........
.................. .............. .....
. .
.... ........... ....
... .. ... ..
.... ............ ...
......... ........
.........

7. What is the number of square units enclosed by the polygon ABCDE?


..........................................................................................................................
............................................................A ...............................................E ........
............................................................................................................
. . . .. . . .. . .
.......................................................................................................................
.....................................................................................................
. . . ........ .. .. .. .. .... .. .. ..
.........................B .............................................................................
................................................................................................................
....................................................................................................
. . . . . . . . . .. . . . .
..............................................................................................................................
.....................................................................................................
....................................................................................................
. . .. . . . . . . . . . . .
.......................................................................................................................
............C
.......................................D ................................................

1
22004 + 22001
8. What is the value of ?
22003 − 22000

9. What is the area of the region of the plane satisfying 1 ≤ |x| + |y| ≤ 2?

10. The square below can be filled in so that each row and each column
contains each of the numbers 1, 2, 3, and 4 exactly once. What does x
equal?
................................................................
... . . . 1 .
..................................................................
... . 2 . . .
............................................................
... . . . .
.....................................x.........................
... .. .. .. .
... 1 ... ... ... 4 ...
.............................................

11. Johnny was ill and had to take a test a day late. His 96 raised the class
average from 71 to 72. How many students, including Johnny, took the
test?

12. The pentagon ABCDE has a right angle at A, AB = AE, and ED =


DC = CB = 1. If BE = 2 and is parallel to CD, what is the area of
the pentagon?
A .
.. .... .........
. .....
...... ....
.
B ... .. E
... ...
........................
C D

√ √
4
13. What value of x satisfies the equation x + 10 + x + 10 = 12?

14. A quadratic polynomial f satisfies f (x) ≥ 0 for all x, f (1) = 0, and


f (3) = 3. What is f (5)?

2
15. In each hand, a girl holds a different end of a 26-inch long piece of light
string that has a heavy bead on it. Initially her hands are together.
How many inches apart must she pull her hands, keeping them at the
same height, so that the bead moves upward by 8 inches?

16. Set t1 = 2 and tn+1 = (tn − 1)/(tn + 1). What is t203 ?

a(1−b) 2

17. If a 6= b and b(1−a)
= 1, what is the value of (a + b)/(ab)?

18. In the following figure, AD = AE, and F is the intersection of the


extension of ED with the bisector of the angle C. If angle B = 40
degrees, how many degrees are there in angle CF E?

....A
........F
...................... .. .... .....
........ .................... D.... .
........ ................... ....
........ ... . .................
........ ... ...E
.
... ...
.. ..G............. ...
.. .
. ..........
. .... .......... .....
. ........ ..
.. .... ........ ...
..
B ................................................................................. C

19. Let N = 999,999,999,999,999,999. How many 9’s are in the decimal


expansion of N 2 .

20. If a, b, and c denote the solutions of the equation x3 − 2x2 − 5x + 8 = 0,


1 1
what is the value of ab + ac + bc1 ?

21. How many integers n satisfy n4 + 6n < 6n3 + n2 ?

22. Let P be the point (8, 0). Suppose P Q is tangent to the circle x2 +y 2 =
4 at the point Q. What is the length P Q?

23. Find all ordered triples of positive integers (a, b, c) for which a3 − b3 −
c3 = 3abc and a2 = 2(b + c).

3
24. What is the largest prime p such that there is a prime q for which p + q
and p + 7q are both squares?

25. A square is partitioned into 30 non-overlapping triangles, so that each


side of the square is a side of one of the triangles. We also require that
the intersection of any two triangles is either empty, a common vertex,
or a common edge. How many points in the interior of the square serve
as vertices of one or more triangles?

26. How many 4-digit numbers (from 1000 to 9999) have at least one digit
occurring more than once?

27. What is the sum of all positive integers x for which there exists a
positive integer y with x2 − y 2 = 1001?

28. A triangle has sides s1 , s2 , and s3 of lengths 10, 9, and x, respectively.


The angle between s1 and s3 is 60 degrees. What is the difference of
the two possible values for x?

29. What is the sum of all 2-digit positive integers which exceed the product
of their digits by 12?

30. In the diagram below a circular arc centered at one vertex of a square
of side length 4 passes through two other vertices. A small circle is
tangent to the large circle and to two sides of the square. What is the
radius of the small circle?
....................................................................................................................
.......... .....
...
... ........ .... ...
.
....... .. .
... ....... ..
... ....... ...........
... ........... ..
... .... .
... ... ..
... ... ..
... ... ..
... ... ..
... .....
... ....
... ..
...............................................................................

4
31. What is the sum of all positive integers n for which 28 + 211 + 2n is a
perfect square?

32. What is the area of the shaded region in the diagram below?

............................3.........................................2..........................
... ........
.. ........ ..... .
.
... ...
2.... ....
. .... .
. ..
....... .. .. .................... ...3
.
... ........ ... .. .
.. .......... ..... .................
... ... ........ ... ..
... .. ......... ..
3.... .... . . ..
.. ...........
...... ... .. ........ ...2
.....................................................................
2 3

33. Suppose the function f satisfies f (x) + f (x − 1) = x2 and f (19) = 94.


What is f (94)?

34. What
p is√the number of degrees in the acute angle θ satisfying cos(θ) =
1
2
2 + 2?

35. A number y is selected at random in the interval [0, 10] (with uniform
probability distribution). A right triangle with hypotenuse of length
10 is formed with one end of the hypotenuse at the point (0, y) and the
other on the nonnegative x-axis. The third vertex is at (0,0). What is
the probability that the area of the triangle is greater than 15?

36. What is the sum of the reciprocals of all positive integers none of whose
prime divisors is greater than 3? That is, what is the value of

1 + 21 + 13 + 14 + 16 + 18 + 19 + · · · .

37. How many ordered pairs (x, y) of integers satisfy x1 + 1


y
= 12 ? (Note
that both positive and negative integers are allowed.)

5
38. What is the smallest value of n for which the product

(22 − 1)(32 − 1)(42 − 1) · · · (n2 − 1)

is a perfect square?

39. Note that 6492 − 13 × 1802 = 1. Find an integer a satisfying 649 < a <
108 such that a2 − 13b2 = 1 for some positive integer b < a.

40. Suppose triangle ABC is equilateral, and AF = BD = CE = 31 AB.


What is the ratio of the area of triangle ABC to that of the shaded
triangle?
C
............
.. .
. .... ... ......
.
E... ......... ....
.
..... ... ........... ......
... .. .............D
.... .
..... ....................... .................
. .
.........................................................................
A F B

6
Solutions to 2004 exam
Numbers in brackets are (a) the number of people out of the top 30 who
answered the question correctly, and (b) the number of people out of the
other 139 who answered the question correctly.

1. 25. [30, 137] 5 = 51 x implies x = 5 · 5.

2. −50. [29, 123] There are 50 pairs, each summing to −1.

3. Jan 18. [26, 72] It will get to height 3 on Jan 1, height 4 on Jan 2, . . .,
and height 20 on Jan 18.

4. 19. [26, 85] 442 = 1936, 452 = 2025, 632 = 3969, and 642 = 4096. Thus
it is the integers from 45 to 63, inclusive.

5. 52. [26, 101] It is (22 + 30 ) ∗ (02 + 31 ) = 5 ∗ 3 = 52 + 33 .

6. 5/6. [28, 85] Let the radius of the large circle be 1. Then the sum of
1 1
the areas of the three circles is π( 36 + 36 + 19 ) = π/6, while that of the
large circle is π.

7. 50. [28, 97] The region can be divided into two trapezoids, divided by
the horizontal line through B. The area of the top one is 4(4+6)/2 = 20
and the area of the bottom one is 6(4 + 6)/2 = 30.

8. 18/7. [26, 48] Divide top and bottom by 22000 , getting (16 + 2)/(8 − 1).

9. 6. [25, 16] It is the area inside a diamond with vertices at (±2, 0) and
(0, ±2) minus that in the diamond 1/4 the size (1/2 in each direction).
Thus it is 43 · 4 · 12 · 2 · 2. Here we have considered the four quadrants.

7
10. 4. [30, 129] In the diagram below, the bottom row is forced, then the
second column, then the top row, and then the third column.
......................................................................
... 2 ... 4 ... 3 ..... 1 ....
..............................................................
... ... ... .. ..
....................2.............1....................
... ... .... .... ...
.....................1...............x .. .
..... ..... ... ..............................
...... 1 .... 3 . 2 .... 4 ....
.. ..........................................

11. 25. [26, 85] If x is the desired number of students, then 71(x−1)+96 =
72x, and so x = 96 − 71.

12. 1 + 43 3. [21, 33] The right triangle ABE has area 12 · 2. The vertical

displacement from BE to CD is 12 3. Thus the area of the bottom is

(2 − 21 ) 12 3.

13. 71. [26, 80] Let y = 4 x + 10. Then y 2 + y − 12 = 0, so y = 3, since y
cannot be negative. Thus x + 10 = 34 .

14. 12. [18, 9] f (x) = a(x − 1)2 and 4a = 3. Thus f (5) = 34 42 = 12.

15. 24. [24, 64] The bead was initially 13 inches below the level of her
hands. When it has moved up 8 inches, it will be 5 inches below the
level of her hands, and will √
be 13 inches from each hand. Thus each
hand will have moved 12(= 132 − 52 ) inches out.

16. −1/2. [24, 28] t2 = 1/3. t3 = −1/2. t4 = −3. t5 = 2 = t1 . Thus the


numbers repeat with period 4, and t203 = t3 .

17. 2. [20, 29] Either a(1 − b) = b(1 − a), in which case a = b, which has
been excluded, or a(1 − b) = −b(1 − a). This gives a + b = 2ab, so
(a + b)/(ab) = 2.

18. 20. [20, 52] Let the desired angle equal x, the base angles of the isosceles
triangle ADE equal y, and the two halves of angle C equal z. Compar-
ing triangles ADE and ABC yields 2y = 2z + 40. Comparing triangles
DF G and BCG yields x + y = 40 + z. Thus x = 40 − 40/2.

8
19. 17. [17, 16] N 2 = (1018 − 1)2 = 1036 − 2 · 1018 + 1. Its decimal expansion
has 9’s in positions 1019 through 1035 .

20. −1/4. [6, 0] The desired quantity equals (a + b + c)/(abc). Since

(x − a)(x − b)(x − c) = x3 − 2x2 − 5x + 8,

we obtain a + b + c = 2 and abc = −8.

21. 4. [26, 35] We must have n(n2 − 1)(n − 6) < 0. Thus the factors n + 1,
n, n − 1, and n − 6 must be nonzero, and an odd number of them are
negative. Hence n = 2, 3, 4, or 5.
√ √
22. 60 or 2 15. [18, 19] If O is the origin, then P OQ is a right triangle
with
√ hypotenuse 8 and one leg 2. The other leg is P Q which equals
60.

23. (2,1,1). [5, 11] The first equation says that a is the largest of the three,
and the second equation that a is even. Thus 2a > b + c = a2 /2, and
hence 4 > a. Thus a = 2, b = 1, and c = 1 is the only possibility.

24. 2. [12, 14] Let p + q = a2 and p + 7q = b2 . Then 6q = (b − a)(b + a).


Thus one of b ± a must be even, and hence both are even (since their
difference is even). Since q is prime, we must have q = 2, then b = 4,
a = 2, and p = 2.

25. 14. [8, 3] If there are v interior vertices, then the total number of
degrees in all the angles (including the interior vertices and the four
vertices of the square) is, on the one hand, 30 · 180 and, on the other
hand, 360v + 4 · 90. Thus 30 = 2v + 2.

26. 4464. [9, 4] There are 9000 4-digit numbers. To enumerate those
with all digits distinct, we have 9 choices for the first digit (since 0
is excluded), then 9 for the second (i.e. all digits except the first), 8
for the third, and 7 for the fourth. Our answer is 9000 − 9 · 9 · 8 · 7.

9
27. 672. [8, 0] Since x2 − y 2 = (x + y)(x − y), we need the factorizations of
1001 = 7 · 11 · 13. The pair (x + y, x − y) can equal any of the following:
(1001,1), (143,7), (91,11), or (77,13). Thus the sum of the x-values is
1
2
(1002 + 150 + 102 + 90) = 672.
√ √
28. 2 6 or 24. [7, 5] The Law of Cosines yields 81 √ = 100 + x2 − 2 ·
2 1
10x cos(60). Thus x − 10x + 19 = 0, so x = 2 (10 ± 100 − 76).

29. 67. [24, 48] (10a + b) − ab = 12 implies (10 − b)(a − 1) = 2 and so the
numbers are 39 and 28.

30. 12 − 8 2. [10, 1] Let r denote the desired radius. The little 45-45-90
triangle in the top right corner with hypotenuse going from the center
of the small circle to the top right vertex
√ of the square
√ and one leg along
the top edge of the square implies 4 2−4−r = r √ 2. (This is two
√ ways
of viewing the length of√that hypotenuse.) Thus r( 2 + 1) = 4( 2 − 1)
and so r = 4(2 + 1 − 2 2)/(2 − 1).

31. 12. [10, 0] 28 +211 +2n = k 2 implies 2n = k 2 −(1+8)28 = k 2 −(3·24 )2 =


(k + 3 · 24 )(k − 3 · 24 ). Thus there exist integers a > b > 0 such that
a + b = n, 2a = k + 3 · 24 , and 2b = k − 3 · 24 . Hence 2b (2a−b − 1) = 3 · 25 ,
from which we deduce b = 5 and a = 7. Alternatively, 1 + 2u + 2v
with 0 < u < v is a perfect square if and only if v = 2u − 2. Then
28 + 211 + 2n must equal 22i (1 + 2u + 22u−2 ). Then 2i + u = 11, 2i = 8,
so 2i + 2u − 2 = 12.

32. 290/17 or 17 1/17. [6, 1] Let h denote the width of a strip, y the length
of the leg of a white triangle emanating from a vertex of the square,
and x the shorter leg of a white triangle. Then similar triangles yield
y/5 = x/3, y/3 = h/2, and (y − x)/3 = 2/(x + y +√h). We obtain
y = 35 x, then h = 10
9
x, then 2x/9 = 2/( 34
9
x) so x = 9/ 34. Hence the
shaded area is 10 + 10 − h2 = 20 − 100/34 = 290/17.

33. 4561. [4, 0] f (94) = 942 − f (93) = 942 − 932 + f (92). Continuing this

10
way yields

f (94) = 942 − 932 + · · · + 222 − 212 + 202 − 94


= (94 + 93)(94 − 93) + · · · + (22 + 21)(22 − 21) + 400 − 94
= 94 + 93 + · · · + 22 + 21 + 306
= 21 (95 · 94 − 21 · 20) + 306 = 4561

√ √
34. 22.5. [8, 2] We have cos(2θ) = 2 cos2 (θ) − 1 = 2( 14 (2 + 2)) − 1 = 12 2.
We deduce 2θ equals 45 degrees.

1
√ p
35. 5
10. [6, 0] We desire the probability that y 100 − y 2 > 30. Equality
is obtained if y 2 (100 − y 2 ) = 900 and so y 4√− 100y√2 + 900 = 0. √
Hence
2
y√ = 10 or 90.
1
√ The desired probability is ( 90 − 10)/10 = (3 10 −
10)/10 = 5 10.

1 1 1 1 1
P P P
36. 3. [12, 10] It is i,j 2i 3j = i 2i j 3j = 1−1/2 1−1/3
= 3.

37. 5. [7, 14] For x, y 6= 0, the equation reduces to 2(x + y) = xy or


(x − 2)(y − 2) = 4. Thus (x − 2, y − 2) equals (2, 2), ±(4, 1), or ±(1, 4).
Note that it cannot equal (−2, −2) since that corresponds to (x, y) =
(0, 0).

38. 8. [12, 4] Writing i2 − 1 = (i − 1)(i +1) and removing the square (i +1)2
that occurs in (i2 − 1)((i + 2)2 − 1), we are asking for the smallest n > 1
for which 2n(n + 1) is a perfect square. Here you can just try until you
find that it works at n = 8. You can make the numbers a little smaller
by letting n = 2a or 2a + 1 and removing the 22 .
√ √
39. 842401. [0, 0] We have (649 + 180 13)(649 − 180 13)√= 1. Expand
√ of both factors, obtaining (842401 + 233640 13)(842401 −
the squares
233640 13) = 1. Hence a = 842401 and b = 233640 works.

40. 7 or 7:1. [1, 0] Let α denote the area of the small triangles, β the area
of the quadrilaterals, and γ the area of the desired small equilateral
triangle, as indicated below. Since the area of triangle CF B is twice

11
the area of CAF , we obtain 2(2α + β) = 2β + α + γ, and hence γ = 3α.
The Law of Cosines says CF 2 = AF 2 +(3AF )2 −2·AF ·(3AF ) cos(60) =
7AF 2 . Since triangle ACF is similar to the little triangle in the lower
left, their areas are proportional to CF 2 : AF 2 = 7. Thus 2α + β = 7α
and so β = 5α. The desired ratio is (γ + 3α + 3β)/γ = (3α + 3α +
15α)/3α = 7.
C......
........ ......
. .. ....
..α
E.. ......... β ......
...
..
.. ......... ...
..... ... ........... ......
... β .. γ ......................D
..... ....................... ...α ... ....
................α.. .. β .................
.... ...............................................................
A F B

12
1
1. 2
+ 13 + 1
5
=

2. The sum of three numbers is 17. The first is 2 times the second. The
third is 5 more than the second. What is the value of the largest of the
three numbers?

3. A chemist has 100 cc of 20% acid, the rest being water. She adds pure
acid to make the solution 33 13 % acid. How many ccs of water must she
then add to return it to 20% acid?

4. An isosceles triangle has two sides of length 10 and one of length 12.
What is its area?

5. What is the y-intercept of the straight line passing through (10,2) and
(8,5)?

6. What is the length of the altitude drawn to the hypotenuse of a right


triangle having legs equal to 5 and 12?

7. Which is the largest of the following: 148 , 242 , 336 , 430 , 524 , 618 , 712 , 86 ,
90 ? (Write the exponential expression, not the large integer which it
equals.)

8. Two bicyclists 2 miles apart start pedaling toward each other with
speeds 9 and 10 miles per hour, respectively. A fly flies at 12 miles per
hour from one bicycle to the other, turns around instantly, flies back,
etc., until the bicyclists meet. How many miles did the fly fly?

9. A sequence is defined by an = an−1 + an−2 + an−3 for n ≥ 4. Suppose


a4 = 20, a5 = 36, and a7 = 121. What is the value of a1 ?

10. What is the number of sides of a regular polygon for which the number
of diagonals is 44 or 45?

11. There are five cards, two red and three black. You draw two cards,
without replacement. What is the probability that both have the same
color?

1
12. The length of each leg of an isosceles triangle is x + 1 and the length
of the base is 3x − 2. Determine all possible values of x. (The triangle
should be nondegenerate; i.e. not just a straight line. Note also that x
need not be an integer; your answer should be an inequality.)

13. One laser blast will break asteroids larger than 20 kg into three pieces,
each one third of the mass of the original. Asteroids smaller than 20
kg are shattered into dust by the laser. How many laser blasts would
be required to reduce a 2000 kg asteroid to dust?

14. What is the radius of the smallest circle that contains both of the circles
x2 + y 2 = 1 and (x − 1)2 + (y − 2)2 = 4?

15. What is the number of different 7-digit numbers that can be made by
rearranging the digits of 3053354? (Note that this includes the given
number, and that the first digit of a number is never 0.)

16. In what positive base b does the equation 4 · 12 = 103 for multiplication
of base-b numbers hold?

17. An octahedron is formed by connecting the centers of the faces of a


cube. What is the ratio of the volume of the cube to that of the
contained octahedron?

18. What is the largest number of pieces into which a circular pizza can be
cut with 9 straight cuts?

19. If the odd numbers are grouped in the following way:

{1}; {3, 5}; {7, 9, 11}; {13, 15, 17, 19}; . . . ,

what is the sum of the numbers in the tenth group?

20. If f (3x) = 3/(1 + x) for all x 6= −1, then 3f (x) =

2
21. Two students attempted to solve a quadratic equation x2 + bx + c = 0.
Although both students did the work correctly, one miscopied the mid-
dle term and obtained the solution set {2, 3}, while the other miscopied
the constant term and obtained the solution set {2, 5}. What is the
correct solution set?

22. Four circles of radius 1 are arranged so that each is tangent to two
others, and their centers lie on vertices of a square of side 2. A small
circle at the center of the square is tangent to the four circles. What
is the radius of the small circle?

23. Find all values of x which satisfy x4 + 6ix3 − 6x2 − 6ix + 1 = (x + i)4 ,
where i2 = −1.

24. If the hypotenuse of a right triangle is 8 units, and the area is 8 square
units, what is the√tangent of the smallest angle of the triangle, written
in the form a + b c, with a, b, and c integers?

25. How many 0’s occur at the end of the decimal expansion of 100100 −100!?
(This number is 100 to the 100th power minus 100 factorial.)

26. What is the number of positive integers n for which there is a triangle
with three positive acute angles and sidelengths 10, 24, and n?

27. Compute S = 15 + 251 2


+ 125 3
+ 625 5
+ 3125 + · · · , where each numerator is
the sum of the two preceding numerators, and each denominator is 5
times the preceding one. Your answer should be a fraction.

28. Let A = 1−4 + 2−4 + 3−4 + 4−4 + 5−4 + · · · denote the sum of the
reciprocals of the fourth powers of all positive integers, and B = 1−4 +
3−4 +5−4 +7−4 +· · · a similar sum for all odd positive integers. Express
A/B as a fraction.

29. Let ABCD be a rectangle with BC = 2AB, and let BCE be an equi-
lateral triangle crossing through the rectangle. If M is the midpoint of
EC, how many degrees are in angle CM D?

3
30. Let C denote the cylinder x2 + y 2 = 4, 0 ≤ z ≤ 6, of radius 2 and
height 6. What is the length of the shortest path on the cylinder from
the point (2, 0, 6) to the point (2, 0, 0) which passes through the point
(2, 0, 2) and passes twice around the cylinder?

31. In triangle ABC, AC = 6 and BC = 5. Point D on AB divides it into


segments of length AD = 1 and DB = 3. What is the length CD?

32. List all pairs (m, n) of positive integers for which n! + 1 = (m! − 1)2 .

33. An acute angle is formed by two lines of slope 1 and 7. What is the
slope of the line which bisects this angle?

34. How many ordered triples (x, y, z) of positive integers satisfy xyz =
4000?

35. An equilateral triangle is filled to the max with n rows of congruent


circles. (The case n = 4 is pictured below.) What is the limit as n
approaches ∞ of the ratio (area in circles)/(area of triangle)?

.....
..................
.... .....
....................................
..... .... .....
.........................................................
.. ....
..... . .. .
.. .......................................................................
.... .. .
. .
. .....
..........................................................................................................

36. List all 3-digit numbers abc for which the 6-digit number 579abc is
divisible by 5, 7, and 9.

4
37. The figure below shows a quarter-circle of radius 1, with A chosen so
that angle AOD is 30 degrees. What must be the distance OX so that
the region bounded by AX, XB, and the arc AB occupies half the area
of the quarter circle?
B.............................
.. ... .......
.. ... .....
.. .... ....
....
.. ... A
.. ... .......
.. ... ...... ....
.. .. ... ..
...............................................................
O X D

38. An equilateral triangle is inscribed in a circle. Let D and E be mid-


points of two of its sides, and let F be the point where the line from D
through E meets the circle. What is the ratio DE/EF ?

39. How many pairs (n, m) satisfying 10 ≤ m < n and m + n ≤ 99 have the
property that m + n and n − m have the same digits in reverse order?
(This allows cases such as (33, 27) where the sum and difference are 60
and 06.)

40. Parallel chords in a circle have length 12 and 16, and the distance
between them is 7. Another chord is midway between them. What is
its length?

5
SOLUTIONS TO 2005 CONTEST
The numbers in brackets are the number of people who answered the
problem correctly, first out of the 31 people who scored at least 19, and then
out of the other 222 people.

1. 31/30. [30,207] It is (15 + 10 + 6)/30.

2. 8. [29,173] Let x denote the second number. Then 2x+x+(x+5) = 17,


hence 4x = 12 and x = 3. The other numbers are 6 and 8.

3. 80. [28,104] Let x be the amount of acid added at first. Then (20 +
x)/(100 + x) = 1/3, which implies x = 20. Now we have 40 acid out
of 120 altogether. To make it 20% acid, we must bring the total liquid
up to 200, by adding 80 cc.

4. 48. [31,159] The altitude h satisfies h2 + 62 = 102 , and so h = 8. Thus


the area is (12 · 8)/2.

5. 17. [29,173] Decreasing x from 10 to 8 increased y by 3. Thus decreasing


x 8 more will increase y four times as much; i.e. by 12, to 17. Or,
y − 2 = − 32 (x − 10) so when x = 0, then y = 2 + 15.

6. 60/13. [30,57] The hypotenuse equals 13. Calculating the area of the
triangle in two ways yields 5 · 12 = 13h.

7. 430 . [27,57] Raise each to the 1/6 power. Then we compare 36 = 729,
45 = 1024, and 54 = 625. The others are smaller.

8. 24/19. [25,48] The bicylists are approaching each other at 19 mph, and
so meet in 2/19 hours. Thus the fly travels 12 · 2/19 miles.

9. 4. [31,118] Find a6 = 121 − 36 − 20 = 65, then a3 = 65 − 36 − 20 = 9,


then a2 = 36 − 20 − 9 = 7, and finally a1 = 20 − 9 − 7 = 4.
¡n¢
10. 11. [30,73]¡The¢ number of diagonals of a regular n-gon is 2
− n. Then
11
note that 2 − 11 = 44.

6
11. 2/5 or 0.4. [29,93] If the event is the two cards in order, then there are
5 · 4 ways of drawing the cards, of which 2 · 1 have both red, and 3 · 2
have both black. The answer is 8/20.

2
12. 3
< x < 4. [27,60] Must have 2(x + 1) > 3x − 2 hence x < 4, and the
base must have positive length, which implies x > 2/3.

13. 364. [20,48] If M = 2000 denotes the initial mass, then 1 blast yields
three of mass M/3, 3 more blasts yield 9 of mass M/9, 9 more blasts
yield 27 of mass M/27, 27 more blasts yield 81 of mass M/81, 81 more
blasts yield 243 of mass M/243, which are now less than 20 = M/100
kg, but another 243 blasts are required to turn them to dust. Now add
the total number of blasts.

14. (3 + 5)/2. [12,16] Extend the segment connecting the centers until it
meets the two circles again. This will be a diameter of the containing
circle. Its length equals
√ the distance between the centers plus the sum
of the radii, which is 5 + 1 + 2.

15. 360. [25,13] The ¡0 ¢can be in any of 6 positions. Then the three 3’s
can¡ be
¢ in any of 63 = 20 positions. Then the two 5’s can be in any
of 32 = 3 positions. Finally the position of the 4 is forced by the
preceding choices. Hence the answer is 6 · 20 · 3.

16. 5. [31,54] We must have 4(b + 2) = b2 + 3, hence b2 − 4b − 5 = 0, so,


since b is positive, it equals 5.

17. 6 or 6 : 1. [16,14] Let the side length of the cube equal 1 unit. The
octahedron is the union of√two pyramids of height 1/2 on a base which
is a square of sidelength 2/2 (since in a plane one side of the square
could be considered to be connecting the points (1/2, 0) and (0, 1/2)).
The volume of each pyramid is hA/3 = 21 12 13 .

18. 46. [27,35] The maximum number of pieces that can be added by a
cut is 1 greater than the number of lines that the new cut intersects.
Hence the answer is 1 + 1 + 2 + · · · + 9 = 1 + (9 · 10)/2.

7
19. 1000. [31,154] If you compute the first four sums 1, 8, 27, 64, you can
probably guess that the sum of the ith group is i3 . One way to prove
it would be to note that the ith group has i numbers and their average
is i2 . To see this when i is odd, note that there will be i(i − 1)/2
odd numbers preceding the group, and the middle entry will be the
((i + 1)/2)nd number in the group. It will thus equal −1 + 2(i(i −
1)/2 + (i + 1)/2) = i2 . A similar argument works when i is even.

20. 27/(3 + x). [30,61] Since f (3x) = 9/(3 + 3x), we have f (x) = 9/(3 + x).
Now multiply by 3.

21. {1, 6} or just 1,6. [30,88] Since the constant term is the product of the
roots, this must be 2 · 3 = 6, while −b equals the sum of the roots, and
so this must be 7. Thus the polynomial is x2 − 7x + 6.

22. 2 − 1. [30,96] Let r denote the desired radius. A 45-45-90 triangle is
formed with hypotenuse connecting the center of the little circle with
the center of one of the four given circles, and other vertex at a point
of tangency of that given circle with another. The hypotenuse is 1 + r
and the legs are 1. Thus (1 + r)2 = 2.

23. 0, 1, −1. [19,28] (x + i)4 = x4 + 4ix3 − 6x2 − 4ix + 1. Equating this to


the given expression yields 4x3 − 4x = 6x3 − 6x, hence 2x(x2 − 1) = 0.

24. 2− 3. [12,2] If x and y denote the legs, then xy = 16 and x2 +y 2 = 64.
Adding √and subtracting
√ twice the
√ first √equation to the √
second
√ yields
x+y = 96 = 4 6 and x−y = 32 = 4√ 2.√ Thus√ x = 2( 6+ 2) and
√ √ √
y = 2( 6− 2). The desired tangent is √6− √2 = √3−1 = ( 3−1)2 /2 =
√ 6+ 2 3+1
(4 − 2 3)/2.

25. 24. [18,6] As 100100 has many more 0’s at the end, we need here the
number of 0’s at the end of 100!. This will be the number of factors
of 5 in its decomposition into primes, since there will be plenty of 2’s
to turn these into 10’s. There are 20 multiples of 5 in 100! and four of
them have a second factor of 5 (i.e. the multiples of 25).

8
26. 4. [18,21] The square of the largest side must be strictly less than the
sum of the squares of the other two. Thus n must satisfy 476 < n2 <
676 and hence n = 22, 23, 24, or 25.

27. 5/19. [7,0] 5S = 1 + 15 + 25


2 3
+ 125 5
+ 625 + · · · . Subtract the series for
1 1 2
S from this, obtaining 4S = 1 + 25 + 125 + 625 + · · · = 1 + 15 S. Thus
19
5
S = 1.

28. 16/15. [9,0] B = A − (2−4 + 4−4 + 6−4 + · · · ) = A − A/16.

29. 75. [24,41] Both CM and CD equal half a side of the triangle. Hence
triangle CM D is isosceles. Since angle M CD equals 90 − 60, the other
angles of the triangle are (180 − 30)/2.
√ √
30. 4 π 2 + 1 + 2 4π 2 + 1. [14,2] Unroll the cylinder. The paths will
be straight lines on the rectangle. The
pfirst time around,
√ going from
height 6 to height 2, will have length (4π) 2 + 42 = 4 π 2 + 1, while
p √
the second time around will have length (4π)2 + 22 = 2 4π 2 + 1.

31. 11/2. [10,6] If x is the desired length and θ = ∠BDC, then we have
x2 + 1 + 2x cos θ = 36 and x2 + 9 − 6x cos θ = 25. Adding 3 times the
first equation to the second yields 4x2 + 12 = 133.

32. (3,4). [14,28] We obtain n! = m!(m! − 2), and so n(n − 1) · · · (m + 1) =


m! − 2. Clearly m ≥ 3 and hence the right hand side is not divisible
by 3 (since m! is), and so, since the product of three consecutive odd
integers is divisible by 3, we obtain n = m + 1 or m + 2. The equations
become m + 3 = m! or m2 + 3m + 4 = m!. It is easy to check that the
first equation has m = 3 as its only solution, while the second has no
solutions.

33. 2. [13,12] Using the formula for the tangent of the difference of two
angles, we obtain
x−1 7−x
= ,
1+x 1 + 7x
where x is the desired slope (or tangent). We obtain 7x2 − 6x − 1 =
−x2 + 6x + 7 and so 8x2 − 12x − 8 = 0 with solutions 2 and −1/2.

9
34. 210. [9,0] Since 4000 = 25 53 , we must have x = 2a 5d , y = 2b 5e , and
z = 2c 5f , and our answer will be AB, where A is the number of ordered
triples (a, b, c) of nonnegative integers such that a + b + c = 5, and B
is the number of ordered triples (d, e, f ) of nonnegative integers such
that d + e + f = 3. Note that a = 0 has 6 possibilities for (b, c),
namely 0 ≤ b ≤ 5, a = 1 has 5 possibilities for (b, c), etc., down to
a = 5 having one possible (b, c). Thus the number of possible (a, b, c)
is 6 + 5 + 4 + 3 + 2 + 1 = 21. Similarly the number of possible (d, e, f )
is 4 + 3 + 2 + 1 = 10.

35. π 3/6. [11,0] Let the sidelength of the triangle equal 1 unit, √ and
let the radius of the little circles equal r. Then 2(n − 1)r + 2r 3 = 1.
This is seen by consideration of the 30-60-90 triangle whose hypotenuse
connects a vertex of the triangle with√ the nearest center of a circle. Thus
2
the area of each circle is π/(4(n + 3 − 1) ), and there are n(n + 1)/2
n(n+1)
circles. The area covered by the circles is π8 (n+ √ . This approaches
3−1)2 √
π/8 as n → ∞. Since the area of the triangle is 3/4, the answer
follows.

36. 285, 600, 915 in any order. [13,6] Let n = 579abc. This n must be
divisible by 5 · 7 · 9 = 315. One can easily check that the 579000 is 30
more than a multiple of 315. Thus the valid abc will be 285, 600, and
915.

37. π/6. [4,0] Let x = OX be the desired length. We will find x so that
the area of the complement of the desired area is π/8. Triangle BOX
has area x/2. Since AE = 1/2, triangle AXD has area (1 − x)/4, and
triangle OAD has area 1/4. Thus the sliver between the line AD and
the arc AD equals π/12 − 1/4. Our equation becomes
1 π
4
(1 − x) + 12
− 14 + x
2
= π8 ,
from which it follows that x = π/6.
B.............................
.. ... .......
.. ... .....
.. .... ....
....
.. ... A
.. ... .............................
.. ........... .... .. ......
.. ............... .... .. ......
..................................................................
O X ED

10

38. (1 + 5)/2. [7,1] The triangles GDB and ADF are similar, since the
angles at B and F subtend the same arc AG. This explains the second
equality in the following string:
DE AD DF DF DE + EF
= = = = .
EF GD DB DE DE
1
If x is the desired ratio, then this shows x = 1 + x
and hence x is as
claimed.

...A
.. ..
..... ....................................................
. . . .
.. .. .. ..... ............ ........
.
..... ..... ... ....... ..
.. ... ..
...............D ... .................................F
G.... ................................E ... .
........ ..... ... ...
...... ... ... .
..... ... ... ..
..........................................................................
B ....... .
.
.... C
....... ... .
.
............ .............
............

39. 12. [3,1] We have n + m = 10x + y and n − m = 10y + x, hence


2n = 11(x + y) and 2m = 9(x − y). Thus n is a multiple of 11, and m
is a multiple of 9. The restrictions 10 ≤ m < n and m + n < 100 then
imply that the following are the possible pairs: (22,18), (33,18), (33,27),
(44,18), (44,27), (44,36), (55,18), (55,27), (55,36), (66,18), (66,27), and
(77,18).

40. 249. [8,1] Let r denote the radius of the circle, and d1 and d2 the
distances from the center of the circle to the chords of length 12 and
16, respectively. Each of the perpendiculars from the center to a chord
makes a right triangle, with other leg equal to half the length of the
chord. Thus 36 + d21 = r2 = 64 + d22 . Assume the chords are on
opposite sides of the center. (If you put them on the same side, you
will get a negative value for one of the d’s.) Since d1 +d2 = 7, we obtain
7(d1 − d2 ) = 64 − 36 = 28, hence d1 − d2 = 4. From this we obtain
d1 = 11/2 and d2 = 3/2, and then r2 = 265/4. The distance from our
midway chord to the center is (d1 − d2 )/2 = 2. If x denotes the length
of the desired chord, then ( x2 )2 + 22 = r2 , and so (x/2)2 = 249/4.

11
1. 1/( 31 − 14 ) =
2. Dick is 6 years older than Jane. Six years ago he was twice as old as she was. How
old is Jane now?
3. A bicyclist riding against the wind averages 10 miles per hour in traveling from A to
B, but with the wind averages 15 miles per hour in returning from B to A. How many
miles per hour is his average speed for the trip?
4. What is the largest possible value for the sum of two fractions such that each of the
four 1-digit prime numbers occurs as one of the numerators or denominators?
5. How many integers x in {1, 2, 3, . . . , 99, 100} satisfy that x2 + x3 is the square of an
integer?
6. What is the number of real numbers x such that 25|x| = x2 + 144?
7. How many pairs (x, y) of positive integers satisfy 2x + 7y = 1000?
8. A ladder is leaning against a house with its bottom 15 feet from the house. When
its bottom is pulled 9 feet farther away from the house, the upper end slides 13 feet
down. How many feet long is the ladder?
9. What is the sum of the three smallest prime numbers each of which is two more than
a positive perfect cube?
10. Amy, Bob, and Chris each took a 6-question true-false exam. Their answers to the
six questions in order were Amy:FFTTTT, Bob:TFFTTT, and Chris:TTFFTT. Amy
and Bob each got 5 right. What is the most that Chris could have gotten right?

11. The two shortest sides of a right triangle have lengths 2 and 5. Let α be the smallest
angle of the triangle. What is cos α?
12. From a point P on the circumference of a circle, perpendiculars P A and P B are
dropped to points A and B on two mutually perpendicular diameters. If AB = 8,
then what is the diameter of the circle?
13. How many 9’s are in the decimal expansion of 999999899992 ? (This is the square of
an 11-digit number.)
14. Let A be the point (7, 4) and D be (−5, −3). What is the length of the shortest path
ABCD, where B is a point (x, 2) and C is a point (x, 0)? This path consists of three
connected segments, with the middle one vertical.
p √ p √
15. Simplify 3 + 2 2 − 3 − 2 2.
16. A square has its base on the x-axis, and one vertex on each branch of the curve
y = 1/x2 . What is its area?

17. Which integer is closest to 12 ( 3 829 + log10 829)?

1
18. In a 9-12-15 right triangle, a segment is drawn parallel to the hypotenuse one third
of the way from the hypotenuse to the opposite vertex. Another segment is drawn
parallel to the first segment one third of the way from it to the opposite vertex. Each
segment is bounded by sides of the triangle on both ends. What is the area of the
trapezoid inside the triangle between these two segments?
19. A rhombus has sides of length 10, and its diagonals differ by 4. What is its area?
20. What is the smallest positive integer k√for which
√ there exist integers a > 1 and b > √ 1
3
for which
√ the correct simplification of k is a b, and the correct simplification of k
is b 3 a?
21. The diagonal BE in pentagon ABCDE is the base of an isosceles triangle ABE and is
also the base of an isosceles trapezoid BCDE. If 6 C = 36 CBE, express the number
of degrees in angle A in terms of angle B (= 6 ABC). (Your answer should be an
expression involving the letter B. Don’t write “A =” on your answer sheet.)
22. In a group of five friends, the sums of the ages of each group of four of them are 124,
128, 130, 136, and 142. What is the age of the youngest?
23. In the figure below, if the area of the letter L part equals the area of the triangle, and
the length of the base and of the height is 1, what is the length x of the ends of the
L?
.........x
.................
... . .. .
... ..... .........
... .. .....
... .. .....
. .....
... ..
. .....
.....
... .
. .....
... ..
. .....
... .
. .....
... ... .....
1 ...
...
...
...
..
..
.....
.....
.....
.....
... .
. .....
... .
. .....
... .
. .....
.
... ............................................................................
.
.
... ..
...
..
..
x
...
............................................................................................
1

24. Let a, b, and c denote the three roots of x3 −17x−19. What is the value of a3 +b3 +c3 ?
25. Determine all values of k such that the solution set of |x − k| < 2 is a subset of the
solution set of 2x−1
x+2 < 1. (You may use interval notation or inequalities to express
your answer.)
26. A rhombus of side length s has the property that there is a point on its longer diagonal
such that the distances from that point to the vertices are 1, 1, 1, and s. What is the
value of s?
27. Define a ∗ b = ab + b. Find the set of all real numbers a such that the equation
x ∗ (a ∗ x) = −1/4 has two distinct real solutions in x. You may use interval notation
or inequalities.
28. Let Q denote the point (4, 2). There are two points P on the circle x2 + y 2 = 2 such
that P Q is tangent to the circle. One of them, P1 , has integer components and the
other, P2 , has fractional (non-integer) components. What are the components of P2 ?

2
29. The angle trisectors of a regular pentagon intersect other vertices of the pentagon; i.e.
they are diagonals of the pentagon. (See the left figure in the diagram below.) What
is the smallest n > 5 such that the angle trisectors of a regular n-gon intersect other
vertices of the n-gon? (The right side of the diagram below illustrates that n = 6 does
not work.)
.....
........ ......
...... .. ...........
. ......... ..... ...... .
.... ...
......
...
...... ......
. .......... ....
. ......
...... .
.................................................
.. .
.. .... .
. ...... ...
. ... ...
........ ..
. ..
..... ... .. ...
...
... ..
..
.
.. ........... .... ...
. ... ....
... .. .. ..
.. ... . . .. ......
. . . .
... .
...
.
......
.
...
.. .
. ........
... ...... ... .
. ..... .....
.
... ... ..... ... ..
... .
.
.....
.
.. .
... ... .
....... ... ... ..
... ..... ...
... .
.
...
.
.....
.
. .
. ... ..... ...
... ...... ... ... .. ..... ...
... .... ...... ... ... .... ...
. ..
... ... .......... ... ... .. .......... ...
.
... .. ...... . .. ... ... ..... ...
............. ... ... .. ...... ...
.................................................................. ....................................................

30. Let han i denote an arithmetic sequence beginning a1 = 6, a2 = 8, a3 = 10. Define


a second sequence hbn i by b1 = 3 and bn = bn−1 + an . Write an explicit expression
for bn in terms of n. (Just write the expression on your answer sheet. Don’t write
“bn =”.)
31. Let d(n) denote the sum of the digits of n. It is well-known that n − d(n) must be a
multiple of 9. What is the smallest positive integer k such that k is a multiple of 9
and neither k nor k + 9 can be achieved as n − d(n) for some n?
32. Let S denote the set of all (positive) divisors of 605 . The product of all the numbers
in S equals 60e for some integer e. What is the value of e?
33. Let S = {1, 2, 3, 4, 5, 6, 7, 8, 9, 10, 11, 12, 13, 14}. A special subset of S is a subset B
which satisfies the following three properties:
a. B has exactly 8 elements;
b. If x ∈ S is even, then x is in B if and only if x/2 is in B;
c. If y ∈ S is odd, then y is in B if and only if (y + 15)/2 is in B.
Which elements of S cannot be part of a special subset?
34. A semicircle is inscribed in a quadrilateral ABCD in such a way that the midpoint
of BC coincides with the center of the semicircle, and the diameter of the semicircle
lies along a portion of BC. If AB = 4 and CD = 5, what is BC?
35. What are the two smallest positive integers n for which 360n has exactly 72 positive
divisors?
36. How many distinct triangles with positive integer sides have perimeter equal to 100?
(Triangles are distinct if they are not congruent.)
37. Let {x} = x−[x] denote the fractional part of x. For example, {3.4} = .4 and {4} = 0.
Let f (x) = {3x/2}. How many solutions does the equation f (f (f (f (x)))) = 0 have
in the interval 0 ≤ x < 1?
38. Each side of a regular dodecagon A1 A2 A3 . . . A12 has length 2. What is the area of
the pentagon A1 A2 A3 A4 A5 ?

3
39. Flip a fair coin repeatedly until the sequence HTH occurs. What is the probability that
the sequence THTH has not yet occurred? (For example, the event whose probability
we seek includes the sequence HHTH, but not the sequence TTHTH.)
40. In the rhombus in problem 26, what is the tangent of the acute angle of the rhombus?
(Your answer should involve square roots, not trig functions.)

4
Solutions to 2006 Contest
Numbers in brackets are the number of people who answered the problem correctly,
first out of the 36 people who scored at least 15, and then out of the other 151 people.

4 3
1. 12. [36,140.5] 1/( 12 − 12 )

2. 12. [35,137] D = J + 6. D − 6 = 2(J − 6). Therefore, J = 2J − 12, so J = 12.


3. 12. [34,61] If d is the distance between A and B, then his time is d/10 + d/15 =
5d/30 = d/6. Thus his overall speed is 2d/(d/6) = 12.
4. 31/6. [32,89] It must be 7/2 + 5/3.
5. 9. [25,18] It is equivalent to say that 1 + x is a square. Then x can be 22 − 1, 32 −
1, . . . , 102 − 1.
6. 4. [34,42] Either x2 − 25x + 144 = 0 or x2 + 25x + 144 = 0. The first has solutions 9
and 16, while the second has solutions −9 and −16.
7. 71. [22,38] We must have y = 2z and then x + 7z = 500. Now z can equal any number
from 1 to 71, and x will be uniquely determined.
8. 25. [34,65.5] If ` denotes the length of the ladder and h the height to which it extends
initially, then `2 = h2 +225 and `2 = (h−13)2 +242 . Thus 26h = 169+576−225 = 520.
Hence h = 20. Then ` = 25.
9. 159. [33,65] It is 3 + 29 + 127, since 1, 27, and 125 are the first odd cubes.
10. 3. [28,114] The correct answers must have been either FFFTTT or TFTTTT. Either
way, Chris got 3 right.

11. 5/3. [33,94] The hypotenuse is 3. The smaller angle has the larger cosine.
12. 16. [26,21] Let the circle be x2 + y 2 = r2 and the diameters be the x- and y-axes. If
the
p coordinates of P are (x, y), then A and B are at (x, 0) and (0, y). Thus AB =
x2 + y 2 = r. Since AB = 8, the diameter is 2r = 16.
13. 9. [11,13] The given number is 1011 −10001. It square is 1011 (1011 −20002)+100020001.
The number of 9’s in this equals the number in 1011 − 20002 = 99999979998.
14. 15. [7,5] Compress the horizontal band between the lines y = 0 and y = 2. Then
the shortest distance between A and D is a straight line, and their displacement is 12
horizontal units and 5 vertical units (with the 2 compressed units omitted). This line
has length 13, and the 2 must be added back.
√ √ √
15. 2. [21,11.5] x2 = 3 + 2 2 + 3 − 2 2 − 2 9 − 8 = 6 − 2.

16. 2 3 2 or 24/3 . [20,9] If one vertex is at (x, 0), then the side of the square is 2x satisfying
2x = 1/x2 . Thus x3 = 1/2, and the area is 4x2 = 24/3 .

17. 6. [30,51] Since 93 = 729 and 103 = 1000, 3 829 is slightly greater than 9 (approx
9.3). Since log10 100 = 2 and log10 1000 = 3, log10 829 is slightly less than 3 (approx

5
2.9). Thus the answer is approximately (9 + 3)/2. (Without making estimates, clearly
the sum is between 11 and 13, and so 1/2 times the sum is definitely closest to 6.)
18. 40/3. [14,10] The area of the entire triangle is 54. The area of the triangle between
the first line and opposite vertex is 49 · 54 = 24. Four ninths of that area is removed
when the second line is drawn. What remains is 59 · 24.
19. 96. [28,31] The diagonals of a rhombus are perpendicular. If b denotes half the length
of the shorter diagonal, then the rhombus is composed of four right triangles with
bases b and b + 2 and hypotenuse 10. Thus b2 + (b + 2)2 = 100, hence b2 + 2b − 48 = 0,
so b = 6. The area of each of the triangles is 6 · 8/2.
20. 32. [35,40] We have k = a2 b and k = ab3 . Thus a = b2 . The smallest allowable values
are b = 2, a = 4.
21. 270−2B. [20,12] In the diagram below, 8x = 360, so x = 45. Now A+2(B−45) = 180.
C...................................................D
...
... 3x
. ...
...
.... ...
... ...
... ...
..... ...
...
... ...
..
B x
. . ..
....................................................................................................
.
.
.....
..... .....
. E
..... .....
..... .....
..... .....
..... .
......
.
..... .
..... .....
..... .....
..... .....
..... ........
........
A

22. 23. [27,27] The sum of these five sums will include each of the people four times. Thus
the sum of all their ages is 660/4=165. So the youngest is 165 − 142 = 23.

23. 1 − 13 6. [20,17.5] Adjoining to the figure the reflection of the triangle across its
hypotenuse yields a square. The area of the triangle mustpequal 1/3 times the area of
the square, i.e. 1/3. Thus 12 (1 − x)2 = 13 and so 1 − x = 2/3.
24. 57. [5,2] Since a3 = 17a + 19, etc., we have a3 + b3 + c3 = 17(a + b + c) + 57. Since
x3 − 17x − 19 = (x − a)(x − b)(x − c), a + b + c equals the negative of the coefficient
of x2 , which is 0.
25. 0 ≤ k ≤ 1. [6,3] 2x−1 x−3
x+2 − 1 < 0 is equivalent to x+2 < 0, which is equivalent to
−2 < x < 3. We require that the open interval from k − 2 to k + 2 be contained in
this.

26. (1 + 5)/2. [7,0] In the diagram below, triangles BOC and ABC are similar. Thus
s : 1 = BC : CO = AC : BC = (s + 1) : s. Hence s2 = s + 1. (Remark: this figure is
a foundational figure in Penrose’s aperiodic tilings.)

B.................................................................................................................... C
...
...... .. .
...... ..
1.............................1........... ........
..
...
....
...... ....O
.
... ...... ...
... ...... ... ...
... ...... ... ...
.
.
...
...
s.........
......
.....
...
... ....
.
.
..
. ... ..
... ...... ... ...
.. ....... ... ...
... .......
A s
.........................................................................................

6
27. a > 0 or a < −1. [14,1] The equation becomes (x + 1)(a + 1)x = − 14 , or (a + 1)x2 +
(a + 1)x + 14 = 0. This has two distinct real solutions if (a + 1)2 − (a + 1) > 0 hence
(a + 1)a > 0. Both factors are positive if a > 0 and negative if a < −1.
28. (−1/5, 7/5). [5,0] Let (x, y) denote the components of either point P . Then x2 + y 2 =
2, and (0, 0) and (4, 2) form the hypotenuse of a right triangle with other vertex at
(x, y). Thus x2 + y 2 + (x − 4)2 + (y − 2)2 = 42 + 22 , and hence 2 = x2 + y 2 = 4x + 2y.
Therefore x2 + (1 − 2x)2 = 2; i.e. 5x2 − 4x − 1 = 0, and so x = 1 or −1/5.
29. 8. [16,22] In general, n must be of the form 2 + 3k. To see this, inscribe the polygon
in a circle as below. If AEB, BEC, and CED trisect the angle, then the arcs AB,
BC, and CD are equal. The angle must be chosen so that these three arcs can be
filled in with k sides of the regular polygon.
.... ..
........................................................
................ ..............
B...................................... ........... C
.........
..... .... ... ......
....
.... ... ..... ..........
...... ... . ......
...... ... ... ......
..... ... ... .....
... .....
........ ...
..
.
.
..... .
.... ... .....
..... . ... .
... .
........
...... ... ..
. .
.. . .
.
... .. ... .
. . .
...... ... ... ......
...... ... ... ......
...... ... .... . ........
.
.......... . ...
............... ..... .... ......................
A ................. .. .. ................
................................................. D
E

30. n2 + 5n − 3. [13.5,5] We have an = 4 + 2n. Since the differences in the b-sequence are
linear in n, the sequence itself will be quadratic. If bn = An2 + Bn + C, then

An2 + Bn + C = A(n − 1)2 + B(n − 1) + C + 4 + 2n,

and hence 2An = A − B + 4 + 2n. Therefore A = 1 and B = 5. Finally 1 + 5 + C = 3.


31. 981. [8,1] If n is a 2-digit number a1 a0 , then n − d(n) = 9a1 . Thus the multiples
of 9 up to 81 can be achieved as n − d(n). If n is a 3-digit number a2 a1 a0 , then
(n − d(n))/9 = 11a2 + a1 . Since a1 can go from 0 to 9, (n − d(n))/9 cannot equal
11 − 1, 22 − 1, . . . , 99 − 1, but can equal all other numbers in this range. The largest
(n − d(n))/9 can be for a 3-digit number is 99 + 9 and the smallest it can be for a
4-digit number is 111. Thus 9 · 109 is our desired answer.
32. 990. [9,0] 60 = 22 · 3 · 5. Hence 605 = 210 35 55 . Thus

S = {2a 3b 5c : 0 ≤ a ≤ 10, 0 ≤ b, c ≤ 5}.

Note S contains 11 · 6 · 6 numbers. For each a, 2a will occur as the 2-power in 36 of


those numbers. Thus the exponent of 2 in the product is 36(0 + 1 + · · · 10) = 36 · 55.
Similarly the exponents of 3 and 5 in the product are each 66(0 + 1 + · · · 5) = 66 · 15.
This latter number is the desired exponent e. Note that 36 · 55 = 2 · 990.
33. 5, 10. [9,19] The subsets of S which satisfy rules (b) and (c) are {1, 2, 4, 8}, {3, 6, 9, 12},
{7, 11, 13, 14}, and {5, 10} and any union of these. To satisfy rule (a), we must take
the union of two of the 4-element subsets just described.

7

34. 4 5. [0,0] The indicated angles in the figure below are equal in pairs. (One way to
see this for θ is to reflect triangle EBO through the origin.) Thus 360 = 2θ + 2φ + 2ψ,
and so 6 BOA = ψ and 6 COD = φ. Thus triangles BOA and CDO are similar, and
hence BO/AB = CD/OC. If x = BC, this implies (x/2)2 = 4 · 5.

D ..
...............................
...........................................
A ..
.. ...
.
......... ........
.
. .
..
.
. .
.....
. ..........................
...
ψψ .......... ..... .....
..... ..
... ............ .....
.
φφ ... ...........
... ..... ..
........... .....
.
...
...
... ....
.
... ...... ..
........
........
...... ... ... .
...
........
.......
. ...
... .
.
... ..
. ........
......
.. .
E .
.
.
....
.
.
.. ......
.
.
...
....
...
...
... ..
...
.. ....
..........
...... ..........
....... .......... ... .
.
... .
.
.
. ........
......
..... ...... . .
. .
....
.. .. ...... ... ...
... ..... ... ....
... ... ...... ... . ... ...... ... ...
... .... ......
...... ... .... .
.
. ..........
. ... ....
.
.. ... ...... ... .. .. ..... . ... ...
.
. .. ...... .. .. .. . . . ...
. .. ... ....
.. .
. .
...... ... .. .. ...... . . ....
... ..
. .. . . .. ... ...
.. ...... ... .. .. ...... .. ...
B θ θ C
. .
. . .. . . . . . .
..........................................................................................................................................................................................................................................
O

35. 28 and 35. [2,0] We must have 360n = 2a 3b 5c 7d with a ≥ 3, b ≥ 2, c ≥ 1, and d ≥ 0,


and
(a + 1)(b + 1)(c + 1)(d + 1) = 2 · 2 · 2 · 3 · 3.
The possibilities for (a + 1, b + 1, c + 1, d + 1) are (8,3,3,1), (6,6,2,1), (6,3,4,1), (6,3,2,2),
(4,9,2,1), (4,6,3,1), (4,3,6,1), (4,3,3,2), (4,3,2,3). The extra factors that they give
beyond the minimal (4,3,2,1) are, respectively, 24 5, 22 33 , 22 52 , 22 7, 36 , 33 5, 54 , 5 · 7,
72 .
36. 208. [4,0] Let A1 ≤ A2 ≤ A3 denote the three sides. We must have A3 < A1 + A2 and
A3 = 100 − A1 − A2 . Thus 50 < A1 + A2 and A2 ≤ 100 − A1 − A2 . This yields
100−A1
max(A1 , 51 − A1 ) ≤ A2 ≤ 2

as the range of values for A2 . Then A3 is uniquely determined by A1 and A2 . As A1


goes from 2 to 33, the number of admissible values of A2 goes

1, 1, 2, 2, 3, 3, . . . , 12, 12, 12, 10, 9, 7, 6, 4, 3, 1.

The sum of these is 208. One way to quickly add them is to pair the opposite numbers
among the first 24 and to pair opposite numbers among the last 12. This yields 16 · 13.
37. 8. [3,2] By looking at the graph of f (x) for 0 ≤ x < 1, we see that the equation
f (x) = y has 1 solution, x = 2y/3, if 1/2 ≤ y < 1, and 2 solutions, 2y/3 and 2(y+1)/3,
if 0 ≤ y < 1/2. First, f (x) = 0 implies x = 0 or 2/3. Then, f (f (x)) = 0 implies
f (x) = 0 or 2/3, and so x = 0, 2/3, or 4/9. Next, f (f (f (x))) = 0 implies f (x) = 0,
2/3, or 4/9, and so x = 0, 2/3, 4/9, 8/27, or 26/27. Finally, f (f (f (f (x)))) = 0 has
2 + 1 + 2 + 2 + 1 solutions.

38. 5 + 2 3. [1,0] The interior angle at each Ai is 150o and so the area of each of the
two skinny triangles in the diagram below is 12 · 2 · 2 sin 150 = 1. If s = A1 A3 , then

8

s2 = 22 + 22 − 2 · 2 · 2 cos 150 = 8 + 4 3. Angle A√1 A3 A5 equals 150 − 15 − 15 = 120.

1 2
The area of triangle A1 A3 A5 is 2 s sin 120 = 3 + 2 3. Thus the total area is 5 + 2 3.

A....3
A2........................................................................................................................A
... 4
... .... ....... .....
..... ....... ....... .....
..... ....... ....... .....
..
................... ....... .....
............
.........
. ....
A1 ...............................................................................................................................
.
...
A5

39. 5/8. [3,2] Let E denote the event that HTH occurs before THTH. If s denotes a
sequence, let P (E|s) denote the probability that E occurs given that the sequence
starts with s. Let x = P (E|H), i.e. the probability that E occurs given that the
sequence starts with an H, and similarly y = P (E|T ). Then

x = 21 P (E|HH) + 14 P (E|HT T ) + 41 P (E|HT H) = 12 x + 14 y + 1


4

and

y = 12 P (E|T T ) + 14 P (E|T HH) + 18 P (E|T HT T ) + 18 P (E|T HT H) = 12 y + 14 x + 81 y.

Here we have noted that whenever HH occurs, it is like starting over with an H, and
similarly for TT. Solving these two equations yields x = 34 and y = 12 . Thus the
desired probability is 21 x + 12 y = 58 .
p √ √
40. 5 + 2 5. [2,0] Recall that s = (1 + 5)/2 and satisfies s2 = 1 + s. Let (x, y) denote
the coordinates of the distinguished point on the diagonal considered in Problem 26,
if A is at (0, 0). Then x2 + y 2 = s2 and (x − s)2 + y 2 = 1. This implies that
s2 − 2xs + s2 = 1, and so x = (2s2 − 1)/(2s) = (2s + 1)/(2s) = (1 + s)/2. Then
y 2 = s2 − (1 + s)2 /4 = (3s2 − 2s − 1)/4 = (s + 2)/4. The coordinates of the point C
in the diagram above are s+1 1 1
s (x, y) = (sx, sy). We have sx = 2 (1 + s)s = s + 2 . Thus
the x-component of B is 1/2. The y-component of B equals

p q
s
√ 1 1
√ 1

2 s+2= 2 (1 + s)(2 + s) = 2 3 + 4s = 2 5 + 2 5.

The desired tangent now equals 2 times the y-component of B, which is as claimed.

9
1 5 1
1. By how much does 3 of 2 exceed 2 of 13 ?
2. What fraction of the area of a circle of radius 5 lies between radius 3 and radius 4?
3. A ticket fee was $10, but then it was reduced. The number of customers increased by
50%, but the amount of money received only increased by 20%. How many dollars
was the reduced ticket price?
4. If 6x + 7y = 2007 and 7x + 6y = 7002, then what is the value of x + y?
5. What is the sum of the digits of 1055 − 55?
6. There are three consecutive positive integers such that the square of the second minus
twelve times the first is three less than twice the third. What is the smallest of the
three integers?
7. How many perfect cubes greater than 1 are divisors of 99 ?
8. The harmonic mean of two numbers is defined to be the reciprocal of the average of
the reciprocals of the numbers. Find x such that the harmonic mean of 4 and x is 6.
9. What is the smallest possible value of |x − 1| + |y − x| + |y − 2007| for any real numbers
x and y?
10. What is the value of log2 (log2 (log2 (16)))?
11. Two fair 6-sided dice, colored red and white, are tossed. What is the probability that
the number on the red die is at least as large as that on the white die?
12. What is the smallest positive number which is equal to the cube of one positive integer
and also is equal to the fourth power of a different positive integer?
13. What value of c occurs in a solution of the system of equations a + b + c = 14, ab = 14,
and c2 = a2 + b2 ?
14. If 11 positive integers a1 ≤ a2 ≤ · · · ≤ a11 have the property that no three of them
are the sides of a triangle, what is the smallest possible value of a11 /a1 ?
15. A cube and sphere have the same surface area. What is the ratio of the volume of the
sphere to that of the cube?
16. A rhombus ABCD with sides of length 5 lies in the first quadrant with A at the
origin. If AD has slope 1/2 and AB has slope 2, then what are the coordinates of C?
17. For how many primes p is p2 + 3p − 1 also prime?
18. A large container, labeled R, is partially filled with 4 quarts of red paint. Another
large container, labeled W , is partially filled with 5 quarts of white paint. A small
empty can is completely filled with red paint taken from R, and the contents of the
can then emptied into W . After thorough mixing of the contents of W , the can is
completely filled with some of this mixture from W , and the contents of the can then
emptied into R. The ratio of red paint to white in R is now 3:1. What is the size of
the can, in quarts?

1
19. What is the largest number A such that the graphs of x2 = y 2 and (x − A)2 + y 2 = 1
intersect?
20. A number is written with one 1, followed by three 3’s, then five 5’s, then seven 7’s,
then nine 9’s, then eleven 11’s, etc. Thus it begins 13335555577. . . , and just as it gets
to the 13’s it has . . . 11131313. . . . What is its 999th digit?
21. What is the radius of a circle inscribed in an isosceles right triangle whose legs have
length 1?
22. In what base b is the equation 53 × 15 = 732 valid? Here all three numbers are base-b
numbers, and b must be a positive integer.
23. A strip of rubber is initially 80 cm long, and after every minute it is instantaneously
and uniformly stretched by 40 cm. An ant moves at a rate of 42 cm per minute,
starting at one end. Each time the strip is stretched, the ant is moved to a position
on the modified strip proportional to its position on the strip before the stretching
took place. How many minutes does it take the ant to cross the strip?
24. The first term of an infinite geometric series is 10, and the sum of the series lies
between 9 and 11, inclusive. What is the range of values for the common ratio r
between terms of this series?
25. If the roots of x2 − bx + c = 0 are sin(π/9) and cos(π/9), then express b in terms of
c. (Don’t write “b =”; just write the expression involving c.)
26. Let R denote the set of points (x, y) satisfying x2 − 4|x| + y 2 + 3 ≤ 0. What is the
area of R?
27. The sum of the 3-digit numbers 35x and 4y7 is divisible by 36. Find all possible
ordered pairs (x, y).
28. What is the ratio of the area of a regular 10-gon to that of a regular 20-gon inscribed
in the same circle? Express your answer using a single trig function with its angle in
degrees.

29. Let k > 0 and let A lie on the curve y = k x so that the vertical and horizontal
segments from A to the x- and y-axes are sides of a square with the origin at the
vertex opposite A. Let B be the vertex of this square lying below A on the x-axis.
Let C be the point on the same curve between the origin and A such that the vertical
segment from C to the x-axis and the horizontal segment from C to AB is a square
with B at the vertex opposite C. What is the ratio of the side length of the second
square to that of the first?
30. What is the smallest multiple of 999 which does not have any 9’s among its digits?
(If the number is n = 999 · d, write n, not d.)
31. Let P (n) denote the product of the digits of n, and let S(n) denote the sum of the
digits of n. How many positive integers n satisfy n = P (n) + S(n)?

2
32. For each point in the plane, consider the sum of the squares of its distances from the
four points (−3, −1), (−1, 0), (1, 2), and (1, 3). What is the smallest number achieved
as such a sum?
33. What is the remainder when x3 + x6 + x9 + x27 is divided by x2 − 1?
34. Find k so that the solutions of x3 − 3x2 + kx + 8 = 0 are in arithmetic progression.
35. In triangle ABC, D lies on BC so that AC = 3, AD = 3, BD = 8, and CD = 1.
What is the length of AB?
36. What is the ninth digit from the right in 10120 ?
37. Two circles of radius 1 overlap so that the center of each lies on the circumference of
the other. What is the area of their union?
38. The first two positive integers n for which 1 + 2 + · · · + n is a perfect square are 1 and
8. What are the next two?
39. In the diagram below, which is not drawn to scale, the circles are tangent at A, the
center of the larger circle is at B, CD = 42, and EF = 24. What are the radii of the
circles?
............E
..........................
........... . ..... ... ... ........
........ .......
...... ... ......
.
......... .............. .........
..
.
.....
.....
..
.... .............. ...........
. .
. .....
... ......... .. ...........
....
.
......
.
. ..... F .
.
.
.
.
.
..
......
.
....
...
...
........ .
.
.
....
. ...
...... .
. ... ...
..... .
. ... ...
. .
.... .
. ... ...
. . ...
....... .... ...
...
...
... .. ... ...

A
..
...
B ..
............................................................................................................................................................
. D
.
. . ...
...
...
.....
.
....
..
C
...
.
...
...
...
. .. ..
...... .... .. ..
...... .. ..
...... .. ... ..
....... ...
..... ....
......... . . .
... ..... ..
... ...... ... ........... ...
.... ...... ..
..... ........
..... .............. .
............
. . ..
......
..... ........................... ....
...... ... ......
....... ......
........
...........
..
. ..
. ..
. .........
..........................................

40. What is the side length of an equilateral triangle ABC for which there is a point P
inside it such that AP = 6, BP = 8, and CP = 10?

3
Solutions to 2007 contest
Numbers in brackets are the number of people who answered the question correctly,
first out of the 28 people who scored at least 20, and then out of the other 228 people.
5
1. 2/3. [28, 193] The difference is 6 − 16 .
2. 7/25. [27, 138.5] The fraction is (42 − 32 )/52 = 7/25.
3. 8. [24, 136] If the new fee is x and the original number of customers was N , then
1.2 · 10N = 1.5N x. Thus x = 12/1.5 = 8.
4. 693. [28, 105] Add the equations to get 13(x+y) = 9009. Thus x+y = 9009/13 = 693.
5. 486. [22, 81] The number has 53 9’s followed by the digits 45. Thus the answer is
54 · 9.
6. 12. [28, 141] If x is our desired number, then (x + 1)2 − 12x = 2(x + 2) − 3. Thus
x2 + 2x + 1 − 12x = 2x + 1, and hence x = 0 or 12.
7. 6. [25, 46] They are 33 , 36 , 39 , 312 , 315 , and 318 .
2xy
8. 12. [24, 140] The harmonic mean of x and y equals 1/( 12 ( x1 + y1 )) = x+y . We have
6 = 8x/(x + 4). We obtain 2x = 24.
9. 2006. [25, 130] This value will be achieved whenever 1 ≤ x ≤ y ≤ 2007. The sum
must be ≥ 2006 by the general fact that |a − b| + |b − c| ≥ |a − c|.
10. 1. [28, 138] log2 (16) = 4. log2 (4) = 2. log2 (2) = 1.
1
11. 7/12. [25.5, 101.5] There is 1/6 chance that they are equal, and then 2 (1 − 16 ) that
the red die is greater. The desired answer is 16 + 12
5
.
12. 4096. [26, 66] This is 212 which equals 163 and 84 . Note that 1 is ruled out since it
is not the cube and fourth power of different integers. To see that there is nothing
smaller, one can easily check that none of 24 , . . . , 74 is a cube.
13. 6. [25, 37] We have c2 = (a + b)2 − 28 = (14 − c)2 − 28. We obtain 28c = 196 − 28,
hence c = 6.
14. 89. [24, 21] We may set a1 = 1. To make the numbers as small as possible, we should
choose a2 = a1 and an = an−2 + an−1 for n ≥ 3. Thus we obtain the Fibonacci
numbers 1, 1, 2, 3, 5, 8, 13, 21, 34, 55, and 89.
p
15. 6/π. [15.5, 23] If s is the side of the cube and r the radius of the sphere, then 6s2 =
4πr2 and so (r/s)2 = 6/(4π). The ratio which we desire is 34 π(r/s)3 = 34 π(6/4π)3/2 =
p
6/π.
√ √
16. (3
√ 5, 3 5). [23,√ 49] The vector AC is the sum of AD and AB, which are, respectively,
5h2, 1i and 5h1, 2i.
17. 1. [22, 58] For p = 3, we obtain the prime 17. Every prime p 6= 3 is of the form 3k ± 1
for some k and hence has p2 − 1 a multiple of 3. Thus p2 + 3p − 1 is also a multiple
of 3.

4
18. 5/4 or 1.25. [15, 19] Let x denote the size of the can. After the first transfer, R has
x
4−x red, while W had 5 white and x red. After the second transfer, R has 4−x+x 5+x
5
red and x 5+x white. Thus (4 − x)(5 + x) + x2 = 3 · 5x. This simplifies to 20 = 16x.

19. 2. [21, 16] Method 1: x2 − 2Ax + A2 + x2 = 1. Thus 2x2 − 2Ax + A2 − 1 = 0.
This has a solution when its√discriminant, (−2A)2 − 8(A2 − 1), is nonnegative. Thus
8 − 4A2 ≥ 0. Hence |A| ≤ 2. Method 2: The circle of radius 1 centered at (A, 0)
will be tangent to the line y = x. The radius to the point of tangency will have slope
−1. This radius, the tangent line from (0, 0), and the line from (0, 0) to (A, 0) form a
45-45-90 triangle with legs of length 1. Thus A2 = 12 + 12 .
20. 5. [9, 48] The sum of the odd integers through 2m − 1 is m2 . Going through the 9’s,
there will have been 25 digits. At the end of the 43’s, we will have used 2·222 −25 = 943
digits. The reason for the 25 subtracted comes from thinking of the digits through the
9’s as if they had been paired, but then subtracting off for the fact that they weren’t.
The 45’s will take us beyond digit 999; the 4’s will occur at even-numbered digits and
the 5’s at odd-numbered.

21. 1 − 22 . [21, 14] Method 1: Place the triangle with vertices at (0,0), (1,0), and (0,1).
The center of the inscribed circle must be at (r, r) such that r2 = ( 12 − r)2 + ( 12 − r)2 .
These are two forms for r2 . The equation reduces to 0 = r2 − 2r + 12 . The quadratic
√ √
formula gives r = 1 ± 21 2. Method 2: The altitude to the hypotenuse equals 2/2

and also equals r + r 2.
22. 13. [26, 26] We must have (5b + 3)(b + 5) = 7b2 + 3b + 2. Thus 2b2 − 25b − 13 = 0.
Find b = 13 or −1/2 either by factoring or by the quadratic formula.
23. 2 10
21 . [19.5, 33.5] After one minute, it will be 42/80. After the stretching, it will be
63/120. After the second minute, it will be 105/120. After the second stretching, it
will be 140/160. It requires another 20/42 minutes to get to the end.
24. − 19 ≤ r ≤ 11
1 10
. [18, 26.5] We have 9 ≤ 1−r ≤ 11. We obtain 11r ≤ 1 and 9r ≥ −1.

25. 1 + 2c. [12.5, 4.5] We have c = sin(π/9) cos(π/9) and b = sin(π/9) + cos(π/9). Then
b2 = 1 + 2 sin(π/9) cos(π/9), since sin2 (−) + cos2 (−) = 1. Hence b2 = 1 + 2c. Finally
note b > 0 so we choose the positive square root.
26. 2π. [11, 2] For x ≥ 0, we require x2 − 4x + 4 + y 2 ≤ 1, hence (x − 2)2 + y 2 ≤ 1, which
is a disk of radius 1 with all x > 0. For x ≤ 0, we require x2 + 4x + 4 + y 2 ≤ 1, hence
(x + 2)2 + y 2 ≤ 1, which is a disk of radius 1 with all x < 0. Thus R consists of two
disks of radius 1, and has area 2π.
27. (1, 7) and (5, 3). [22, 38] We have 350 + x + 407 + 10y = 36j for some j. Since
757 = 36 · 21 + 1, we have 10y + x = 36k − 1 for some k. Since 0 ≤ 10y + x ≤ 99, we
have 10y + x = 35 or 71.
28. cos(18o ). [14, 2] Let the radius of the circle equal 1. The 10-gon consists of 20 right
triangles with hypotenuse 1 and an angle of 18 degrees. Thus the area of the 10-
gon is 20 · 12 sin(18) cos(18) = 10 sin(18) cos(18). Similarly the area of the 20-gon is

5
20 sin(9) cos(9). By the double angle formula, this latter equals 10 sin(18). Thus the
ratio is cos(18).

29. ( 5 − 1)/2. [7, 1] A lies on √
the intersection of the curve with the line y = x and so its
x coordinate satisfies
√ x = k x, hence x = k 2 . The side length s of the second
√ square
2 2 4 1 2
satisfies s = k k − s. Thus s + k s − k = 0 and hence s = 2 (−k + k + 4k 4 ).
2 4

The desired ratio s/k 2 equals 21 (−1 + 1 + 4).
30. 111888. [15, 21] The multiples of 999 have the form 1000d − d. If d ≤ 100, this has a
9 in the 100’s position. If 101 ≤ d ≤ 110, it has a 9 in the 10’s position. If d = 111, it
has a 9 in the 1’s position. If d = 112, it equals 111888.
31. 9. [16, 29] Clearly no 1-digit number works. If n = 10a + b is a 2-digit number
satisfying the required property, then 10a + b = ab + a + b and so b = 9, yielding the
nine numbers 19, 29,. . . ,99. If n = 100a + 10b + c is a 3-digit number, we would need
100a + 10b + c = abc + a + b + c and so 99a + 9b = abc. Since bc ≤ 81, this has no
solutions. A similar argument will show that there will be no solutions if n has more
than three digits.
32. 21. [9, 3] For the point (x, y), this sum is (x+3)2 +(y+1)2 +(x+1)2 +y 2 +(x−1)2 +(y−
2)2 +(x−1)2 +(y −3)2 = 4x2 +4x+12 +4y 2 −8y +14 = (2x+1)2 +11+4(y −1)2 +10.
The minimum value of this occurs when x = −1/2 and y = 1, and is 21.
33. 3x + 1. [4, 5] The remainder must be a first-degree polynomial ax + b. We have
x3 + x6 + x9 + x27 = q(x)(x2 − 1) + ax + b. Letting x = 1 yields 4 = a + b, while letting
x = −1 yields −2 = −a + b. We solve these equations, obtaining a = 3 and b = 1.
34. −6. [13, 6] Let the roots be a − d, a, and a + d. The sum of the roots is the coefficient
of −x2 ; hence a = 1. The negative of the product of the roots equals the constant
term. Thus d2 − 1 = 8. Hence the roots are −2, 1, and 4. Then k is the sum of
products of pairs of roots, so equals −2 − 8 + 4.
35. 9. [15, 24] Let AE be the altitude from A, and let h denote its length. Note that
triangle ACD is isosceles, and so ED = EC = 1/2. We have h2 = 32 − ( 21 )2 , and
AB 2 = h2 + ( 17 2 2
2 ) . Thus AB = 9 +
16·18
4 = 81.

B.
..... .
.....
.. ...
.. ...
.... ....
. ..
... ..
.. ...
.... .....
. ...
... ...
.. ...
.... ...
.. ...
... ...
.. ...
... ...
.. ...
... ...
.. ...
...
.. ...
.. ...
... ...
.. ...
... ...
.. ...
... ...
. ...
.... ...
.. ...
... ...
.. ...
.. ...
...
...
..
. ... . . . . . . . . D
.........
...
.. ..
...
. .
A ... ..............................
.........
..
........................................................................
................... E .
C
6
36. 6. [8, 4] We expand
¡20¢ 2 ¡20¢ ¡20¢
(100 + 1)20 = 1 + 20 · 100 + 2 100 + 3 100 3
+ 4
4 100 + · · · .

Neither the first three terms nor the omitted terms at the end will affect the ninth
digit from the right. The fourth and fifth terms are 1140000000 and 484500000000.
The desired digit is 1 + 5.

37. [2, 1] 4π 3
3 + 2 . Their overlap is twice the area A to the right of the vertical line in the
diagram below. This vertical line is at radius 1/2. Thus the indicated angle is 120o ,
and so the area of the wedge subtended by the√ angle is π/3. Then A equals π/3 minus √
the area of the indicated triangle, which is 12 23 . Thus the area of overlap is 2π
3 − 3
2 .
Finally the area of the union is 2π minus the overlap.
...
.............. ....................
....... .. ..
....... ..... ......
...
. ... .. ....
....
. .
.... .... .....
.. ... ...
... ... ...
... ... ... ...
..... .... ...
. ..
...
...
...
... .
.
. . ...
... ... ... . ...
... ... . .
.
... ... .. ....
..... ... ... .....
...... ............
........ . .
..............................

38. 49 and 288. [9, 1] We need that n(n + 1) is 2 times a perfect square. Since n and
n + 1 contain no common divisors, one must be a square and the other twice a square.
The square must be odd. We examine the first few odd squares m2 to see whether
m2 − 1 or m2 + 1 is 2 times a square. For m2 − 1, we obtain 8, 24, 48, 80, 120, 168,
224, and 288. For m2 + 1, we obtain 10, 26, 50, 82, 122, 170, 226, and 290. Of these
8, 50, and 288 are 2 times a perfect square. The pairs (n, n + 1) are (8, 9), (49, 50),
and (288, 289).
39. 75, 96. [11, 1] Let R (resp. r) denote the radius of the large (resp. small) circle. Then
R = r + 21. Let O denote the center of the small circle. Referring to the diagram
on your exam sheet, the right triangle OF B implies r2 = (R − 24)2 + 212 , and hence
r2 = (r − 3)2 + 441, which implies 6r = 450.
p √
40. 2 √25 + 12 3. [3,√ 0] Let s be the desired length, and place the triangle with A =
(s 23 , 2s ), B = (s 23 , − 2s ), and C = (0, 0). If P = (x, y), we have x2 + y 2 = 100,
√ √
(x − 23 s)2 + (y + 2s )2 = 64, and (x − 23 s)2 + (y − 2s )2 = 36. The second and
third imply √ sy = 14. Substitute this and the 2first into the second and get sx =
2
(s + 50)/ 3. Multiply the first equation by s and substitute our expressions for
sx and sy, √obtaining an equation which reduces to s4 − 200s2 + 3088 = 0. Get
s2 = 100 + 6912.

7
If a question asks you to “find all” or “list all” and you think there are none, write “None.”

1. Simplify 1/( 13 − 14 ).
2. The price of an item increases by 10% and then by another 10%. What is the overall
price increase of the item, in percent?
3. What is the area of a triangle with sides 5, 5, and 6?
4. A student has scores of 71, 81, 74, and 86 on her first four exams. What range of
scores on the fifth exam will leave her in the C-range? The C-range is the interval
[70, 80), i.e., all averages satisfying 70 ≤ ave < 80. You should express your answer
either in interval notation or else as an inequality.
5. What is the volume of a cube inscribed in a sphere of radius 1?
6. You randomly select two distinct integers from 1 to 10. What is the probability that
they are consecutive numbers; i.e., that they differ by 1?
7. Which of the following numbers is the largest? (Leave your answer in the given form.)
sin(20o ), cos(20o ), tan(20o ), 1/ sin(20o ), 1/ cos(20o ), 1/ tan(20o ).
8. How many real solutions are there to the equation x4 + |x| = 5?
9. Write 104060401 as a product of factors which are primes or powers of primes.
p √ p √
10. Simplify 19 + 297 − 19 − 297.
2007
X
n−1
11. Let Sn = 1 − 2 + 3 − 4 + · · · + (−1) n. What is the value of Sn ?
n=1

12. Rectangle ABCD is divided into four subrectangles by two lines, one parallel to AB
and the other parallel to BC. Three of the subrectangles have areas 1, 2, and 3. What
are the possible values for the area of the rectangle ABCD?
13. Find all ordered pairs (x, y) of positive real numbers such that 3, x, y is a geometric
progression, while x, y, 9 is an arithmetic progression.
14. Let f (x) = 6/(1 + 7ex ) with domain all real numbers x. How many integers are in the
range of f ? (Here e = 2.78128 . . . . You may replace ex by 2x if you wish; the answer
will be the same.)
15. Inside a regular hexagon ABCDEF is drawn triangle ACE. What is the ratio of the
area of triangle ACE to that of the hexagon?
16. In triangle ABC, 6 A = 30o and B is a right angle. The point D lies on AB so that
AD = 2 and 6 CDB = 45o . What is the length of BC?
17. John randomly selects an integer from 1 to 9, and so does Mary, independently of
John’s choice. What is/are the most likely value(s) for the units digit of the sum of
their numbers?

1
18. Out of all positive real numbers x and y, what is the smallest value of x + y for which
x + y ≤ xy?
1 2 3 99
19. Write a simple value for the sum 2! + 3! + 4! + ··· + 100! . Your answer may involve
the factorial symbol.
20. Two squares of sidelength 1 have a vertex of one at the center of the other. What is
the smallest possible value for the area of their intersection?
21. Triangle ABC has tan(A) = 3/4 and tan(B) = 21/20. What is AC/BC?
22. If n is a positive integer, let r(n) denote the number obtained by reversing the order
of the digits of n. For example, r(16) = 61. For how many 2-digit positive integers n
is n + r(n) a square of a positive integer?
23. Find all ordered pairs (x, y) of positive integers such that x4 = y 2 + 71.
24. How many diagonals are there in a regular 19-gon? (A diagonal is a line connecting
two non-adjacent vertices.)
25. What is the sum of all positive integers n for which there exists a positive integer m
satisfying 6(n! + 3) = m2 + 5?
26. Three circles of radius 1 are tangent to one another. What is the area of the small
region bounded by the three points of tangency and minor arcs of the three circles?
27. A square train car is moving forward at 60 miles per hour. An ant in a front corner of
the train crawls toward the diagonally opposite corner on the floor of the car at 1 mile
per hour. At what speed in miles per hour is the ant moving relative to the ground?
28. A triangular number is one which is the sum of the first n positive integers for some
n. An equidigit number is one all of whose digits are equal to one another. List all
3-digit equidigit triangular numbers.
29. How many integers between 1 and 1000 cannot be expressed as the difference of squares
of integers?
30. A silo-shaped figure is formed by placing a semicircle of diameter 1 on top of a unit
square, with the diameter coinciding with the top of the square. What is the radius
of the smallest circle which contains this figure?
31. Let p(x) be a polynomial of degree 4 satisfying p(2) = p(−2) = p(−3) = 1 and
p(1) = p(−1) = 2. What is p(0)?
32. A cube of sidelength 4 has top square ABCD directly above bottom square EF GH.
Thus A is directly above E, etc. Point X lies on AD at distance 1 from A. Point Y
lies on HG at distance 1 from H. Point Z lies on BF at distance 1 from F . What is
the area of triangle XY Z?
33. A Fibonacci-like sequence of numbers is defined by a1 = 1, a2 = 3, and for n ≥ 3,
an = an−1 + an−2 . One can compute that a29 = 1149851 and a30 = 1860498. What
28
X
is the value of an ?
n=1

2
34. A circle is inscribed in an equilateral triangle of side length 2. Then another circle
is inscribed tangent to two sides of the triangle and to the first circle. An infinite
sequence of such circles is constructed, each tangent to two sides of the triangle and
to the previous circle. The figure below depicts the first three circles. What is the
sum of the areas of all the infinitely many circles?

....
... ...
..... .....
. ...
... ...
... ..
.........................................
..
......
. ........
.. ....
... ...
......
.
.....
.... ......
. .
... ...
. .. ... ....
.. ..
. ... ....................
.
.. ... .
.
. ...
... .... . ....
...... ...... .........
... . ...... ...
....................................................................................................................

35. Let an denote the n-digit positive integer all of whose digits are 1. For example,
a3 = 111. What is the greatest common divisor of a45 and a140 ?
36. Find all ordered triples (x, y, z) of integers satisfying x2 + y 2 + z 2 + 3 < xy + 3y + 2z.
37. If n is a positive integer, let s(n) denote the integer obtained by removing the last
digit of n and placing it in front. For example, s(731) = 173. What is the smallest
positive integer n ending in 6 satisfying s(n) = 4n?
38. Find all ordered triples (a, b, c) of prime numbers satisfying a(b+c) = 234 and b(a+c) =
220.
39. If n = a b c is a 3-digit number with digits a, b, and c, let

f (n) = a + b + c + ab + ac + bc + abc.

Here the things being summed are products of digits. For example, f (234) = 2 + 3 +
4 + 6 + 8 + 12 + 24 = 59. How many 3-digit positive integers n satisfy f (n) = n?
40. Let ABCD be a square of sidelength 2, and let E and F be midpoints of AD and
CD, respectively. Let G and H be the points where AF intersects EB and BD,
respectively. What is the area of quadrilateral DEGH?

3
Solutions to 2008 contest
The first number in brackets is the total number of students (out of 257) who answered
it correctly, and the second number is the number of correct responses from the 32 people
who scored at least 20.

1 1 1
1. 12. [234,32] This follows since 3 − 4 = 12 .

2. 21 or 21%. [198,29] The price is multiplied by 1.1 · 1.1 = 1.21.


3. 12. [198,32] The isosceles triangle is composed of two 3-4-5 right triangles.
4. [38, 88). [158,32] Her total so far is 312. Out of 500 points, the C-range starts at a
total of 350 and ends just before 400.
√ 2
5. 8 3/9. [53,25] Place the sphere as x √ + y 2 + z 2 = 1. One vertex of the cube is
(x, x, x) satisfying 3x2 = 1, so x = 1/ 3. Thus one side goes from (− √13 , √13 , √13 ) to
√ √
( √13 , √13 , √13 ), and hence has s = 2/ 3. The volume is thus 8/(3 3).
¡ ¢
6. 1/5. [105,28] There are 10 2 = 10 · 9/2 possible pairs of numbers, and of these, 9 pairs
(from {1,2} to {9,10}) are consecutive. The probability is 9/45.
1 cos(20o ) 1 1 1
7. 1/ sin(20o ). [144,30] We have tan(20o ) = sin(20o ) < sin(20o ) and cos(20o ) < sin(20o ) .

8. 2. [111,23] For x > 0, x4 + x is an increasing function going from 0 to ∞. Thus it


equals 5 for exactly one positive value of x. The values of x4 + |x| for negative x are
exactly the same as those of x4 + x for positive x, and so x4 + |x| = 5 for exactly one
negative value of x.
9. 1014 . [38,22] It is 1 + 4 · 100 + 6 · 1002 + 4 · 1003 + 1004 = (1 + 100)4 , and 101 is prime.
√ √
10. 22. [32,17] Squaring the given expression yields 38 − 2 361 − 297 = 38 − 16 using
the Binomial Theorem.
11. 1004. [120,31] Compute the first few values. S1 = 1, S2 = −1, S3 = 2, and S4 = −2.
This pattern will continue, and so S1 + · · · + S2006 = 0, and the desired sum equals
S2007 = (2007 + 1)/2.
12. 6 23 , 7 12 , and 12. [48,15] Arrange the subrectangles so that the unknown one is in the
lower right. If the upper left rectangle has area, respectively, 1, 2, or 3, then the lower
right one has area 6, 3/2, or 2/3, each of which must be added to 6 to obtain the total
area.
y
13. ( 92 , 27 x 2
4 ). [27,14] Since 3 = x , we have x = 3y. Since y −x = 9 −y, we have 2y = 9+ x.
2
This implies 2x = 3x + 27, and so x = (3 ± 15)/4. Only the positive solution x = 9/2
can be used.
14. 5. [64,23] The denominator can equal any real number greater than 1. Thus the range
of f is (0, 6), all positive numbers less than 6. The integers in the range are 1, 2, 3, 4,
and 5.

4
15. 1/2. [148.5,32] Let O denote the center of the hexagon. Then triangles ABC and
AOC are congruent, and similarly for two other pairs of triangles. Triangle ABC is
composed of three of these, while the hexagon is composed of six of them.
√ √ √
16. 1+ 3. [77.5,26.5] In the diagram below, we have x+2
x = 3. Thus x = √2
3−1
= 3+1.

..... C
.......... .
........... ..
.
................... ....
....... ..... ....
....... ....
...... ......... ..
....... . ..

..
...... ...........
..
.
..
.....
. .
....
. ..
...
..
x
..
...... .
....
. ..
........ .....
. ...
........ ..
.... ..
.....
... ..
...
. ..
......
. .
...
....................................................................................................................
. .
A 2 D x B

17. 0. [106,29] Out of the 81 equally likely pairs, we tabulate the number of ways each
sum can occur:
2 3 4 5 6 7 8 9 10 11 12 13 14 15 16 17 18
1 2 3 4 5 6 7 8 9 8 7 6 5 4 3 2 1

We see that a last digit 0 occurs in nine ways, while any other last digit occurs in
eight ways.
18. 4. [131,30] Adding 4xy to both sides of (x − y)2 ≥ 0 yields (x + y)2 ≥ 4xy. Thus, if
the asserted inequality holds, then 4(x + y) ≤ 4xy ≤ (x + y)2 , and so x + y ≥ 4. As
the inequality holds for x = y = 2, we find that 4 is the smallest value of x + y for
which the inequality holds.
1 n n+1 1 1 1
19. 1 − 100! . [31,19] The trick here is to notice that (n+1)! = (n+1)! − (n+1)! = n! − (n+1)! .
Thus the desired sum is
1 1 1 1 1 1
( 1! − 2! ) + ( 2! − 3! ) + · · · + ( 99! − 100! ),

and all the intermediate terms cancel.


20. 1/4. [165,31] It is always 1/4. Two perpendicular lines passing through the center of
a square divide it into four congruent parts.
.............................................................
... ...
... .. .................
........... ...
..... ...
........... ...
.. ... ..
... ...
... ... ............ .. ...
. .. .
..... .
. ...
.... . .
......... .
.
. ...
.. ...
... ............. ... .
.
.
...
....... .
.. ... .
. ...
... .
. ...
... ... .
. ...
... .
. .
.
........................................................... . .
........
... .....
. ...
... .
...
.. ...
... .......
... .............
.....

21. 35/29. [23,13.5] By the Law of Sines, or a comparison of right triangles, we have
AC sin(B)
BC = sin(A) . Looking at 3-4-5 and 20-21-29 right triangles shows that sin(A) = 3/5
and sin(B) = 21/29. Thus
AC 21 5 35
= · = .
BC 29 3 29
5
22. 8. [88,30] The numbers n are 29, 38, 47, 56, 65, 74, 83, and 92. To see this, note that
if n = 10a + b with 1 ≤ a ≤ 9 and 0 ≤ b ≤ 9, then n + r(n) = 11(a + b). For this
to be a square, we must have a + b = 11, and the listed ways are the only ways to
accomplish this.
23. (6, 35). [35,21] We have (x2 − y)(x2 + y) = 71. Thus x2 + y = 71 and x2 − y = 1.
Adding and subtracting these equations yields the only solution.
24. 152. [106,29] For an n-gon, there are n(n − 3)/2 diagonals. This is obtained by noting
that each vertex can be connected to n − 3 vertices by diagonals, but each diagonal is
counted twice in the product n(n − 3).
25. 5. [68,24] For 1 ≤ n ≤ 4, we have that n = 1 and n = 4 do not yield a corresponding
integer m, but n = 2 and n = 3 yield m = 5 and m = 7, respectively. If n ≥ 5, then
n! ends in 0, so 6(n! + 3) ends in 8, but no square ends in 3, so there are no more m’s.

26. 3 − π2 . [81,29] The centers of the circles are vertices of an equilateral triangle of

area 3. Inside the intersection of the triangle with the circles, and hence outside our
region, are three wedges of area π/6 each.
........................... ...........................
...... .... ......... ....
.....
. ... ... ...
..... ...
... .... ...
..... ..............................................
... ... .
.... .. ...
... ... .
.
.
. ... . ..
. .
....
.... . .
... .................. ... . ...
...... ..
................................. ....................................
... .. .. ....
... ..... ..... ....
... ... .. ...
.... .... ..
...
... ....
.
.... ...
...... ....
...........................

p √
27. 3601 − 60 2. [30,14] Its velocity vector is (60 − √12 )i + √1 j,
2
and so its speed is
q q √
given by (60 − √2 ) + 2 = 3600 − 60 2 + 12 + 12 .
1 2 1

28. 666. [45,24] Such a number must be of the form 111a for 1 ≤ a ≤ 9 and of the form
n(n + 1)/2 for some n. Thus n(n + 1) = 2 · 3 · 37 · a. Either n or n + 1 must be divisible
by 37. But n = 37 requires a to be divisible by 19, which it isn’t. The only solution
is obtained when n + 1 = 37, yielding a = 6. If n + 1 ≥ 74, then a is too large.
29. 250. [20,14] It is all numbers of the form 4k + 2 from 2 to 998. To see this, first note
that if n = x2 − y 2 = (x − y)(x + y), then n is the product of two even numbers or
of two odd numbers, and hence n cannot be of the form 4k + 2. On the other hand,
2k + 1 = (k + 1)2 − k 2 and 4(m + 1) = (m + 2)2 − m2 .
p
30. 5/6. [25,17] See the figure below. The radius r satisfies r + r2 − 1/4 = 3/2. Thus
r2 − 1/4 = (3/2 − r)2 , and so 3r = 10/4.
............
................. .... .........................
............ ..... ......
.
................ ...
... ... ......
.... . ... ....
.... .....
.
...
. ... ....
.
. . ... . ...
..
.. ............................................................
. ...
..
. ..
. ..
. .... ...
.... ... ... . ...
.. ... ... .
. ...
... ... .... ..
. .
... .... .... .... . ..
.
... ...
...
...
..
... r.. ....
... ...
...
... .
..
..
...
... ....
. .. ... ... ..
... ..
...
.. ... .. .....
..... .. . ... ... ....
....... . .. ..
.........................................................................
........
.......................................

6
31. 5/2. [19,12] We have p(x) = (x − 2)(x + 2)(x + 3)(ax + b) + 1, and then 2 = (−1) · 3 ·
4(a + b) + 1 and 2 = (−3) · 1 · 2(−a + b) + 1. Thus 1 = −12a − 12b and 1 = 6a − 6b.
We obtain 3 = −24b, and so p(0) = (−2) · 2 · 3/(−8) + 1 = 5/2.

32. 13 3/2. [26.5,20] See the figure below.
√ Each side of triangle XY Z is the √ hypotenuse
of a right triangle√with legs 1 and 32 + 42 . Thus the side lengths are 26, and the
desired area is 26 3/4.
A X
...............................................................................................................D
........ . .
... ...
..... .... ... ..... .. .
..... ..
.... .. .. ... ... .... ...
..
...... .... .... ..... ...
. .
.......
. ....
............................................................................................................
B ..
... ... ... .....
.
.
...
... C ....
.
..
...
..
.
.. .... ... ..... ...
... .
.
. ..
... ... ..... ... .
.
. ...
.. .. ..... ... . ..
... ... ...... ... ... ..
. . . . .
... .. . ...
.... .... .... .... ... . . ..
.. ... .... .. ... .. ..
... .. ... ... ..... ...
... ... ... ... .... ..
... .. .. . . ... .
. .
... ..
. . .
. . ...
........... .... .. ....
.
. ... ...
Z
.......
...
E ....
... .
.....................................................................................................................................................................
..................................................... H
... ........ ... .......
. ..
.... .........
... ......
. ...
Y
...................................................................................................
F G
Pn
33. 1860495. [18,12] We have that for all n, i=1 ai = an+2 − 3. (A similar, but not
identical, relation holds for the ordinary Fibonacci numbers.) The claimed relation
is easily proved by induction on n. (Verify it for n = 1; then assume it is true for n
and add an+1 to both sides to see that it is true for n + 1.) Thus the desired sum is
a30 − 3.
34. 3π/8. [15,13] Since the altitudes of an equilateral triangle
√ meet at a point one third
of the way along each, the radius of the first circle is 13 3 and so its area is π/3. All
remaining circles form a shrunken version of the entire set of circles, but shrunk by a
linear factor of 1/3. Thus the area of all the circles except the first is 1/9 of the total
desired area. If the desired area is A, then we have A = π/3 + A/9, and so A = 3π/8.
35. 11111 or a5 . [35,15] Every common divisor of a140 and a45 is a divisor of

a140 − 1095 a45 − 1050 a45 − 105 a45 = a5 .

On the other hand, a5 is a divisor of both a140 and a45 . For example, a45 = a5 (1040 +
1035 + · · · + 105 + 1). Since a5 is a common divisor and is a multiple of every common
divisor, it is the gcd.
36. (1, 2, 1). [10,6] First express the inequality as (x − y2 )2 + 34 (y − 2)2 + (z − 1)2 < 1.
Since x, y, and z are integers, this implies z = 1 and y = 1, 2, or 3. If y = 1, we
obtain x2 − x < 0, which has no integer solutions. If y = 2, we obtain x2 − 2x < 0,
whose only integer solution is x = 1. If y = 3, we obtain x2 − 3x + 2 < 0, which has
no integer solutions.
37. 153846. [22,15] Since 4 · 6 = 24, n ends 46. Since 4 · 46 = 184, n ends 846. Since
4 · 846 = 3384, n ends 3846. Since 4 · 3846 = 15384, n ends 53846. Since 4 · 53846 =
215384, n ends 153846. Since 4 · 153846 = 615384, n = 153846.

7
38. (13, 11, 7). [24,17] Since a(b + c) = 2 · 32 · 13, b(a + c) = 22 · 5 · 11, and c(a − b) = 2 · 7,
then b + c must equal 117, 78, or 18, b must equal 2, 5, or 11, and c must equal 2 or
7. Thus we must have b = 11 and c = 7. Then a = 13 and all equations are satisfied.
39. 9. [13,7] Note that f (n) = (a + 1)(b + 1)(c + 1) − 1. Thus we must have (a + 1)(b +
1)(c + 1) = 100a + 10b + c + 1. This simplifies to a(b + 1)(c + 1) + b(c + 1) = 100a + 10b.
Since b, c ≤ 9, we have (b + 1)(c + 1) ≤ 100 and c + 1 ≤ 10, and so equality is obtained
if and only if b = c = 9. However, a can equal 1, . . . , 9.
40. 7/15. [5,5] We use parentheses to denote area of a figure,
√ and refer to the diagram
1
below. We have (HDF ) = 2 HD · DF sin(45) = HD/(2 2), and similarly (AHD) =
√ √
HD/ 2. Since (HDF ) + (AHD) √ = 1, we obtain HD =
√ 2 2/3 and (HDF ) = 1/3.
√ If
α = 6 F AD, then sin(α)
√ = 1/ 5 and sin(90 − α) = 2/ 5. Then (AGE) = AG/(2
√ 5)
and (AGB) = 2AG/ 5. Since (AGE) + (AGB) = 1, we obtain AG = 2 5/5 and
(AGE) = 1/5. Finally (DEGH) = (ADF ) − (HDF ) − (AGE) = 1 − 31 − 15 .

A........................................................E
.......................................................D
... ...α
........ ...
. .. .
..... ..
... ....... .... ..... ...
........... .... ....
....G
... . .... ..
..
..
... ....... ..... ..
... .. ... ..... ..... ............. .
... ..
... .
.......... ....
... .. .... .............. .
... ....
. .
... H
..... ....... .....
.. ......
...
...
...
.
..
...
..
.....
.
....
.
.. ..
... F
... ..
. ..
...... ..
..
..
. ....
... . .. ...
... .... ......... ..
... .... ....... ..
... .. .... . .. ...
. . . . ..
.................. ..
..
.............................................................................................
.
B C
Alternatively, one can write equations of the three diagonal lines, and use these to
find the coordinates of the points G and H, obtaining that the altitudes from G to
AE and from H to DF are 2/5 and 2/3, respectively. Thus the areas of triangles
AGE and HDF are 1/5 and 1/3, respectively.

8
æ
1 1
1. Simplify 2 + 3 + 14 .
2. A quart of liquid contains 10% alcohol, and another 3-quart bottle full of liquid con-
tains 30% alcohol. They are mixed together. What is the percentage of alcohol in the
mixture?
3. You run the first mile at 10 miles per hour, and the second mile at 8 miles per hour.
How many miles per hour is your average speed for the two-mile run?
4. What is the circumference of a circle inside of which is inscribed a triangle with side
lengths 3, 4, and 5? (Note that the triangle is inside the circle.)
5. If all people eat the same amount of pizza, and a pizza 12 inches in diameter serves
two people, how many inches in diameter should each of two pizzas be in order to
serve three people? (Pizzas are circular and are eaten entirely.)
6. How many integers between 90 and 100 are prime?
3x+1
7. If f (x) = 2x−1 , then write a formula in simplified form for f (a + 2). There should be
no parentheses in your formula.
8. Circle B passes through the center of circle A and is tangent to it. Circle C passes
through the center of circle B and is tangent to it. What fraction of the area of circle
A lies inside circle B but outside circle C?
9. How many integers between 1 and 99, inclusive, are not divisible by either 3 or 7?
(This means that they are not divisible by 3 and also they are not divisible by 7.)
10. The number 1234567891011 · · · 585960, which consists of the first 60 positive integers
written in order to form a single number with 111 digits, is modified by removing 100
of its digits. What is the smallest number which can be obtained in this way? (If the
number begins with some 0-digits, you may write it either with or without the 0’s.
Credit will be given for both answers.)
11. Point P is inside regular octagon ABCDEF GH so that triangle ABP is equilateral.
How many degrees are in angle AP C?
12. What 5-digit number 32a1b is divisible by 156? (Here a and b represent digits.)
13. Mary paid $480 to purchase a certain number of items, but the nice vendor gave her
two extra. This decreased the price per item by $1. How many items did she receive
(including the two extra)?
14. There is only one positive integer n for which the number obtained by removing the
last three digits of n exactly equals the cube root of n. What is this integer n?
15. In how many ways can a group of 16 people be divided into eight pairs? (You may
write your answer as a product of integers without multiplying out. You may also use
symbols such as factorials or exponentials.)

1
16. Let S denote the set of positive integers ≤ 18. Thus S = {1, 2, . . . , 18}. How many
subsets of S have sum greater than 85? (You may use symbols such as exponentials
or factorials in your answer.)
17. What is the remainder when 32009 is divided by 21?
18. The roots of x2 + bx + c are the squares of the roots of x2 + dx + e. Express b in terms
of d and e.
19. A rhombus has a 60 degree angle. What is the ratio of its area to that of a circle
inscribed inside it? Write your answer as a fraction.
20. What is the largest positive integer which leaves the same remainder when divided
into each of 99, 141, and 204?
21. Deck A is a standard deck of cards from which the four aces have been removed. Deck
B is a standard deck from which one ace, one king, one queen, and one jack have been
removed. You randomly pick one of these decks and select two cards. They turn out
to be a pair (i.e., both have the same rank; e.g., two 9’s or two kings, etc.). What is
the probability that the deck which you selected was Deck A?
22. You walk up 12 steps, going up either 1 or 2 levels with each stride. There is a snake
on the 8th step, so you cannot step there. How many ways can you go up? For
example, one way is 1-1-1-1-1-1-1-2-1-1-1, while another is 2-2-2-1-2-2-1.
23. Let ` and `0 be parallel lines. Four points, a, b, c, and d, on ` and five points, A, B,
C, D, and E, on `0 are selected, and segments are drawn from each of a, b, c, and d
to each of A, B, C, D, and E in such a way that no three segments meet in the same
point. (We are not considering the intersections on ` and `0 .) How many points of
intersection of these segments are there?
24. In triangle ABC, angle A is 120 degrees, BC + AB = 21, and BC + AC = 20. What
is the length of BC?
25. There are three 3-digit numbers with distinct digits, all digits being nonzero, which
have the property that the number equals the sum of the six 2-digit numbers which
can be formed from its digits (in any order). What is the second largest of these
3-digit numbers?
26. How many solutions x > 2/5 does the equation logx (5x − 2) = 3 have?
27. From a rectangular solid, 300 by 500 by 700 , are removed tunnels whose cross sections
are 100 by 100 squares connecting the centers of each pair of opposite sides. The sides
of these tunnels are parallel to the sides of the solid, and the centers of the tunnels are
at the centers of the rectangular faces. How many square inches is the surface area of
the remaining figure (including its tunnels)?

28. In triangle ABC, BC = 2, CA = 6, and angle ACB = 135 degrees. If C 0 is the
midpoint of AB, what is the length of CC 0 ?
29. List all integers x which satisfy: there exists an integer y > x with x2 + y 2 = xy + 13
and x + y = 2xy − 17?

2
1 1 1 1
30. Evaluate 1·2·3 + 2·3·4 + 3·4·5 + ··· + 98·99·100 as a simple fraction.
31. What is the maximum√number√ of points with integer coordinates which can lie on a
circle with center at ( 2, 3)?
32. Polygon ABCDEF has AB = BC = CD = DE = 2 and EF = F A = 1. Its interior
angle at C is between 90 and 180 degrees, its interior angle at F is greater than 180
degrees, while the rest of its angles are right angles. What is its area?
33. In the polygon ABCDEF of the preceding problem, what is the length of the line
segment drawn from A to E?
34. What√ is the largest
√ positive integer y such that there exists a positive integer x satis-

fying x + y = 525?
p p
35. If 0 ≤ θ ≤ π, and sin( θ2 ) = 1 + sin(θ) − 1 − sin(θ), what are all possible values of
tan(θ)?
36. Let P be the point (2, 10) and Q = (14, −5). What is the maximum value of the
difference P X − QX for all points X on the x-axis? Here P X and QX denote
distance between points.
37. In triangle ABC, B 0 lies on AC with AB 0 /B 0 C = 5/2, and A0 lies on CB with
CA0 /A0 B = 4/5. Let P be the point of intersection of the segments BB 0 and AA0 ,
and let C 0 be the point where the extension of CP meets AB. What is the ratio
CP/P C 0 ?

38. What integer equals 2000 · 2008 · 2009 · 2017 + 1296?
39. Suppose ABCD is a quadrilateral with integer sides, right angles at A and C, and
diagonal BD = 25. List all integer values for the distance AC.
40. What is the sum of the real solutions of x6 − 14x4 − 40x3 − 14x2 + 1 = 0?

3
Solutions to 2009 contest
The numbers in square brackets are the number of people (out of 209) who answered
the question correctly, followed by the number of correct responses from the top 31 people
(those with scores of ≥ 18).
1. 13/12 or 1 1/12. [200,31]It is (6 + 4 + 3)/12.
2. 25 or 25%. [164,30] In 4 quarts, you have .1 + 3 × .3 = 1 quart of alcohol.
1 1 18
3. 80/9 or 8 8/9. [96,30] Your total time in hours is 10 + 8 = 80 . Your speed is
2/ 18
80 = 80/9.

4. 5π. [137,27] Since the triangle is a right triangle, its hypotenuse will be the diameter
of the circle.

5. 6 3. [82,24] Two people eat 36π sq in, so three people will √ need 54π
√ sq in. If this is
in two pizzas, each must have area 27π, and hence radius 27 = 3 3.
6. 1. [142,28] Only 97 is prime. Note that 91 = 7 · 13, and 93 and 99 are divisible by 3.
3a+7 3(a+2)+1
7. 2a+3 . [159,30] It is 2(a+2)−1 .

8. 3/16. [124,31] The diameter of circle B is half that of circle A, and hence its area is
1/4 of that of the larger circle. Similarly, the area of circle C is 1/4 of that of circle
B. The answer is 41 − 14 14 = 16
3
.
9. 56. [92,26] There are 33 which are divisible by 3, and there are 14 that are divisible
by 7. But there are 4 which are divisible by both. Thus there are 47 − 4 = 43 which
are divisible by at least one of 3 and 7, and hence 99 − 43 = 56 which are divisible by
neither.
10. 00000123450. [65,17] The long number has six 0’s. We should use all of them, but
the last one must come at the end. The other digits are the smallest digits remaining
which come after the previous selected digit.
11. 112.5. [55,26] The relevant portion of the octagon is pictured below.

A..............................................B
.........
.. ....
... .. ........
... ... .....
...
... ....
. .....
.....
... .. .....
... .... .....
... .... ..
... ..
... ... ......................... . .
.. ...... C
...........
............
P

Angle ABC = 135, so angle P BC = 135 − 60 = 75. Since BP = AB = BC, angle


BP C = (180 − 75)/2 = 52.5, so angle AP C = 60 + 52.5.
12. 32916. [139,29] Our number must be divisible by 3, 4, and 13. Consideration of
divisibility by 3 and 4 implies that the number is one of 32112, 32412, 32712, 32016,
32316, 32616, or 32916. Mod 13, we have 32112 ≡ 2 and 32016 ≡ 10. Since 300 ≡ 1,
the seven numbers listed above are congruent to 2, 3, 4, 10, 11, 12, 0, respectively.

4
Another way is to note that 206 · 156 = 32136 is the smallest multiple of 156 exceeding
32000. Keep adding 156 until you get to a number whose 10’s digit is 1.
480 480
13. 32. [118,31] If x is the number of items that she received, then x−2 − x = 1. This

simplifies to 960 = x(x − 2), yielding x = 1 + 961 = 32.
14. 32768. [53,20] The integer n = 1000x + y = x3 with 0 ≤ y < 1000. We must have
1000x ≤ x3 and hence 1000 ≤ x2 . The smallest such integer is x = 32. The desired
n is 323 = 32768. Since we must also have x3 < 1000x + 1000 ≤ 1000x + x2 , then
x2 < 1000 + x, which implies x < 33.
15. 15 · 13 · 11 · 9 · 7 · 5 · 3 or 2027025 or 16!/(28 8!). [28,13] Arbitrarily order the people.
There are 15 ways to choose the partner of #1. Now there are 13 ways to choose the
partner of the person with the lowest number of the 14 remaining people. Continue
in this fashion, or argue by induction.
16. 217 or 131072. [8,7] The sum of all the numbers from 1 to 18 is 18·19/2 = 171. Exactly
half of the subsets have sum greater than 85 because the complements of these subsets
are exactly those with sum ≤ 85. Since there are 218 subsets altogether, the answer
is 217 . (You should realize that the whole set and the empty set are both considered
as subsets.)
17. 12. [40,18] Working mod 21, the powers 3n for n = 1, . . . , 7 are 3, 9, 6, 18, 12, 15, 3.
Then it starts repeating with period 6. Since 2009 = 334 · 6 + 5, 32009 ≡ 35 ≡ 12 mod
21.
18. b = −d2 + 2e or just −d2 + 2e. [25,17] If x2 + dx + e = (x − α)(x − β), then
x2 +bx+c = (x−α2 )(x−β 2 ), and so b = −(α2 +β 2 ) = −(α+β)2 +2αβ = −(−d)2 +2e.

8 3
19. . [38,22] Let the side length of the rhombus equal 1. Then its altitude equals
√3π
3/2, and hence this also equals its area, and equals the diameter of the inscribed
circle, which hence has area 3π/16. See diagram below.
.........................................................................
............... .............. ...
........ .....
...
..... ....
..
... ... ....
.
. ... .
..... ... .....
...... ... ...
..... .... .. ...
.
. . .....
... .... .....
... ...
...
.
.... ...
..... ..
.
... ..
...... .......
... ... .........
........................................................................................

20. 21. [42,20] If x is the number, then both 141 − 99 and 204 − 141 must be multiples of
x. Thus x must be the greatest common divisor of 42 and 63. Hence x = 21.
¡¢ ¡¢ ¡¢
21. 12/23. [32,19] Deck A has 12 42 = 72 pairs, while Deck B has 4 32 + 9 42 = 66 pairs.
The desired probability is 72/(72 + 66). A more thorough solution can be given using
conditional probability. The answer depends on the fact that both decks have the
same number of cards (48). If this were not the case, the answer would be much more
complicated.
22. 63. [29,15] You must do a 2-step from 7 to 9. Thus our desired answer equals the
number of ways to do 7 steps times the number of ways to do 3 steps. The latter is 3

5
¡¢ ¡¢ ¡¢
(viz., 1-1-1, 2-1, and 1-2), while the former is 1 + 61 + 52 + 43 = 7 + 10 + 4 = 21.
These latter possibilities are determined by how many 2-steps were taken among the
7. For example, if you used ¡ ¢ two 2-steps in getting to 7, then you took five strides
altogether, and there are 52 places where the two 2-steps could have been done.
23. 60. [36,18] For any ordered pair (x, y) from {a, b, c, d} with x to the left of y and
any ordered pair (X, Y ) from {A, B, C, D, E} with X to the left of Y , there will be
one
¡4¢¡5intersection,
¢ of the segment xY with the segment yX. Thus there are exactly
2 2 = 6 · 10 points of intersection.

24. 13. [41,19] Let b = AC. Then, by the Law of Cosines, (20 − b)2 = b2 + (b + 1)2 −
2b(b + 1) cos(120), hence 400 − 40b + b2 = 2b2 + 2b + 1 + b(b + 1). We obtain 0 =
2b2 + 43b − 399 = (b − 7)(2b + 57). Thus b = 7 and BC = 20 − b = 13.
25. 264. [40,21] If the number equals 100a + 10b + c, then this must equal (10a + b) +
(10a + c) + (10b + a) + (10b + c) + (10c + a) + (10c + b) = 22(a + b + c), and hence
78a = 12b + 21c. We must have c = 2c0 with 1 ≤ c0 ≤ 4 and, dividing by 6, we obtain
13a = 2b + 7c0 . The RHS is ≤ 46, and so we must have a = 1, 2, or 3. One now easily
checks that the only possible numbers are 132, 264, and 396.
26. 2. [71,24] The equation is equivalent to f (x) = 0 where f (x) = x3 − 5x + 2. Since
f (−100) < 0, f ( 25 ) > 0, f (1) < 0, and f (100) > 0, f has one root x satisfying x < 2/5,
one satisfying 2/5 < x < 1, and one satisfying x > 1.
27. 184 or 184 sq in. [37,20] On the faces, there is 2(3 · 5 + 3 · 7 + 5 · 7) − 6 = 136.
The 1-by-1-by-1 cube at the very center gives no surface area. Leading into it are
six tunnels of cross-sectional perimeter 4 and lengths 1, 1, 2, 2, 3, and 3, yielding an
additional surface area of 48.
√ √ √ √
28. 12 26. [18,13] Place B at (0, 0), C at ( 2, 0) and A at (4 2, 3 2). Then C 0 is at
√ 3√ q√ √ √ q √
(2 2, 2 2), and so the desired length is ( 2) + ( 2 2) = 2 1 + 49 = 26/2.
2 3 2

A.
......
........
..
..
................
.........
............
...... .....
...................
.... .. .
0 ............. ............
C ..
..
.
........ .........
....... ....
...... .. .....
...... ... .....
........... ..... .........
..... ............
...... .. ....
...... .........
.....
..
.... ........
....................................
B C

29. 3. [26,11] Let u = x + y and v = xy. Then u2 = 3v + 13 and u = 2v − 17, which


yields 2u2 − 3u − 77 = 0 and hence u = 7 or −11/2. The corresponding values
of v are 12 and 23/4, respectively. If (u, v) = (7, 12), then {x, y} = {3, 4}, while
(u, v) = (−11/2, 23/4) does not give integer values for x or y.
1 1 1
30. 4949/19800. [7,6] Note that (n+1)n(n−1) = 2n(n−1) − 2(n+1)n . Thus the desired sum
is 4 − 2(100)(99) , as all intermediate terms cancel out. This equals 50·99−1
1 1
200·99 .

6
31. 1. [19,9] For any
√ √ pair (m, n) of integers, there is clearly a circle passing through (m, n)
centered at ( 2,√ 3). Suppose √ (m 1 , n1 ) and (m2 , n2 ) are
√ distinct2points
√ which lie2 on
2 2 2 2
the
√ circle (x −√ 2) + (y − 3) = r . Then m1 √ − 2 2m1 + n1 − 2 3n1 √ = m2 −
2 2m2 +n22 −2 3n2 . Hence (m1 −m2 )(m1 +m2 −2 2)+(n1 −n2 )(n1 +n2 −2 3) = 0.
If √
m1 , m2 , n1 ,√and n2 are integers, then this gives integers A, B, and C such that
√ 2 = B + C 3, and at least one of A and C is nonzero. If A = 0, we obtain that
A
3 is rational, which is false. Similarly we cannot have C = 0. If both are nonzero,
then
√ squaring both sides gives the impossible conclusion that a nonzero integer times
3 equals an integer. Thus there cannot be two integer points on this circle.
32. 6. [33,15] In the figure below, rectangles ABGF and DEF H have area 2 each, while
triangles CF H and CF G have areas 1 each.

D ......
... ..........
.. ......
..... ......
.. ......
. ...... ......
.
.........
H
..
. . .... ..........
.. .. . ......
... .. ......
.. .. ......
... ... .....
... ..
.
.
........
.
... ..
.........
....... .... C
.. . ... ...
.....
. ...
E ....... ... .......
...... ... ....... ...
...... ... ............
. ..
..
......
...... ... .......... ...
...... .................
.........................................................................................
F
..
....
...
...
G
.. ..
... ..
. ...
.... ..
.. ..
... .
.....................................................................................
A B

33. .8 5. [8,7] The tangent of angle GF C is 1/2. Angle GF H is twice as large, so its
1 4
tangent is 1 = 3 . Therefore the angle which F E makes with a horizontal to the
1− 4
left from F has tangent equal to 3/4. If the figure isp placed so that F =√(0, 0), then
4 3
E = (− 5 , 5 ) and A = (0, −1), so the distance AE is (.8)2 + (1.6)2 = .8 1 + 4.

34. 336. [32,14] We have y = 525 + x − 10 21x; hence x = 21m2 for some positive integer
m. Similarly y = 21n2 for some positive integer n. We obtain m + n = 5, and hence
(m, n) = (1, 4), (2, 3), (3, 2), or (4, 1). The largest y is 21 · 42 .
p
35. 0 and −4/3. [3,3] Squaring yields sin2 ( θ2 ) = 2 − 2 1 − sin2 θ = 2 − 2| cos θ|. Hence
1 − cos θ = 4(1 − | cos θ|). If cos θ ≥ 0, we obtain 3 cos θ = 3, so cos θ = 1 and hence
tan(θ) = 0. If cos θ < 0, then cos θ = −3/5 and so tan θ = −4/3.
36. 13. [5,1] Note that QX = Q0 X, where Q0 = (14, 5). For any X, P X ≤ P Q0 + Q0 X,
hence P X − Q0 X ≤ P Q0 = 13. Equality will be attained when P , Q0 , and X lie in
a straight line, which occurs when X is the point of intersection of the extension of
P Q0 with the x-axis.

7
37. 6/5 or 1.2. [2,0] Refer to the drawing below.

C............
... ........
.. ..........
... ........
... .... ........
... ....
... ... ......
.. ... .....
... ... .....
0.. .
B ..........
.. .......
... .....
... .....
... ..
.. ...... ... ........
... ..... .. .....
..... .... .....
..... 0
.. .....
.
....
..
..... ...
..... ..
..... ... A
..... .. ..............
... ........ .. .....
....... .. .....
. ........
..
.... P .
.
. .. .....
.
... ..............
.....
.....
.....
.. .. . ... ..... .....
.. . . ... ..... . .....
... . .
. ... ...... . ..
.. .... ... ..... ........
... . . ... . . ..... ......
.. . .. ... ...... .....
.. .. . ... ..... ....
. .. ..... ....
... .. ... ..... ....
... ..... ... ..... ....
..... .....
... .. ... ..... ....
. ...
.. ..... ... ..... ....
.........
... ... ... .........
..... ... ....
.......................................................................................................................................................
A C0 B

One slick way to solve this uses what is called Mass Point Geometry. If you have not
heard of this, you can learn about it by googling the term. If you put a mass 5 at C,
2 at A, and 4 at B, then the center of mass of {A, C} is at B 0 , and it is as if there was
mass 7 there. The center of mass of {A, B, C} will lie on BB 0 . Similarly the center of
mass of {B, C} lies at A0 , since the products of mass times distance from A0 are the
same for B and C. Thus the center of mass of {A, B, C} also lies on AA0 , and hence
is at P . This P must also be the center of mass of CC 0 , with mass 6 at C 0 . Hence
the ratio CP/P C 0 is 6/5. The problem can be worked in other ways. One combines
Ceva’s theorem and Menelaus’s theorem. Another uses vectors. Append directions to
all the segments in the diagram, and write vector equations for each triangle. Insert
the given information: for example, if you let v = AB 0 , then B 0 C = 52 v. You can
find the desired ratio by solving the six equations. Another method uses equations
involving areas of subtriangles.
38. 4034036. [8,6] Let A = 2000 and B = A2 + 17A + 36. The expression inside the
square root is A(A + 17)(A + 8)(A + 9) + 1296 = (A2 + 17A)(A2 + 17A + 72) + 1296 =
(B − 36)(B + 36) + 1296 = B 2 . Thus the answer is B = 20002 + 34000 + 36.
39. 20, 24, 25. (any order) [5,4] The sets {AB, AD} and {BC, CD} can each be either
of {7, 24} or {15, 20}. BD is the diameter of a circle passing through all four points,
so the Theorem of Ptolemy applies to say that 25AC = AB · CD + AD · BC. The
possibilities for the RHS are 7 · 7 + 24 · 24, 7 · 24 + 24 · 7, 7 · 15 + 24 · 20, 7 · 20 + 24 · 15,
15 · 15 + 20 · 20, and 15 · 20 + 20 · 15. These sums are 625, 336, 585, 500, 625, and 600,
respectively. The integer ratios when these are divided by 25 are as claimed.
40. 5. [1,1] Let y = x + x1 , and note y 3 = x3 + x13 + 3y. After dividing the given equation
by x3 , it becomes y 3 − 17y − 40 = 0 or (y − 5)(y 2 + 5y + 8) = 0, which has y = 5 as
its only real root. Thus x + x1 = 5, so x2 − 5x + 1 = 0, which has real roots whose
sum is 5.

8
æ
1 1
1. What simple reduced fraction equals 3 + 4 + 16 ?
2. The legs of a right triangle have ratio 5:12. The hypotenuse has length 39. What is
the length of the shortest leg of the triangle?
(4x−1)(x−2)
3. What values of x satisfy x+1 = 0?
4. The population increases by 20% one year and then decreases by 20% the next year.
What is its total change, in percent, over the 2-year period?
5. How many 2-digit numbers have the sum of their digits divisible by 6? (Note, 6 is not
considered a 2-digit number.)
6. What is the sum of the areas of the two triangles in the diagram below, in which
AB = 3, CD = 6, and the distance between the parallel lines AB and CD is 6?

C..........................................................................D
...
... .....
... ....
... ..
.....
... ......
... ....
........
...
...
..... .....
.... ...
.... ...
.
.............................
.
A B

7. If f (x) = x2 + 3 and g(x) = 2x − 1, express f (g(x)) as an explicit function of x, using


no parentheses in your answer.
8. Determine the number of pairs (m, n) of positive integers for which

m3 + n = 1000000008 = 109 + 8.

9. The streets in a city form a grid, with some running north-south, and others east-
west. Alice, Bob, Carol, and Don live at four street intersections which are vertices of
a rectangle, with Alice and Don at opposite vertices. The school that they all attend
is at an intersection inside the rectangle, and they all walk to it by their shortest
possible route. Alice walks 10 blocks, Bob walks 12 blocks, and Carol walks 15 blocks.
How many blocks does Don walk to get to school?
10. What is the number of noncongruent rectangles with integer sides and area 315?
11. A rectangular sheet of paper is folded in half, with the crease parallel to the shorter
edge. It happens that the ratio of longer side to shorter side remains unchanged.
What is this ratio?
12. How many lines in the plane are at distance 2 from the point (0, 0) and also at distance
3 from the point (6, 0)?
13. What is the remainder when x135 + x125 − x115 + x5 + 1 is divided by x3 − x?
14. A “turn” is a flip of a pair of fair coins. What is the probability that, in 4 turns, you
obtain a pair of heads at least once?

1
15. For positive integers a, b, and c, what is the smallest possible value of a + b + c for
which abc = 7(a + b + c)?
16. The exterior angles of a triangle are in the ratio 3 : 5 : 6. What is the ratio, similarly
expressed and reduced to lowest terms, of the interior angles adjacent to them, in the
same order?
17. All positive integers which can be written as a sum of one or more distinct powers of
3 are written in increasing order. The first four of them are 1, 3, 4, and 9. What is
the 81st?
18. Triangle ABC has AB = 25, AC = 24, and BC = 23. Let D be the point on AC for
which BD ⊥ AC. What is the difference AD − DC?
√ √
19. List all solutions in the domain 0 ≤ x ≤ 2 of the equation 2 − x = 1 + x.
20. Two 3-by-9 rectangles are arranged as below, so that an opposite pair of vertices of
each coincide. What is the area of the parallelogram which forms their intersection?
..
.........
...... .....
.
.........
. ...
.... ...
..... ...
.
..
........ ...
...
.....
. .
.........................................................................................................................................
.
... .
. .
.
... ......
. ...
.
..
... ....
. .
..
... ...... ...... ....
... ...... ......
...... ...... ..
...
... .
......... ...
....... ...
..
.
....
. .....
. ..
... ...... .
.....
.......... .. .
...................................................................................................................................
... .. .
...
... ..
.
... .....
... ......
... ......
...
...........
... .....
.........
..

21. Three boys, A, B, and C, can work separately or together on a job. If working
together, their efforts combine efficiently. It takes A working alone twice as long as it
takes B working alone. It takes A four times as long as B and C working together
to do the job. Also, it takes A two hours longer to do the job than it takes all three
boys working together. How many hours does it take B to do the job?
22. We call a number ascending if each digit is greater than the digit that precedes it. For
example, 457 is ascending, but 447 is not. How many ascending numbers are there
between 400 and 5000?
√ √
23. Out of all polynomials with integer coefficients which have both 12 2 and 12 3 as
roots, consider those of smallest possible degree. Among all of these, what is the
smallest positive coefficient which occurs in any of them?
24. A triangle has vertices at (0, 0), (4, 2), and (5, 1). What is the tangent of its angle at
the vertex (4, 2)?
25. What is the smallest positive integer n such that the product 19999 · n ends in the
four digits 2010?
26. A parallelogram has area 36 and diagonals of length 10 and 12. What is the length
of its longest side?

2
27. The numbers 1 to 9 are arranged in tables with 3 rows and 3 columns in all the 9!
possible ways. For each table, we take the smallest row-sum and the largest row-sum.
We do this for all the tables, and add all the 2 · 9! numbers obtained in this way. What
is the value of this sum? You may use factorials in your answer.
28. There are six doors in front of you. Behind each of five of them is $120, and behind
the sixth is a goat. You may open as many doors as you like. If all the doors you open
contain money, then you keep all the money. But if the goat is behind one of the doors
that you open, it eats all the money, and you get nothing. How many doors should
you open to maximize the expected value of your winnings, and what is the expected
value? Your answer should be the ordered pair (n, m), where n is the number of doors
you open, and m the expected number of dollars that you win.
29. List all integer values of x for which x2 − 5x − 1 is a perfect square. (Note that x may
be positive or negative.)
n
30. For what positive integer n does the decimal expansion of 810 equal a repeating decimal
.9d59d5 · · · with 3-digit repetend 9d5, where d is some digit from 0 to 9?
31. For a positive integer n, let p(n) denote the (base 10) product of the digits of the
base-5 expansion of n. For example, p(24) = 4 · 4 = 16 since the base-5 expansion of
P
124
24 is 44. Evaluate p(n).
n=1

32. Out of four integers, which are not necessarily distinct, we form the sums of each of
the six pairs, and find that the values of these sums are 27, 41, 44, 47, and 61. Thus
one of these sums is achieved twice. What is the largest of the four integers?
33. How many distinct integers occur in the list
· 2 ¸ · 2 ¸ · 2 ¸ · ¸
1 2 3 20102
, , ,... , ,
2010 2010 2010 2010

where [x] denotes the greatest integer less than or equal to x?


34. 229 is a 9-digit integer with distinct digits. What digit (from 0 to 9) does it not
contain?
p3
√ p3

35. What rational number equals 9 + 4 5 + 9 − 4 5?
36. An army marches at a steady pace. When the front of the army passes a suspected
bomb, a runner runs at a steady pace from the front of the army to the rear, and then
turns around and runs back to the front at the same steady pace, arriving back at the
front just as the rear passes the suspected bomb. What is the ratio of the speed of
the runner to the speed of the army?
37. Let ABC be an isosceles right triangle with right angle at
√ C. Let P be a point inside
the triangle such that AP = 3, BP = 5, and CP = 2 2. What is the area of the
triangle ABC?
p
38. What reduced fraction q with 8 ≤ q ≤ 99 is closest to 37 ?

3
39. Write all positive integers which occur as max{a, b, c, x, y, z}, where a, b, c, x, y, and
z are positive integers satisfying abc = x + y + z and xyz = a + b + c. A correct answer
to this question will consist of four integers. Half credit will be granted for an answer
consisting exactly of three of the four.
40. Let EF be the diameter of a semicircle, A a point on this diameter, and C, D, and
B points on the semicircle such that AD ⊥ EF , AD bisects angle CAB, BE bisects
angle ABC, and CF bisects angle ACB. If AB = 6 and AC = 3, what is AD?

4
SOLUTIONS: In brackets is the number of people who answered it correctly out of
the 54 people who got at least 20 right.
4+3+2
1. 3/4. [53] It is 12 .
p
2. 15. [54] If the legs are 5a and 12a, then the hypotenuse is (5a)2 + (12a)2 = 13a.
Hence a = 3.
3. 1/4 and 2. [53] The fraction is not defined when x = −1.
4. 4% decrease or −4% or just −4. (We eventually gave credit for an answer of 4 or 4%,
too, because so many high-scoring students wrote that as their answer. My feeling
is that “change” is a signed quantity, but since so many smart people interpreted it
differently, credit was granted.) [51] If the initial population is P , then after 1 year it
is 1.2P , and after 2 years it is .8 · 1.2P = .96P , 4 percent less than the initial.
5. 14. [44] The sum of the digits can be 6, 12, or 18. The numbers are 15, 24, 33, 42,
51, and 60; then 39, 48, 57, 66, 75, 84, and 93; finally 99.
6. 15. [53] The triangles are similar. Thus their altitudes are 2 and 4, and the sum of
the areas is 12 (3 · 2 + 6 · 4).
7. 4x2 − 4x + 4. [52] It is (2x − 1)2 + 3.
8. 1000. [45] m can be any integer from 1 to 1000. Then n = 109 + 8 − m3 .
9. 17. [51] The total distance walked by Alice and Don equals that total walked by Bob
and Carol, and so Don walks 12 + 15 − 10 blocks.

10. 6. [43] It is the number of factors of 315 = 32 · 5 · 7 which are less than 315, and this
is between 17 and 18. These factors are 1, 3, 5, 7, 32 , and 3 · 5, which give the shorter
side of the rectangle.

11. 2. [51] Letting ` and s refer to the length of the longer and shorter sides of the
initial rectangle, we have s` = `/2
s
. Hence 2s2 = `2 .

12. 4. [36] The circles of radius 2 centered at (0,0) and of radius 3 centered at (6,0) do not
intersect. The desired lines must be tangent to each. There are two common internal
tangents and two common external tangents.
13. 2x + 1. [30] Write x135 + x125 − x115 + x5 + 1 = q(x)(x3 − x) + ax2 + bx + c. Letting
x = 0, 1, and −1 yields 1 = c, 3 = a + b + c, and −1 = a − b + c. These are easily
solved to give c = 1, b = 2, and a = 0.
175
14. 256 .[43] The desired answer is 1 minus the probability of never getting a HH on four
turns. This latter probability is (3/4)4 = 81/256.
15. 15. [43] One of the integers must be divisible by 7. Call this one a. For the smallest
possible value, we should try a = 7. Then we have bc = 7 + b + c, which yields
8 = (b − 1)(c − 1). Then {b, c} = {3, 5} or {2, 9}. Thus {a, b, c} = {7, 3, 5} yields the
smallest sum.

5
16. 4 : 2 : 1. [45] The sum of the exterior angles is 360. Hence the three exterior angles
are 540 900
7 , 7 , and
1080
7 . The interior angles are
1260
7 minus these, and hence are 720
7 ,
360 180
7 , and 7 , which are clearly in the ratio 4 : 2 : 1.
17. 811. [34] These are the positive numbers whose base-3 expansion consists entirely of
0’s and 1’s. The order of these numbers will be the same as that of the corresponding
base-2 numbers. Since 81 = 26 +24 +20 , our desired number is 36 +34 +30 = 729+81+1.
18. 4. [43] Let x = AD and h = BD. There are right triangles with 252 = x2 + h2 and
232 = (24−x)2 +h2 . Subtracting yields 252 −232 = 48x−242 . Thus (25−23)(25+23) =
48(x − 12) and so x = 14. The desired difference is 14 − 10.

19. 1 − 12 3. [20] Sketching the graphs suggests that there should be exactly one solution.
√ √ √
Squaring yields 0 = 2x + 2 x − 1. The quadratic formula gives x = 12 (−1 + 3),

hence x = 1 − 12 3.
20. 15. [45] In the diagram below, the Pythagorean theorem implies x2 = 32 + (9 − x)2 ,
so x = 5. The area of the parallelogram is 3 · 5.
.......
...... .....
...... ...
..
..
....... ...
... ...
.
..
....... ...
...
9−x . .
..
. .
...
x ..
..........................................................................................................................................
... ...
. .. ...... ...
... ...
... ..... ..... ....
... ...... ......
... ...... ...... ..
3 ...
...
x ..
....
. ..
........
.
...
......
......... ...
..
..
... ......... ..
..
.... ...
......... ...
..
...................................................................................................................................
... ...
..
... .....
... ......
... ......
...
... .
..........
... .........
.........
.

21. 5/4 or 1.25. [41] Let fA , fB , and fC denote the fraction of the job that each of the boys
can do in one hour. Then fB = 2fA , fB + fC = 4fA , and 1/(fA + fB + fC ) + 2 = 1/fA .
We obtain 2 = 45 · f1A . Hence fA = 25 so fB = 45 . The desired answer is the reciprocal
of 4/5.
22. 141.
¡6¢ [40] For 3-digit numbers, the digits
¡9¢ can be any three of 4, 5, 6, 7, 8, and 9, thus
3 = 20 such numbers. There ¡5¢
are 4 = 126 4-digit ascending numbers altogether.
From these, we delete the 4 = 5 which start with a digit ≥ 5.
√ √
23. 3. [38] Such polynomials must also have − 12 2 and − 12 3 as roots, and hence be
divisible by (x2 − 12 )(x2 − 43 ). The desired polynomials are all integer multiples of
8(x4 − 54 x2 + 38 ), and so the smallest positive coefficient is 3.

24. −3. [33] The Law of Cosines says that 26 = 20 + 2 − 2 40 cos α, hence cos α = − √110
and so tan α = −3. Alternatively, the sides going out from the vertex in question
make angles with the horizontal having tangents equal to 1/2 and 1. Thus the sum
of these two angles has tangent ( 12 + 1)/(1 − 12 ) = 3, and our desired angle, being 180
minus this sum, has tangent the negative of 3.
25. 7990. [42] The last four digits of a number are its residue mod 10000. Since 19999 ≡ −1
mod 10000, our n will satisfy 2010 ≡ 19999n ≡ −n mod 10000, and hence n =
10000 − 2010 = 7990.

6

26. 109. [23] Let BD be the longer diagonal, A another vertex, and let AM ⊥ BD.
Since the area of ABD is 18, AM = 3.√Since the point O wherepthe diagonals meet
divides each in half, AO = 5 so OM = 52 − 32 = 4 and AD = 32 + (4 + 6)2 .
27. 30 · 9! (or 3 · 10!). [17] To each table, associate the complementary table obtained by
replacing each entry i by 10 − i. The largest row-sum of a table plus the smallest
row-sum of its complementary table is 3 · 10 = 30. Similarly for the smallest of the
first plus the largest of the complementary table. Thus each of the 9! 2 pairs of tables
contributes 60. An easier, but less thorough, method uses that 30 = 23 (1 + · · · + 9) is
the average of the sum of the largest and smallest rows of each of the 9! tables.
28. (3, 180). [40] If you open n doors, your expected winning is 120n(1 − n6 ), since the
probability that the goat is behind one of the selected doors is n/6. For n = 1, 2, 3, 4, 5,
this is, respectively, 65 120 = 100, 23 240 = 160, 12 360 = 180, 13 480 = 160, and 61 600 =
100.
29. −5 and√ 10. (Must have both.) [28] From x2 − 5x − 1 = n2 , we obtain that x =
1 2 2 2
2 (5± 29 + 4n ) and hence 29+4n = t for some integer t. Thus 29 = (t−2n)(t+2n)
and so t − 2n = 1 and t + 2n = 29. (Note that negating n does not change x, and so
reversing the 1 and 29 gives no new information.) We obtain t = 15 and n = 7. Thus
x = 12 (5 ± 15).
n
30. 750. [40] We have 810 = 9d5
999 , hence 37n = 30 · 9d5. Thus 9d5 is divisible by 37,
and trying the division shows it must be that the quotient is 25 and d = 2. Thus
n = 30 · 25.
31. 1110. [32] Note that p(5)+· · ·+p(24), the sum over all numbers whose base-5 expansion
has two digits, is (0 + 1 + 2 + 3 + 4)2 = 100, since both of these give all products of
two of these digits. Similarly the sums over all numbers whose base-5 expansion has
1 (resp. 3) digits is 10 (resp. 1000). Our desired sum is then 10 + 100 + 1000.
32. 32. [40] Let the integers be a ≤ b ≤ c ≤ d. Then a + b = 27 and c + d = 61. Thus
a + b + c + d = 88, and we can see that a + d = 44 and b + c = 44. Now b + d = 47, the
second largest sum. Adding yields a + b + c + 3d = 152 and so 2d = 152 − 88 = 64.
h 2 i
n n2
33. 1508. [9] Let g(n) = 2010 and f (n) = 2010 . Then f (n) − f (n − 1) = 2n−1
2010 is < 1 if
n ≤ 1005, and is > 1 if n > 1005. Thus every nonnegative integer ≤ g(1005) = 502 is
achieved as g(n) for at least one integer value of n, yielding 503 such values, while for
1005 ≤ n ≤ 2010, each g(n) takes on a different value. Thus the answer is 503 + 1005,
with the 1005 being the values of g(n) for 1006 ≤ n ≤ 2010.
34. 4. [38] Using 26 ≡ 1 mod 9, we obtain 229 ≡ 14 · 32 ≡ 5 mod 9. Since 0 + 1 + · · · + 9 =
45 ≡ 0 mod 9, we deduce that the missing digit is 9 − 5. In fact, 229 = 536870912.
p
3
√ p
3
√ √ √
35. 3. [22] Let α = 9 + 4 5 + 9 − 4 5. Since (9 + 4 5)(9 − 4 5) = 1, we obtain
q q
3
√ 3 √ 3 √ √
α = (9 + 4 5) + 3 9+4 5+3 9 − 4 5 + 9 − 4 5 = 18 + 3α.

7
Thus α is a root of x3 − 3x − 18 = 0, of which x = 3 is clearly a root. In fact,
x3 − 3x − 18 = (x − 3)(x2 + 3x + 6) and so 3 is the only real root.

36. 1 + 2. [24] Let L denote the length of the army, f the desired ratio, and αL the
distance the army had moved when the runner reached the rear. Then the runner
ran L − αL while the army moved αL, and the runner ran L + (L − αL) while the
army moved L − αL. Hence 1−α α √= f = 2−α 2
1−α so that 2α − α = 1 − 2α + α and
2
√ √
hence α = 1 − 21 2. Thus f = 2−√ 2
2
= 1 + 2. Alternative solution: Without
loss of generality, assume that the army is 1 mile long and marches at 1 mph. Let r
denote the speed of the runner. During the first part of his run, he is moving at rate
(1 + r) with respect to the army and moves 1 mile with respect to the army, so the
time required for this is 1/(1 + r). During the second part of his run, he is running
at rate (r − 1) with respect to the army and again moves 1 mile with respect to the
1 1
army. Thus his total time is 1+r + r−1 and this equals 1 since the army will have
2

marched for an hour. Thus r − 2r − 1 = 0 and r = 1 + 2.

37. 29/2 = 14.5. [5] Let s be the length of a leg of the triangle, so that its area is
s2 /2. Place the triangle with right angle at the origin, and other vertices at (s, 0) and
(0, s). Let (x, y) be the coordinates of P . Then x2 + y 2 = 8, (s − x)2 + y 2 = 9 and
(s − y)2 + x2 = 25. We obtain s2 − 2sx = 1 and s2 − 2sy = 17. Solve for x and y in
terms of s, and place in the first equation, obtaining (s2 − 1)2 + (s2 − 17)2 = 4s2 · 8.
This simplifies to s4 − 34s2 + 145 = 0, hence s2 = 5 or 29. Only s2 = 29 is consistent
with the given data.
¯ ¯
¯ ¯
38. 41/96. [13] Since ¯ 73 − pq ¯ = |3q−7p|
7q , we try to find q as large as possible but ≤ 99 for
which there is an integer p with 3q − 7p = ±1. We look for multiples of 7 which are
slightly less than 300 and are not divisible by 3. The largest is 287 = 41 · 7, which
41
differs from 3 · 96 by 1. Thus 96 differs from 3/7 by 1/(7 · 96), which is clearly as small
as can be obtained.

39. 3, 6, 7, 8. (any order, but all must be listed). [7] We may assume a ≤ b ≤ c and
x ≤ y ≤ z. We easily see that it is impossible to have a = b = c = 1 or x = y = z = 1.
Therefore z ≥ 2. If a = b = 1, then xyz = 2 + c = 2 + x + y + z ≤ 4z which implies
xy ≤ 4. We obtain the following solutions (x, y, z, c) = (1, 2, 5, 8), (1, 3, 3, 7), and
(2, 2, 2, 6). Now we may assume b ≥ 2 and y ≥ 2. Then

abc = x + y + z ≤ 3z ≤ 23 xyz = 32 (a + b + c) ≤ 3
2 · 3c = 92 c.

Thus ab ≤ 4. Similarly xy ≤ 4. Examination of ten possible ways of selecting a, b, x,


and y with products ≤ 4 yields only the solution (a, b, c, x, y, z) = (1, 2, 3, 1, 2, 3).

40. 3 2. [7] Angles ECF and EBF are right angles. Let α, β, and γ be the half angles of
triangle ABC at A, B, and C, respectively. Then α + β + γ = 90. 6 CAE = 90 − α =
6 BAF . Now 6 ACE = 90 − γ = α + β = 180 − 6 BAF − 6 ABF = 6 AF B. Thus
triangles ACE and AF B are similar, and so AE AB 2
AC = AF . Now AD = AE · AF =

8
AB · AC = 18.
D ...........
....... ..
...............................
......
.........B
....... ... ...................... ........
.
.. .
..... ............................................... .............
.
.
........................ . .
... ...... ... ....
C . .
......
...................... ... ........... .......
...... . ... ...
... ...
......... ......... .................................. ......... ... ...
... ...
..... ..... ....... ... ................ ... ...
. ... . .
. ... .
........ .................. .... ........ ...................... .. .
...... ........ ..... .. ..... ........... ..........
...................................................................................................................................................
E A F

9
æ
1 1
1. If 3 + 4 = x1 , then x =
2. A gold bar is a rectangular solid measuring 2 × 3 × 4. It is melted down, and three
equal cubes are constructed from this gold. What is the length of a side of each cube?
3. What is the area of a triangle whose sides have length 10, 13, and 13?
4. The perimeter of a rectangle is 28. A second rectangle is three times as long as the
first, and twice as wide. The perimeter of the second rectangle is 72. What is the area
of the first rectangle?
5. A lamppost is 20 feet high. How many feet away from the base of the post should a
person who is 5 feet tall stand in order to cast an 8-foot shadow?
6. Workers in an office have a coffee machine and make 1 12 cents profit on each cup sold.
The profits were split at the end of the year, and everyone got $2.50, with a total of
$25 remaining after that. It was suggested that instead everyone should get $3, but
that would have left three people with no money. How many cups of coffee were sold?
7. In a circle whose diameter is 30, a chord is drawn perpendicular to a radius. The
distance from the point where the chord intersects the radius to the outer end of the
radius is 3. What is the length of the chord?
8. What is the smallest number of coins (possible denominations 1, 5, 10, 25, and 50
cents) which will enable one to pay the exact price for any item costing from 1 cent
up to and including one dollar (100 cents)?
9. What is the solution set of the inequality x3 + x2 − 2x ≥ 0?
10. In the diagram below, line v is perpendicular to line h, and the angle at B is 75 degrees.
A light ray has angle of reflection equal to angle of incidence. Our convention is to
consider the angle which is ≤ 90 degrees. A ray has initial angle of incidence with v
of 50 degrees. It follows a path in which it next hits wall h, then s, and then v again.
What will be the number of degrees of its angle of incidence with wall v this (second)
time? ...
.
..
.....
...
.
... .... ...
... ..... ...
... ......... ...
... ....... ...
............ ...
.... ...
.... ...
...
...
... s ...
v ...
...
...
...
...
...
... ...
... ...
... ...
... ...

A h
.......................................................................................
B

11. What is the smallest positive integer p for which there is a positive integer n satisfying
2n+p ≡ 2n mod 100? (This p is the period of the sequence h2n mod 100i, which just
considers the last two digits of the numbers.)
12. A polynomial has remainder 2 when divided by x − 1, and has remainder 1 when
divided by x − 2. What remainder is obtained when this polynomial is divided by
(x − 1)(x − 2)?

1
√ √
13. Write the decimal approximation of 9,000,001 − 9,000,000, rounded to three sig-
nificant figures.
14. What is the largest integer n such that 20! is divisible by 80n ? (Note 20! = 1·2·3 · · · 20.)
15. For what positive integer b does the number whose base-b expansion is 265 equal the
number whose base-9 expansion is 1b1?
16. The cross-sectional area of a tree is a linear function of time. The diameter is 2 feet
in 1930 and 4 feet in 1990. What is its diameter in feet in 2010?
17. A quadrilateral is inscribed inside a circle. If an angle is now inscribed in each of the
four arcs outside the quadrilateral but inside the circle, what is the sum of the number
of degrees in these four angles?
18. In a certain town, there are 5000 bicycles, each of which is assigned a license number
from 1 to 5000. No two bicycles receive the same number. What is the probability
that the number on a randomly selected bicycle will not have any 8’s among its digits?
(You may write your answer as a fraction or a decimal number.)
x
19. Let f (x) = x+3 for x 6= −3. List all values of x for which f (f (x)) = x.
20. What is the coefficient of x6 in the expansion of (2x2 − x1 )6 ?
21. For how many integers x between 1 and 91 inclusive is it true that x2 − 3x + 2 is a
multiple of 91?
22. Two roots of the polynomial 3x3 + αx2 − 5x − 10 are r and −r for some real number
r. What is the value of α?
23. Let A = (sin(.01))2 , B = (sin(.01))3 , C = sin(sin(.01)), and D = sin(sin(sin(.01))),
where the .01 refers to radian measure. Write the letters A, B, C, and D in order of
size, starting with the smallest and ending with the largest of the four.
24. A point P is chosen inside an equilateral triangle of side length 1, and perpendiculars
drawn to the three sides. Let S(P ) denote the sum of the lengths of the three per-
pendiculars. What is the largest possible value for S(P ) out of all points P inside the
triangle?
p p
25. What is the smallest positive integer N such that both N/3 and 3 N/2 are integers?
26. The bottom part of a network of roads is shown below. It extends up to level 99 in a
similar fashion. 2100 people leave point A (level 0). At every intersection, half go in
direction L and half in direction R. How many people will end up at the next-to-left
position at level 99? .. ... ... ... ... ... .
... ... ... .. .. .. .
... .... .... ...... ..... ..... ...
... .... ..... .... ..... .... .... .... ..... ..... ..... ....
... ... ... ... ... ... ... ... ... .. ... ...
...... .
..... ...... . .
..... .
..... .
.....
5 ... . ..
... .......... ......... ......... ......... ....
... ... ... ... ... ... ... ... ... ...
. .. .
...... ...... ...... ... .. ........
... .... . . . ..
... .. .... ..... ..
. ... ... ... .. ... .. ...
........ .... .... .... .... ..... .... ..... .... .......
... ...... ...... ...... ...... ...
... ... .. .. .. .
... ... ...... ..... ... ...
.. ... ..... ..... ..... .... ..... ....
... .. .. .. ... ..
..... . ..... . . .
.....
L ... ..
... .......... .....
... ... ... ...
R
...... ......
1 ...
...
... ....
... ...
....
......
.
A
2
27. In how many ways can the faces of a cube be colored using six colors, if each face is
to be a different color, and two colorings are considered the same when one can be
obtained from the other by rotating the cube?
28. An ordinary sheet of graph paper has lines wherever x or y is an integer. How many
paths are there from (0, 0) to (3, 4) which always move along lines in either the positive
x or y direction?
29. A flagpole is on top of a building which is 40 feet high. From a point 50 feet away
from the building, the flagpole subtends an angle A whose tangent is 1/5. How many
feet tall is the flagpole? . ..
.....
.... ..
.
..... ....
.
....
....
....
...
... flagpole
..
...... ........
..
... ...... ..
..... ..
..... ....
.... ......... ..
.... .
.... ......... ...
....
....
.
. ......
.. .......
A ..
.. building
. .
.. ...... ...
..
......... ..
..
. ..
..
......... ...
..
......... .
....
.....................................................................
..
.

30. The perimeter of a right triangle is 30, and the length of the altitude perpendicular
to the hypotenuse is 5. What is the length of the hypotenuse?
31. A convex polygon with n sides has all angles equal to 150 degrees, with the possible
exception of one angle. List all the possible values of n.
32. Alan and Bill walk at 4 miles per hour, and Chad drives 40 miles per hour. They set
out from the same point in the same direction with Alan walking, and Bill riding with
Chad. After an hour, Bill gets out of the car and starts walking in the same direction
they were going, while Chad turns the car around and drives back to pick up Alan.
When the car gets back to Alan, Alan gets into the car, the car turns around and
drives until it meets Bill. How many miles has Bill traveled when the car meets him?
You may assume that no time is required to turn around or to change passengers.
33. List all prime numbers which are of the form x3 − 11x2 − 107x + 1177 for some integer
x.
34. A Fibo sequence a1 , a2 , . . . is one in which a1 and a2 are positive integers, and an =
an−2 + an−1 for n ≥ 3. If a1 , a2 , . . . , 81, . . . is the Fibo sequence which contains 81
and has the largest possible number of terms preceding the 81, write the ordered pair
(a1 , a2 ).
35. What fourth degree polynomial p has p(0), p(1), p(2), p(3), and p(4) equal to 7, 1, 3,
1, and 7, respectively?
36. Four spheres of radius 1 are placed so that each is tangent to the other three. What
is the radius of the smallest sphere that contains all four spheres?
P
999
37. Evaluate [.729n], where [q] denotes the greatest integer ≤ q.
n=1

3
38. Points A, B, C, and D lie on a circle. AC is a diameter, and 6 CBD = 6 DBA. If
BC = 2 and AB = 4, what is the length of BD? The diagram below is not quite to
scale. .............................
......... .
.. ......
...... .....
..... .....
.... ...
B ..
.
..
.....
......... ........................
.
...
...
...
.. .
. .
. ............. ..
....... ..... .............
............. ....
............................................................................................
C ...
...
...
...
A.
...
... ...
.....
... ... ..
...
.... .....
. ...
...
..... ..
...... ... .......
.
......... ....
.......................................
D

39. How many ways are there to give 18 indistinguishable cookies to Alice, Bob, Carol,
Don, and Elizabeth in such a way that everyone gets at least one, and no one gets
more than 6?
40. List all real values of p for which the two solutions, α and β, of the equation

x2 − 2px + p2 − 2p − 1 = 0

have the property that ((α − β)2 − 2)/(2((α + β)2 + 2)) is an integer.

4
SOLUTIONS, annotated with number of people out of the 46 people who scored at
least 22 answering it correctly.

4 3 7
1. 12/7. [46] 12 + 12 = 12 .

2. 2. [46] The volume of the solid was 24, √ which will be divided into three cubes, each
of volume 8. They will have side length 3 8.

3. 60. [45] The triangle is isosceles, and has a base of 10 and a height of 132 − 52 = 12.
4. 48. [45] L + W = 14 and 3L + 2W = 36. We solve these to find L = 8 and W = 6.
5. 24. [44] If x is the desired distance, then, by similar triangles, 5/20 = 8/(8 + x).
6. 13,000. [36] If N is the number of people, then 52 N + 25 = 3(N − 3), and so N = 68.
Thus the profit was 195, and the number of cups was 195/.015 = 13,000.
7. 18. [45] Let O be the center of the circle, OP the radius which is perpendicular to the
chord, R the point where the chord intersects the radius, and Q one end of the chord.
Then OR = 15 − 3, and ORQ is a right triangle with hypotenuse OQ = 15 and one
leg OR = 12. Its other side, RQ equals 9, and is half the length of the chord.
8. 9. [32] The coins would be 1, 1, 1, 1, 5, 10, 10, 25, 50. The first six coins are required
to get you up to 19 in the most efficient way. You must add another 10 to get to 25,
and then the 25 and 50 cover all other cases most efficiently.
9. [−2, 0] ∪ [1, ∞) or (−2 ≤ x ≤ 0 or x ≥ 1). [44] The inequality is x(x + 2)(x − 1) ≥ 0.
All three or exactly one of the factors must be nonnegative.
10. 80. [41] If it hits h at C, the angle there is 40 degrees. Then it hits s at point D with
angle 180 − 75 − 40 = 65. If it next hits v at point E, then ABDE is a quadrilateral
with three of its angles 90, 75, and (180 − 65). Since a quadrilateral has 360 degrees,
this makes the desired angle 360 − 280 = 80.
11. 20. [40] Start with 1 and keep doubling, but remove all but the last two digits. Keep
doing it until a repeat occurs. 2, 4, 8, 16, 32, 64, 28, 56, 12, 24, 48, 96, 92, 84, 68, 36,
72, 44, 88, 76, 52, 4. Now it keeps repeating. There were 20 steps from one 4 to the
next. We have 2n+20 ≡ 2n mod 100 for n ≥ 2.
12. −x + 3. [32] Write the polynomial as q(x)(x − 1)(x − 2) + ax + b. Then a + b = 2 and
2a + b = 1, leading to a = −1 and b = 3.
√ √
13. .000167. [20] Multiply by S/S, where S = 9,000,001 + 9,000,000, which is very
close to 6000. We obtain 1/S.
14. 4. [43] 80n = 24n 5n , and 20! has exponent of 2 equal to 10 + 5 + 2 + 1 = 18 and
exponent of 5 equal to 4.
15. 7. [45] We have 2b2 + 6b + 5 = 81 + 9b + 1, hence 2b2 − 3b − 77 = 0, which can be
solved either by factoring or by the quadratic formula.

5

16. 2 5. [40] The area grew from π to 4π in 60 years. In the next 20 years it √
will increase
by an additional
√ 3π/3. Thus the area will be 5π, so the radius will be 5, and the
diameter 2 5.
17. 540. [33] If AP B is one of these angles, then the number of degrees in angle P equals
1/2 times the number of degrees in the arc AB which does not include P . All portions
of the circle will be included in three of the four of these arcs. Therefore the answer
is 21 · 3 · 360.
18. .729. [32] The first digit is definitely not an 8, while for each of the other three digits,
the probability that it is not an 8 is 0.9. The answer is .93 .
19. 0, −2. [40] We must have
x/(x + 3)
x = x.
x+3 + 3

This simplifies to x = x(4x + 9), which is true for x = 0 or −2.


¡¢
20. 240. [41] The x6 -term is 64 (2x2 )4 (− x1 )2 , so the coefficient is 24 15.
21. 4. [21] Since x2 − 3x + 2 = (x − 1)(x − 2), both x = 1 and x = 2 make the expression
equal to 0, which is a multiple of 91. In addition to these, we look for multiples of 13
for which adding or subtracting 1 yields a multiple of 7. This works for 13 and 78.
22. 6. [40] We must have 3x3 + αx2 − 5x − 10 = (x2 − r2 )(3x + 10
r 2 ). The second factor is
chosen to make the constant and cubic terms work out. Thus α = 10 2
r 2 and −5 = −3r .
Hence α = 10/(5/3).
23. BADC. [25] (sin(.01))3 ≈ .000001, (sin(.01))2 ≈ .0001. The other two are approxi-
mately .01, but sin(x) < x for x > 0.

24. 12 3. [41] S(P ) has the same value for all points P . The lines from P to the vertices
of the triangle divide the triangle into three subtriangles. The area√ of each subtriangle
1 1
is 2 h, where h is its altitude. The total area of the triangle, 4 3, equals the sum of
the areas of the three subtriangles, which is 12 (h1 + h2 + h3 ). The sum which we desire
is h1 + h2 + h3 .
25. 432. [44] Since extraneous factors are to be avoided, N should be of the form 2a 3b .
The first requirement forces a to be even and b to be odd. The second requirement
forces a ≡ 1 mod 3 and b ≡ 0 mod 3. Thus a = 4 and b = 3 is the smallest possibility,
and N = 16 · 27 = 432.
26. 198. [33] This is like Pascal’s
¡ ¢ triangle. At level n, the fraction of the people in
horizontal position i is ni /2i , and so the number of people in position 1 at level 99
¡ ¢ 100 99
is 99
1 2 /2 = 198.
27. 30. [22] Call the colors 1 through 6. There are 5 choices for the color opposite color 1.
Then there are 3 choices for the color opposite the smallest color not yet used. The
two ways of placing the remaining two colors yield inequivalent colorings. Thus the
answer is 5 · 3 · 2.

6
¡7¢
28. 35. [39] It is the binomial coefficient 3 since there must be 7 steps of which exactly
3 move in the x direction.
29. 19 11
21 or
410
21 . [32] Let B be the angle to the top of the building.
1
x + 40 tan(A) + tan(B) +4 25
= tan(A + B) = = 5 154 = .
50 1 − tan(A) tan(B) 1− 55 21
1250
Thus x = 21 − 40.
30. 12 67 or 90
7 . [21] Let c denote the hypotenuse, and a and b the legs. Then a + b + c = 30,
5c = ab, and a2 + b2 = c2 . Eliminating c yields 30 − a − b = ab ab 2 2
5 and ( 5 ) = a + b .
2

Equating the two expressions for ( ab 2 2


5 ) leads to 60a + 60b = 30 + 2ab, while the first
says 60a + 60b = 30 · 60 − 12ab. Equating these yields ab = 900/14 and hence c = 90 7 .

31. 8,9,10,11,12. [32] The sum of the interior angles of a convex n-gon is (n − 2)180.
This must equal (n − 1)150 + a with 0 < a < 180. We obtain a = 30(n − 7), Thus
7 < n < 13. It is easy to check that each of these convex n-gons can be formed.
3
32. 47 11 or 520
11 . [18] Let t denote the number of hours during which Chad was driving
9
back to pick up Alan. Then 4(1 + t) = 40 − 40t, hence t = 11 . Although you can
compute it by solving equations, you can also reason it out that Bill will have to
drive forward the second time for exactly one hour in order to catch up with Bill. The
reason for this is that if we forget about the time period when they were both walking,
we see that when they eventually meet, they both will have been in the car the same
amount of time. So, during the two hours when one was walking and one riding each
will have traveled 44 miles, and during the other 9/11 hour, each will have traveled
36/11 miles.
33. 7, 37. [21] (x − 11)(x2 − 107) is composite or 0 except possibly when x − 11 = ±1 or
x2 − 107 = ±1. Since 106 and 108 are not perfect squares, the latter does not happen
for integer x. If x = 10 or 12, then the product equals 7 or 37.
34. (3, 2). [25] Suppose the term immediately before 81 is x. Then, working backwards
from there, the next terms are 81 − x, 2x − 81, 162 − 3x, 5x − 243, 405 − 8x, 13x − 648,
1053 − 21x, and 34x − 1701. To make these be all positive, we must have x ≤ 80,
x ≥ 41, x ≤ 53, x ≥ 49, x ≤ 50, x ≥ 50, x ≤ 50, and x ≥ 51. The last inequality is
inconsistent with those which precede it. Hence x = 50, and the sequence is 3, 2, 5,
7, 12, 19, 31, 50, 81.
35. x4 − 8x3 + 21x2 − 20x + 7. [22] One way is to write p(x) = ax4 + bx3 + cx2 + dx + 7
and solve four linear equations for a, b, c, and d. A better way is to note that if
q(x) = p(x + 2), then q(±1) = 1 and q(±2) = 7, and so q(x) = ex4 + f x2 + 3. We
get e + f = −2 and 16e + 4f = 4, from which we obtain e = 1 and f = −3. Thus
p(x) = q(x − 2) = (x − 2)4 − 3(x − 2)2 + 3, which is easily expanded to the claimed
polynomial.

36. 1 + 12 6. [10] The centers of the four spheres form a regular tetrahedron of side length
2. The center of the the bounding sphere must be at the point in the tetrahedron which

7
is the intersection of its altitudes.
q Each altitude will have one end at the centroid of
√ p
a face, so will have length 2 − ( 23 3)2 = 8/3. Just as the intersection of the
2

altitudes of an equilateral triangle is 1/3 of the way up the altitudes, the intersection
of the altitudes of a regular tetrahedron is 1/4 of the way up the altitudes. Thus
the
p distance √from the center of the bounding sphere to one of the other centers is
3 1
4 8/3 = 2 6. We add 1 to this to get the radius of the bounding sphere.
37. 363636. [6] This is the case a = 1000 and b = 729 of the following, which has
P b
a−1
(a − 1)(b − 1)/2 as the answer. Evaluate [ a n] if a and b have no common factors.
n=1
The answer is best obtained by thinking of the grid along integer lines in the rectangle
[0, a] × [0, b]. Since a and b have no common factors, the diagonal from (0, 0) to (a, b)
will not pass through any lattice points, except at its endpoints. Thus the desired
answer is the number of lattice points having y > 0 lying below the line which satisfy
1 ≤ n ≤ a − 1. This diagonal will exactly bisect the rectangle [1, a − 1] × [0, b]. The
total number of lattice points in this rectangle, excluding the top and bottom edges,
is (a − 1)(b − 1), and half of them lie below the line.

38. 3 2. [11] Since AC is a diameter, B is a right angle, and so angle CBD and DBA are
45 degrees. Angle CAD is also 45 degrees√(π/4 radians), √ since it subtends the same
arc as does 2 2
√ angle CBD. We have AC = 2 + 4 = 2 5, and, if θ = BAC, then
6
sin θ = 1/ 5. By the Extended Law of Sines, BD/ sin(6 BAD) equals the diameter
of the circle. Thus
√ √ √ √ √
BD = 2 5 sin(θ + π4 ) = 2 5 22 (sin(θ) + cos(θ)) = 10 √35 = 3 2.

..............................
......... ......
...... .....
..
....... .....
...
..
B . .
.
..
..
.......
.
.... ................
. .............
...
...
...
......... ..... .............
.............
...
.... ... ..
.....................................................................................................
C ...
...
...
... ..
..... ..
. ...
. A
.
... ... ...... ....
.....
... ... ...... ..
...
.... .....
. .
...... ...
..... .. ...
......
..
....
.
...... ... ......... . ...
........ ...... ......
.......................................
D

39. 780. [3] After giving one to each, there are 13 left to distribute. You can enumerate
the number of distributions directly, by considering all the ways to have five numbers
add up to 13, and multiply each possibility by the appropriate permutation number.
But generating functions provide a better method. Let p(x) = 1 + x + · · · + x5 . Then
the monomials in p(A)p(B)p(C)p(D)p(E) whose exponents sum to 13 correspond to
the desired ways. The number of these is the coefficient of x13 in
µ ¶5
5 1 − x6
p(x) = = (1 − 5x6 + 10x12 − · · · )(1 − x)−5 .
1−x
¡n+4¢ ¡5¢ ¡11¢ ¡17¢
Since the coefficient of xn in (1 − x)−5 is 4 , the answer is 10 4 − 5 4 + 4 =
50 − 5 · 330 + 2380 = 780.

8
40. − 14 , 21 . [3] We have α + β = 2p and αβ = p2 − 2p − 1. Thus

(α − β)2 = (2p)2 − 4(p2 − 2p − 1) = 4(2p + 1).


4p+1
We want real values of p for which 4p 2 +2 = n, with n an integer. If n = 0, we get
1
p = − 4 . If n 6= 0, this yields a quadratic equation for p, with solution
p
1± 1 − n(2n − 1)
p= .
2n

This gives a real solution for p only if − 12 ≤ n ≤ 1. The only nonzero integer n in
this range is 1, yielding p = 21 . (Although not necessary for solving the problem, one

can find that for p = − 14 , {α, β} = {(−1 ± 2 2)/4} and if p = 12 , then {α, β} =

{(1 ± 2 2)/2}.)

9
æ
2012 contest. Questions are annotated with the number of people answering them correctly.
The first is the number out of the 44 people who scored at least 21, and the second is out
of the other 316 people.
1 2
1. [44,314] Express 3 + 5 as a reduced fraction.
(x+2)(2x−1)
2. [44,280] List all values of x that satisfy x−3 = 0.
3. [43,271] What is the area of a triangle whose sides are 13, 13, and 10?
4. [43,238] How many of the 2-digit numbers from 10 to 99 have the property that both
digits are perfect squares? For example, 10 is the smallest such number and 99 is the
largest.
5. [42,134] How many digits are in the base 10 number 422 · 540 ? For example, 9001 has
four digits.
6. [43,239] Find A if there is a polynomial identity

(x3 +2x2 −3x−2)(x4 +3x3 +Ax2 −6x+1) = x7 +5x6 +5x5 −13x4 −23x3 +16x2 +9x−2.

7. [38,98] If 1.00000423762 = 1.00000xyz521795725376, what is the value of x + y + z?


8. [42,249] The ratio of children to adults at a party is 2:3. A busload of 30 more children
arrives at the party, and now the ratio of children to adults is 3:2. How many people
were at the party before the bus arrived?
9. [41,109] Find the area of the region consisting of all points (x, y) satisfying 1 ≤ |x| +
|y| ≤ 2.
10. [38,113] A sequence is defined by a1 = 1 and, for n ≥ 1,

 0 if an = 0 and n is odd



 2 if
 an = 0 and n is even
an+1 = 1 if an = 1 and n is odd



 0 if an = 1 and n is even


1 if an = 2.

How many of the numbers a1 , a2 , . . . , a2012 are equal to 2?


11. [35,130] You are trying to guess an integer between 1 and 1000, inclusive. Each time
you make a guess, which must be an integer, you are told whether your number is
too high, too low, or correct. What is the smallest number of guesses required to
guarantee that you will have guessed the number?
12. [27,30] A number system based on 26 uses the letters of the alphabet as its digits,
with A = 0, B = 1, C = 2, . . . , Y = 24, and Z = 25. Express ON E + ON E in this
system.
13. [38,65] A circle is inscribed in quadrilateral ABCD. If AB = 4, BC = 5, and CD = 8,
what is DA?
14. [41,195] What is the 4-tuple (w, x, y, z) which satisfies all four of the following equa-
tions?

w+x+y =4
x + y + z = −5
y+z+w =0
z + w + x = −8

15. [41,195] The outside of a cube is painted black, and then it is cut up into 64 smaller
congruent cubes. How many of the smaller cubes have at least one face painted black?
16. [34,43] This problem deals with 5-digit numbers, and for the purposes of this problem,
we allow 0’s as the initial digits. Thus the number of 5-digit numbers is 105 under
this interpretation (which differs from the usual convention). Write as a decimal the
probability that a randomly selected 5-digit number contains exactly 4 distinct digits
(under the interpretation of this problem).
17. [37,47] How many ordered pairs (x, y) of integers satisfy x2 + 6x + y 2 = 16?
18. [33,72] Circles of radius 10 and 17 intersect at two points. The segment connecting
these points of intersection has length 16. List all possible values for the distance
between the centers of the circles?
19. [40,80] Of all the nonempty subsets S of {1, 2, 3, 4, 5, 6, 7}, how many do not contain
the number |S|, where |S| denotes the number of elements in S? For example, {3, 4}
is one such subset, since it does not contain the number 2.
20. [32,40] Let (x1 , y1 ), . . . , (x6 , y6 ) denote the vertices of a regular hexagon whose center
is at (2, 0) and which has one vertex at (3, 0). Let z1 = x1 + iy1 , . . . , z6 = x6 + iy6
denote the corresponding complex numbers. What is the product z1 · · · z6 of these six
complex numbers?
21. [33,47] What is the value of the continued fraction
1
3+ 1 ?
4+ 1
3+ 4+···

The 3’s and 4’s alternate indefinitely.


22. [37,48] List all ordered triples (a, b, c) of positive integers which satisfy a + cb = 11 and
b + ac = 14.
23. [23,6] Let Fn denote the Fibonacci numbers, defined by F1 = F2 = 1 and Fn+2 =
Fn+1 + Fn for n ≥ 1. Express the infinite sum
F1 F2 F3 F4
S= + 2 + 3 + 4 + ···
5 5 5 5
as a reduced fraction.
24. [16,8] Let A1 , . . . , A6 denote the vertices of a regular hexagon inscribed in the circle
x2 + y 2 = 3. Circles of radius 1 are drawn with centers at each of these six points,
and a seventh circle of radius 1 is drawn, centered at the origin. What is the area of
the region common to at least two of the 7 circles of radius 1; i.e., the union of the
intersections?
25. [26,17] Let A = sin x + cos x. Write sin4 x + cos4 x as a polynomial in A.
26. [33,67] What is the largest positive integer n such that n is divisible by every positive
integer m which satisfies m2 + 4 ≤ n?
27. [27,31] Points A, B, C, and D lie in a plane, with A, B, and C in order in a straight
line. The angles between some of these points, measured in degrees, satisfy BDC = 78,
DBC > DCB, ABD = 4x + y and DCB = x + y, for some numbers x and y. How
many positive integer values can y take on, satisfying all these conditions?
28. [27,37] Alice and Bill are walking in opposite directions along the same route between
A and B. Alice is going from A to B, and Bill from B to A. They start at the same
time. They pass each other 3 hours later. Alice arrives at B 2.5 hours before Bill
arrives at A. How many hours are required for Bill’s trip from B to A?
29. [25,29] What is the smallest positive integer that can be written as the difference of
two positive perfect squares in at least three distinct ways?
30. [30,18] What is the ratio V 2 /A3 , where V is the volume of a regular octahedron and
A is its surface area?
31. [21,9] In triangle ABC, the median from A is perpendicular to the median from B. If
AC = 6 and BC = 7, then what is AB?
32. [5,1] Eleven points are arranged on a semicircle with five on the straight line segment
and six on the arc. See the diagram below for a possible configuration. Every pair
of these points is joined by a straight line segment, and it turns out that no three of
the line segments intersect at a common point in the interior of the semicircle. How
many points are there in the interior of the semicircle where two of the line segments
intersect? ...........•
........•
....................
.......• •..................
.
.......
..•
.
...
.
...
...
.
•..............
... ... ...
... ...
... ...
... ...
.... ...
... ...
..
• • • ••
..............................................................................................................

33. [3,0] A trapezoid has ratio of its bases 2:1, ratio of its legs (nonparallel sides) 2:1, and
ratio of its diagonals 2:1. What is the ratio of its shorter base to shorter leg?
34. [13,8] If x and y are randomly selected real numbers between 0 and 1, what is the
probability that the integer closest to y−x
y+x is odd?

35. [27,14] In triangle ABC, AB = 3, BC = 4, and AC = 6. If BC is extended through


C to D so that CD = BC, what is AD?
36. [15,1] Alice, Bill, Chris, Don, and Emily take a test. In how many different orders
can they finish if ties are allowed? (For example, if there were three people, it would
be 13, since there are 6 orders without ties, 1 with a 3-way tie, 3 with a 2-way tie for
first, and 3 with a 2-way tie for last.)
37. [9,0] List all positive integers b for which there is a positive integer a < b such that
exactly 1/100 of the consecutive integers a2 , a2 + 1, . . . , b2 are perfect squares.
38. [18,4] Let an denote the nth smallest positive integer for which the sum of its decimal
digits equals 3. For example, a1 = 3, a2 = 12, a3 = 21, and a4 = 30. How many digits
are there in a2012 ?
39. [1,1] You flip a fair coin repeatedly until either four consecutive Heads (H) or six
consecutive tails (T) occur. What is the probability that the sequence HHHH occurs
before the sequence TTTTTT?
¡¢
40. [7,1] Eight points on the circumference of a circle are chosen and all 82 = 28 chords
connecting them are drawn. It turns that no three chords intersect in the same point
in the interior of the circle. Into how many regions do these chords divide the interior
of the circle?
Solutions to 2012 contest.

5 6
1. 11/15. It is 15 + 15 .

2. −2 and 1/2 (must have both). Must make the numerator equal to zero while keeping
the denominator nonzero.
3. 60. This is an isosceles triangle composed of two 5-12-13 right triangles, each of which
has area 30.
4. 12. The first digit can be any of 1, 4, or 9, while the second digit can be any of 0, 1,
4, or 9.
5. 42. The number is 42 · (4 · 52 )20 = 16 · 1040 .
6. 2. The coefficient of x2 in the product, 16, must equal −2A + 18 + 2. Solve to get
A = 2.
7. 19. 1 + t2 = 1 + 2t + t2 . Here t = .0000042376. The t2 is much too small to affect the
first three nonzero digits of 2t. We obtain xyz = 847.
8. 60. C = 23 A, while C + 30 = 32 A. Solving yields 56 A = 30, so A = 36 and C = 24.
9. 6. The region consists of the points lying inside a square whose vertices are at (±2, 0)
and (0, ±2) and outside a square whose vertices are at (±1, 0) and (0, ±1). The outer
square has area 8 and the inner square area 2.
10. 502. Starting with a1 , the sequence goes 1, 1, 0, 0, 2, 1, 0, 0, 2, and keeps repeating
with period 4. Thus an = 2 if and only if n = 5, 9, etc., 1 greater than a positive
multiple of 4. a2009 will be the 502nd 2.
11. 10. This is because 210 = 1024 > 1000. The best that you can guarantee after 1 guess
is to limit it to 500 numbers, then 250, 125, 62, 31, 15, 7, 3, and 1. After 9 guesses,
you know what the answer is, but you still must make the tenth guess.
12. BDAI. The rightmost position is 4 + 4 = 8 = I. The next position is 13 + 13, for
which we enter 0 = A and carry 1. The next position is 1 + 14 + 14, for which we
enter 3 = D and carry 1 = B to the leftmost position.
13. 7. Let a, b, c, and d denote the length of a tangent from A, B, C, and D to the
inscribed circle. Then a + b = 4, b + c = 5, and c + d = 8. Subtract the second
equation from the sum of the others to obtain a + d = 7.
14. (2, −3, 5, −7). Add the four equations, obtaining w + x + y + z = −3. Subtract each
equation from this to obtain the value of one of the variables.
15. 56. The cube is cut into fourths in each direction. You can enumerate the number
of pieces from the center of a face (24), an edge (24), and a corner (8), or you can
subtract the 2 × 2 × 2 cube in the center of the big cube, which are the only subcubes
without a black face.
¡¢
63
16. .504 or 125 . There are 10 ways to choose which digit is repeated, and 52 = 10 ways
to choose which two positions of the number will have the repeated digit. Then there
are 9 · 8 · 7 ways to fill the other three positions with distinct remaining digits. The
answer is 10 · 10 · 9 · 8 · 7/105 .
17. 12. The equation can be written as (x + 3)2 + y 2 = 25. This implies that the set
{x + 3, y} is {0, ±5} or {(±3, ±4}. There are four ordered pairs of the first type and
eight of the second.
18. 9 and 21. (Must have both.) The line connecting the centers is the perpendicular
bisector of this chord, intersecting it at point P . If O and O0 are the centers of the
circles, then OO0 = P O0 ± OP , and OP and P O0 are bases of right triangles with
height 8 and hypotenuses 10 and 17, respectively. By the Pythagorean Theorem, OP
and P O0 are 6 and 15.

............................... ...............................
........ ...... ........ ......
...... ..... ...... .....
........ ..... ........ .....
......
...... ........ .
........... . ... . . ....................... ...
......
. . .. ... ... .
. ....... ......
.... ...
.... . ........ .............. ... ....................... ...
... ...... ... ... ........... ... ...... ... .... ............ ..... ...
... .. .. .. ...
. ....... ... ... ... .... ..... .......... ... ...
... ............................................................ 0 .. ... ... ................................... . ..
... .
O .... .... .... . . O ... ... ... ... O 0 .... ...
...
... ..... ... .
... ...... ..
........
O .
.
. .
...
. . .
..... .........
...
.. ........ ..... ...
..
........
........................
. ...............................
... ..... ...
.....
...... ........ ..... ........
....... ... ...... ......
.
..................................... .........
............................

19. 63. There are 27 − 1 = 127 nonempty subsets¡ 6of¢{1, . . . , 7}. The number which have
k elements and do contain the number k is k−1 . Summing this as k goes from 1 to
7 yields 26 = 64 subsets S which contain the number |S|. The number which do not
is 127 − 64 = 63.
√ √
20. 63. The numbers zj are 3, 1, 52 ± i 23 , and 32 ± i 23 . Their product is 3 25+3
4
9+3
4 = 63.
Another method notes that the numbers zj are the complex roots of (z − 2)6 − 1 = 0
and so their product is 26 − 1.

21. 32 + 3. If x is the answer, then 3 + 1/(4 + x1 ) = x. This reduces to x = (x − 3)(4x + 1),
hence 4x2 − 12x − 3 = 0, which is easily solved by the quadratic formula. Since the
number is positive, we choose only the positive square root.
22. (8, 12, 4). Let a = αc and b = βc, with α and β integers. Subtract equations and
obtain (β − α)(c − 1) = 3. If c = 2, then β − α = 3 and 2α + β = 11, implying α = 8/3,
not an integer. If c = 4, then β − α = 1 and 4α + β = 11, yielding α = 2, β = 3.
F2 F3
23. 5/19. 5S = 1 + 5 + 52 + · · · . Subtract the original equation and obtain

F2 − F1 F3 − F2 F4 − F3 F1 F2 1
4S = 1 + + + + · · · = 1 + + + · · · = 1 + S.
5 52 53 52 53 5
19
Thus 5 S = 1.

24. 4π − 6 3. There are 12 regions, each of which is twice as large as the difference
between a 60 degree wedge in√
a unit circle and an equilateral triangle of side length 1.
π 3
Thus the answer is 24( 6 − 4 ). Note that the centers of any two adjacent circles are

3 apart, which implies that the radius to an intersection point makes a 30 degree
angle with the line connecting the centers.
...................
..... ...
.......... . ... ... ........
. .
...... ...
........ ....... .........
. . ....... ..
.....
..... .
.
.........................
...
..
... . ... ..
. ........ ...... .
... ...... ... .. ..
............................. .........................
. .
..... ............ . ..
....... ..
.. .. . ............ ..
...
... ....... ......... ........... ...
..... ...... ....... ..
... .. ............................. ....
.
..... . . .. ...... .
..
.
................ . . .
..................
... ...
....
......................

25. 1 2 1 4
2 +A − 2 A . We have A4 = sin4 x+cos4 x+4 sin x cos x(cos2 x+sin2 x)+6 sin2 x cos2 x.
Also A = sin x + cos2 x + 2 sin x cos x. Thus sin x cos x = 12 (A2 − 1), and sin4 x +
2 2

cos4 x = A4 − 4( 21 (A2 − 1)) − 6( 21 (A2 − 1))2 = A4 − 2A2 + 2 − 23 (A4 − 2A2 + 1).


26. 24. 24 works, since it is divisible by 1, 2, 3, and 4, while 25, 26, 27, and 28 do
not, since they are not divisible by both 3 and 4. To see that no larger n works, let
k 2 + 4 ≤ n < (k + 1)2 + 4, with k ≥ 5. Then n must be divisible by both k and k − 1,
and hence by k(k − 1), since k and k − 1 are relatively prime. However, since k ≥ 5,
we have 2k(k − 1) ≥ (k + 1)2 + 4, so n is less than 2k(k − 1) and hence must equal
k(k − 1), and thus cannot be ≥ k 2 + 4.
27. 24. Let α = DBC and β = DCB. We have α + β = 102, 180 − α = 4x + y, and
x + y = β. These imply 3x = 78, so x = 26. Then 76 − y = α > β = 26 + y, so y < 25.
We easily check that each value of y from 1 to 24 works.
28. 7.5. Let sA and sB denote their speeds, and t the desired time when Bill arrives at A.
Then sA (t − 2.5), sB t, and 3(sA + sB ) all equal the distance between the two points.
Let r = ssBA
. Then r(t − 2.5) = t = 3(r + 1), which reduces to t2 − 8.5t + 7.5 = 0, so
t = 7.5, since t > 1.
29. 45. It can be written as (7 + 2)(7 − 2), (9 + 6)(9 − 6), and (23 + 22)(23 − 22). This
shows that what we are really looking for is factorizations of n as a product of two
positive integers whose difference is even (since (a + b) − (a − √ b) = 2b). If n is odd,
we require three divisors of n (including 1) which are √ less than n, while if n is even,
we require three even divisors d of n, each less than n, such that n/d is also even.
It is easy to see that 45 is the smallest n for which this can be done.

30. 3/324. The octahedron √ is formed √
from eight equilateral triangles. Let s denote the
2 2
sidelength. Then A = q8s 3/4 =√2 3s . The altitude from the middle of the center
3 1 3

square to a peak is s 4 − 4 = s 2/2, so the volume is s 2/3. The desired ratio is
2/9
√ .
24 3

31. 17. Let x and y be as in the diagram below, in which the two medians are perpen-
dicular. Then x2 + 4y 2 = 49/4 and 4x2 + y 2 = 9. Hence

AB 2 = 4x2 + 4y 2 = 45 ( 49
4 + 9) = 17.

A..
............
... .. .......
.... ..... ..........3
. ......
. ...
... . ......
... 2x..... ......
.
.... ... y ........................
... ............. ......
......
...
. ......... ....3
......... ....x
... 2y ......... ......
.. ........ .
. ......
............... ... ...
..................................................................................................................
.
B 7/2 7/2 C

32. 265. Any four of the eleven points which include at least two on the semicircle will
determine
¡6¢ ¡6¢¡5¢exactly
¡6¢¡5one
¢ point of intersection. The total number of such groups is
4 + 3 1 + 2 2 = 15 + 100 + 150.
√ √
33. 12 10 : 1 or just 21 10. We let the shorter leg equal 1, and wish to find x in the
diagram below, in which the height is h. We have 1 − y 2 = h2 = 4 − (x − y)2 , and
then comparing diagonals, 4(h2 + (x + y)2 ) = h2 + (2x − y)2 . We substitute the two
expressions for h2 into this, and obtain

4(4 − (x − y)2 + (x + y)2 ) = 1 − y 2 + (2x − y)2 ,

which simplifies to 20xy+15 = 4x2 . Equating the two equations for h2 yields 2xy+3 =
x2 . Multiply this by 10 and subtract the earlier equation to obtain 6x2 = 15.

x
.............................................................
...... ........................ .... ...........
.... .... ..... ................... ......
1 .
.. ..
.. .. .......
... .............
......
..
2.
... ............ ...........
......... ......
......... ......
.. .
.. ...... .. ... ...............
........ . . .............
............................................................................................................................
y x x−y

34. 1/3. Let r = xy . The specified ratio is r−1 2


r+1 = 1 − r+1 . This is an increasing function
of r for 0 ≤ r < ∞, and assumes values −1, − 12 , 12 , 1, for r = 0, 31 , 3, and ∞. Thus
for (x, y) in the unit square (and, in fact, in the first quadrant), the integer closest to
the specified ratio is −1 for (x, y) below the line y = 13 x, 1 above the line y = 3x, and
0 in between. For (x, y) in the unit square, the area for which this value is odd is 1/3,
the union of two right triangles with legs 1/3 and 1.

35. 95. By the Law of Cosines 62 = 32 + 42 − 24 cos B, so cos B = −11/24. Now
AD2 = 32 + 82 − 48 cos B = 73 + 22.

A..................................
... ........ .............
... ....... .............
.......
...
... ....... .......................
....... ............
... ... ............
3 .... 6 .................... ............
............
............
... ........ ............
... ..
....... ............
... .......... ..
...................................................................... ................................................................................
B 4
.
C 4 D
36. 541. Let f (n) denote the number of outcomes with n people, with the convention that
n−1
X¡ ¢
n
f (0) = 1. Then f (n) = i f (i), where the i-summand is where there are i people
i=0
who were not first or tied for first. Iteratively compute f (1) = 1, f (2) = 3, f (3) = 13,
f (4) = 75, and f (5) = 541.
37. 60, 68, 100 (must have all). We require (b − a + 1)/(b2 − a2 + 1) = 1/100. This can
be manipulated to (b + a − 100)(b − a) = 99. Then each 2b − 100 equals the sum of
the two factors of a factorization of 99 as a product of two integers, i.e. (11,9), (33,3),
and (99,1). Therefore 2b equals 100 plus one of 20, 36, or 100. (The corresponding
values of a are 49, 65, and 99, respectively.)
¡ ¢
38. 22. There are d+2 3 numbers with ≤ d digits of the desired form. This can be
seen by filling in d + 2 positions with exactly three O’s and the rest X’s, with the
correspondence being that the ith digit of the number is the number of O’s between
the (i − 1)st ¡and¢ ith X’s. For ¡example,
¢ with d = 5, XXOOXXO corresponds to
00201. Since 24 3 = 2024, while 23
3 is much smaller, the result follows.
39. 21/26. Let E denote the event that HHHH occurs before TTTTTT. If s a sequence
of tosses, let P (E|s) denote the probability that E occurs, given that s occurs at the
start. Let x = P (E|H) and y = P (E|T ). Then

x = 21 P (E|HT ) + 14 P (E|HHT ) + 18 P (E|HHHT ) + 1


8 = 78 y + 81 ,

while

y = 12 (P (E|T H) + 14 P (E|T T H) + 18 P (E|T T T H) + 1


16 P (E|T T T T H)
1
+ 32 P (E|T T T T T H) = 31
32 x.

32 31
Solving these equations yields x = 39 and y = 39 , so the desired probability is
1 63
2 (x + y) = 78 .

40. 99. The eight points on the circumference and points of intersection of the chords
together with the portions of chords
¡ ¢ and arcs between vertices form a graph (in the
sense of graph theory). It has 84 = 70 vertices in the interior, and so 78 vertices
altogether. There are 12 (70 · 4 + 8 · 9) = 176 edges, obtained as 12 times the number
of vertex-edge intersections. Here we have used that each of the eight vertices on the
circumference meets 7 chords plus two arcs. By Euler’s formula, 78 − 176 + R = 1.
1. What is the sum of the digits of (1010101)2 ?
2. Find two numbers whose sum is 52 such that one is three times
as large as the other.
3. What is the area of a circle inscribed in a square of side-length
3?
4. A student takes three exams. The second has twice as many
questions as the first, and the third has three times as many
questions as the first. The student answers exactly 75% of the
questions correctly on the first exam, exactly 81% on the second
exam, and exactly 85% on the third exam. Out of all the ques-
tions on the three exams, what percent did he answer correctly?

5. How many members of the set { 71 , 72 , 37 , . . . , 99


7
} are integer mul-
2
tiples of 5 ?
6. List all positive solutions of x3 + x2 − 2x − 2 = 0.
7. Find all possible ordered pairs (A, B) of digits for which the
decimal number 7 A 8 B is divisible by 45.
8. A sequence satisfies a1 = 3, a2 = 5, and an+2 = an+1 − an for
n ≥ 1. What is the value of a2013 ?
9. Find the ordered triple (a, b, c) of positive integers which satis-
fies both (a + b)(a + c) = 77 and (a + b)(b + c) = 42.
10. The set of points (x, y) which satisfy |x| ≤ 1, |y| ≤ 1, and
|y| = |x| + x consists of several line segments. What is the sum
of the lengths of these segments?
11. What is the smallest positive integer with more than 13 positive
divisors?
12. How many positive integers less than 1200 have no repeating
digits; i.e., no digit occurs more than once.

1
2

13. The sum of the first ten terms of a nonzero geometric series is
244 times the sum of the first five terms. What is the common
ratio?
14. Five circles of equal radius are placed inside a square of side
length 1 in such a way that no two intersect in more than one
point. What is the largest possible radius for these circles?
15. You take six steps, each time moving either one step forward
or one step back, with probability 12 of each each time. What
is the probability that you end up back where you started?
16. In the diagram below, DA = AB = BE, GA = AC = CF , and
IC = CB = BH. If EF = 5, DI = 6, and GH = 7, what is
the area of triangle ABC?
D
..... G
.....
... ...... ... ......
.. ..... ... .....
... ....
..
...
...
... ...A...... ...
...
... ... ...... ...
I ...............................C .......................................H
..............B
.. ....
... ....
..........................................................
F E

17. Assume that for every person the probability that they have
exactly one child is 1/4, the probability that they have exactly
two children is 1/2, and the probability that they have exactly
three children is 1/4. What is the probability that a person will
have exactly four grandchildren?
√ 1
18. If x = 2 + 3, find an integer or fraction equal to x4 + 4 .
x
19. Let S = {3, 4, 5, 6, 8, 9, 10, 11, 12}. What is the sum of the ele-
ments in S which are divisors of 7021500420?
20. What is the largest base-10 number whose base-3 and base-4
expansions have the same number of digits?
3

21. Find the area of triangle ABC satisfying the following proper-

ties. AC = 13 and CM = 4 10, where M is the midpoint of
AB. The altitude from C does not meet AB itself, but it meets
the extension of AB, and this altitude has length 12.
22. Four circles of radius 1 are tangent to one another as in the
diagram below, except that clearly the top and bottom circles
are not tangent to one another. A tight band is wrapped around
the four circles as in the diagram. What is its length?
.
..................
....... .
.
........... ................
. .
..... ................ .......
.... .
...... ................ ......
........... ..........
..... ......
.... ..
................

23. Diameter AB of circle with center O has length 2. From the


midpoint Q of OA, a perpendicular is drawn intersecting the
circle at P . Find the radius of the circle which can be inscribed
in triangle AP B.
24. Find the maximum value of x + y taken over all ordered pairs

(x, y) of real numbers which satisfy x2 + y 2 = 4 and x + 5y ≥

2 5.
25. The bases of a trapezoid have lengths 12 and 15. Find the length
of the segment parallel to the bases which passes through the
point of intersection of the diagonals and extends from one side
to the other.
26. How many ordered pairs of integers (x, y) satisfy x+y−xy = 49?

27. Compute the number of ordered 4-tuples (a, b, c, d) of positive


integers such that a + b + c + d = 14.
4

28. For n ≥ 1, let dn denote the length of the line segment connect-
ing the two points where the line y = x + n + 1 intersects the
1000
2 1
X 1
parabola 8x = y − 32 . Compute the sum .
n=1
n · d2n
29. Let P (x) be a monic polynomial of degree 3. (Monic here means
that the coefficient of x3 is 1.) Suppose that the remainder when
P (x) is divided by x2 −5x+6 equals 2 times the remainder when
P (x) is divided by x2 − 5x + 4. If P (0) = 100, what is P (5)?
30. What is the range of values of a + 3−b a
taken over all ordered
pairs (a, b) of real numbers for which x2 −ax+b has two distinct
positive roots? Use interval notation to express your answer.
31. A bug starts at a corner vertex of the graph below and at each
step moves along an adjacent edge, with equal probability for
any adjacent edge. It continues until it gets to the center vertex.
What is the expected (average) number of steps required until
the first time it gets to the center vertex?
•....................•....................•...
... . .
.•..................•....................•....
... .. ..
... .. ..
•.................•..................•..

32. List all positive integers which have exactly 6 positive divisors,
the sum of whose reciprocals is 2.
33. List all numbers which can be written as x + y where x and y
are positive integers satisfying x4 + y 4 = 15266.
34. Compute all integers r such that a circle of radius r with di-
ameter AC has a point B on the circle such that AB = 30 and
BC is a positive integer.
35. Out of all positive integers n which have exactly 2107 positive
divisors, what is the largest number of positive divisors that n2
could have?
5

36. What is the smallest positive integer that cannot be written as


the sum of 11 or fewer factorials? Note that the factorials need
not be distinct.
3
37. Let a, b, and c be distinct nonzero real numbers such that 1+a
a
=
1+b3 1+c3 3 3 3
b
= c . Determine all possible values of a + b + c .
38. In the triangle below, angles P AB, P BC, and P CA are each
equal to the same value α. The sides of triangle ABC have
lengths 7, 8, and 9. Find tan(α).
C....
. ...
... ..........
.. ...α....
... .... ......
.. P......... ....
... ............ ..................
. ..........
..α....... .. .
....................................................................α......................
B A

39. What is the largest positive integer n, not a multiple of 10, such
that removing the last two digits of n2 leaves a perfect square?
40. Let P (x) be a polynomial of degree 10 such that P (2i ) = i for
0 ≤ i ≤ 10. What is the coefficient of x1 in P (x)?
6

Solutions to 2013 test, annotated with the number of the 72 people who scored at
least 20 answering the question correctly.

1. 16. [69] The product is 1020304030201.


2. 13 and 39. [71] We have x + 3x = 52 so x = 52/4 = 13.
3. 9π/4. [72] The radius of the circle is 3/2.
4. 82 or 82%. [70] If there were n questions on the first exam, then
the student answered correctly .75n + .81 · 2n + .85 · 3n out of
6n. The answer is (.75 + 1.62 + 2.55)/6 = 4.92/6 = .82.
5. 7. [64] If a7 = k 25 , then 14k = 5a, so a must be a multiple of 14.
There are 7 such numbers less than 100.

6. 2. [63] The polynomial factors as (x + 1)(x2 − 2).
7. (3, 0) and (7, 5). [70] The number is divisible by 5 and by 9, so
B must be 0 or 5, and the sum of the digits must be divisible
by 9.
8. 2. [64] One easily checks the first eight values to be 3, 5, 2, −3,
−5, −2, 3, 5. It becomes clear that these are repeating with
period 6. Since 2013 is 3 more than a multiple of 6, a2013 =
a3 = 2.
9. (6, 1, 5). [70] Since the only common divisors of 77 and 42 are 1
and 7, and a + b > 1, we must have a + b = 7. Then a + c = 11
and b+c = 6. Subtracting the first two of these equations yields
c − b = 4. Adding this to the third gives 2c = 10, from which
the rest follows easily.

10. 1 + 5. [56] The lines are y = ±2x for x > 0, and y = 0 for
x < 0. Inside the box, we have segments from (0, 0) to (−1, 0),
√ √
and to ( 21 , ±1), of lengths 1, 12 5, and 12 5.
11. 120. [35] Since 120 equals 23 · 3 · 5, it has 4 · 2 · 2 = 16 divisors.
Smaller candidates with lots of divisors are 60 and 96, but each
of them only has 12 divisors.
7

12. 794. [39] There are 9 such 1-digit numbers, 9 · 9 = 81 such 2-


digit numbers (since the first digit can be 1 to 9 and the second
digit any other number from 0 to 9). Similarly the number of
such 3-digit numbers is 9·9·8 = 648, since the third digit cannot
equal either of the first two. For 4-digit numbers beginning 10,
there are 8 · 7 and there are none beginning 11.
10
−1 5
−1
13. 3. [69] We have rr−1 = 244 rr−1 . Thus r10 − 244r5 + 243 = 0.
Hence r5 = 1 or 243, but r = 1 doesn’t make sense. So the

answer is 5 243 = 3.

14. 12 ( 2 − 1). [61] If one circle is at the center of the square and
the others are in the corners, then we have 1 = 2r + 4 √r2 . To
see this, let A (resp. B) be the center of the upper right (resp.
lower left) circle. Then the vertical displacement from A to

B is 4r/ 2, and the vertical displacement from each of these
to the nearer horizontal edge is r. Thus r(2 + √42 ) = 1, and

multiplying both sides by 2 − 1 yields the claim. It is not
difficult to prove that any way of putting five equal circles of
radius r in the square must have two centers in a square of

side-length 21 − r, and hence must satisfy 2r ≤ ( 12 − r) 2, which
implies that the above result is optimal.
15. 5/16. [69] There are 26 sequences of F (forward) and B (back),
each equally likely. The number with 3 F’s and 3 B’s is 63 = 20.



16. 3 6/2. [66] By similar triangles, AC = 12 DI, AB = 12 GH,
and CB = 21 F E. Thus the area of triangle ABC is 1/4 times
the area of a triangle with sides 5, 6, and 7, which, by Heron’s

formula, is 9 · 2 · 3 · 4.
17. 27/128. [56] There are three ways to have four grandchildren.
One is to have two children, each of whom have two children.
3
The probability of this is 12 = 81 . Another is to have two
children, with one having one child and the other three. The
2
probability of this is 2 · 12 · 14 = 16
1
. Note that the factor 2 occurs
8

because it can be either the first child or the second child who
has the three children. Similarly the probability that they have
three children, one of whom has two children (and the other two
have one) is 3 · 14 · 32
1 3
= 128 . Adding these three probabilities
yields the desired result.
√ √
2−√3
18. 194. [57] x + x1 = 2 + 3 + 2+1√3 2− 3
= 4. Then x2 + x12 =
(x + x1 )2 − 2 = 14, and x4 + x14 = (x2 + x12 )2 − 2 = 142 − 2.
19. 51. [63] The number is clearly divisible by 4 but not by 8, and
by 5. By the sum-of-digits test, it is divisible by 3 but not by 9.
By the alternating-sum-of-digits test, it is divisible by 11. Thus
of the numbers in S, it is divisible by 3, 4, 5, 6, 10, 11, and 12.
20. 80. [59] Numbers with 4-digit expansions are 27 to 80 in base
3, and 64 to 255 in base 4. For 5-digit expansions, the ranges
are already nonoverlapping.
21. 12. [62] Let P be the point where the altitude from C meets the
extension of AB. Then, by the Pythagorean Theorem, AP = 5,

and the distance of M from P is 160 − 144 = 4. Since M lies
between A and P , AM = 5 − 4 = 1, so AB = 2. Thus the
desired area is 21 2 · 12.
.C
............
..
..... .
.. ............ ...
.. .. . .
. .............. ...
.... .... .. ..
.................................................
AMB P

22. 8 + 2π. [47] The diagram below makes it quite clear that the
band is composed of four segments of length 2 plus four circular
arcs which altogether compose 360 degrees of arc, 120 for the
9

top and bottom and 60 for the two sides.


......
....... .......
......................................
... .... ... .... .. ....
..............................................................
.... . .. . ....
.... ..................................................................... ....
....... .... ............ ... ......
..................... .....................
... . ... .. ... ..
..... ........ .....
.......... ........
..... .....
............


23. 21 ( 3 − 1). [54] Triangle AP O is equilateral, so angle A equals

60 degrees and AP = 1. Thus P B = 3. Thus triangle AP B
√ √
has area A = 3/2 and perimeter p = 3 + 3. Its inradius r
1

3 1

satisfies A = 2 rp, so r = 3+√3 = 2 ( 3 − 1).
...............
. ..................... ...............
.....P
. . . ....
... . . ... .......
........ ... ...... ............... .....
...............................................................................................
.
.
A.. Q O .B
... ..
.
... .
....
...... . . ...
............ ................
......

24. 2 2. [38] The largest value of x + y on the circle occurs at
√ √
the point ( 2, 2). We check to see whether it satisfies the
√ √ √
inequality 2(1+ 5) ≥ 2 5. Squaring both sides, this reduces

to 5 ≥ 2, which is true.
25. 40/3. [54] In the diagram below, by similar triangles we have
PY
AB
= YBCC PY
and CD = BY BC
. Adding these yields P Y ( 12 1 1
+ 15 )=1
and so P Y = 60/9. A similar argument shows that XP also
equals 60/9.
A.................................................B
.. .... .... ..
... ......... .......... ....
. .. .
X .................................................................Y
... ......... P ....... ...
.. ....... ..... ...
..... ..
............. ..
..............................................................................
D C
10

26. 20. [46] The equation may be rewritten as (x−1)(y −1) = −48.
Each of the five factorizations of 48 as a product of two positive
integers, 1 · 48, 2 · 24, 3 · 16, 4 · 12, and 6 · 8 leads to four ordered
pairs, by reversing order and choosing which one to negate.
27. 286. [51] There is a standard trick for this. Put 14 x’s in
a row, and then put vertical lines at three distinct positions
separating adjacent x’s. This divides the x’s up into 4 groups.
The number in each group corresponds to the 4-tuple. Since
there are 13 spaces where the lines can go, the number of such
sequences is 13
 13·12·11
3
= 6 = 286.
28. 1000/1001. [22] By the quadratic formula, the points of inter-
1

section have x = 16 ± 14 2n + 2. The difference between these

x-values is 12 2n + 2. Since the lines have slope 1, the length of

the line connecting the points is dn = n + 1. The associated
1
summand is n(n+1) = n1 − n+1
1
. The sum of these as n goes from
1
1 to 1000 is 1 − 1001 .
29. 110. [16] There exist numbers a, b, c, and d such that
(x + c)(x2 − 5x + 4) + ax + b = P (x)
= (x + d)(x2 − 5x + 6) + 2ax + 2b.
Then 4c + b = 100 = 6d + 2b. Now P (5) = 4(5 + c) + 5a + b =
120 + 5a and P (5) = 6(5 + d) + 10a + 2b = 130 + 10a. Since
these are equal, a = −2 and P (5) = 110.
30. (3, ∞). [8] By the quadratic formula, we have two distinct pos-
itive roots if and only if a > 0 and 0 < b < a2 /4. The desired
expression can equal any value strictly between 34 a + a3 to a + a3 .
This can be arbitrarily large. To find its minimum value with-
out using calculus, we note that the minimum value of a sum
of two positive numbers whose product is fixed occurs when
they are equal. Thus the minimum value of 34 a + a3 occurs when
a = 2, and the value equals 3. Our expression must be strictly
greater than this.
11

31. 6. [26] If x is the desired expected value, then x = 13 ·2+ 23 (x+2),


since after two steps the bug will either be at the center or back
at a corner vertex. Solving yields x = 6.
32. 28. [28] The only numbers with exactly 6 divisors are p5 and
p2 q, with p and q prime, q 6= p. The sum of the reciprocals of
the divisors of p5 is less than 2, so we want (1+ p1 + p12 )(1+ 1q ) = 2,
hence (p2 + p + 1)(q + 1) = 2p2 q. This implies p2 + p + 1 = 2q
and q + 1 = p2 , or p2 + p + 1 = q and q + 1 = 2p2 . These simplify
to p2 − p − 3 = 0 or p2 − p − 2 = 0, of which only the second
has a positive integer solution, p = 2, yielding q = 7.
33. 16. [23] Since all fourth powers are either 0 or 1 mod 5, either x
or y must be a multiple of 5. Since 54 = 625, and 15266 − 625 =
14641 = 114 , (x, y) = (5, 11) works. Since 84 < 5266 < 94 , we
cannot get a pair using 104 , and 154 is too large.
34. 17, 25, 39, 113. [18] Since ABC is a right triangle, we must
have 302 + x2 = 4r2 , where x and r are positive integers. Note
that x must be even, so let x = 2y, and we have 225 + y 2 = r2 ,
for integers r and y. Thus 225 = (r − y)(r + y), and we are led
to consider the ways of writing 225 as a product a · b of positive
integers. Then r = 21 (a+b). The factorizations are 225·1, 75·3,
45 · 5, 25 · 9, and 15 · 15. The last one would yield y = 0, so it
is invalid.
35. 14365. [38] Note that 2107 = 72 · 43. Thus n can be of the form
p6 q 6 r42 or p48 q 42 or p6 q 300 or p2106 , where p, q, and r are distinct
primes. The number of divisors of n2 in these three cases are
13 · 13 · 85, 97 · 85, and 13 · 601, resp. The first of these is the
largest.
36. 359. [13] Since (n + 1) · n! = (n + 1)!, there would be no point
in using more than n copies of n!. Every positive integer can
P
be written uniquely as ai · i! with 0 ≤ ai ≤ i. This can
be proved by induction on the largest value of i. [If all sums
with i < n give all numbers less than n!, then adjoining all
12

multiples of n! gives all numbers less than (n + 1)!.] Thus using


10 (= 1+2+3+4) or fewer factorials gets us up to 5!−1 = 119.
Adding 5! to these gets us up to 2 · 5! − 1 = 239 using 11 or
fewer. Using 2 · 5! plus lower combinations gets to everything
less than 2 · 5! + 4 · 4! + 3 · 3! + 2 · 2! + 1! with 11 or fewer, but this
value, which equals 3 · 5! − 1, cannot be done with 11 factorials.
37. −3. [11] Let k be the common value of the three fractions.
Then a, b, and c are roots of the polynomial x3 − kx + 1, and
since they are distinct, it factors as (x − a)(x − b)(x − c). Hence
x3 − kx + 1 = x3 − (a + b + c)x2 + (ab + ac + bc)x − abc. Thus
a + b + c = 0, abc = −1, and
a3 + b 3 + c 3
= (a + b + c)(a2 + b2 + c2 − ab − ac − bc) + 3abc = −3.


38. 24 5/97. [3] Let AB = 7, BC = 8, and CA = 9, and AP = x,
BP = y, and CP = z. Then
y 2 = 72 + x2 − 14x cos α
z 2 = 82 + y 2 − 16y cos α
x2 = 92 + z 2 − 18z cos α
Adding these gives (14x + 16y + 18z) cos α = 194. Computing
the area of triangle ABC both by Heron’s formula and by sum-

ming the areas of the three subtriangles yields 12 · 3 · 4 · 5 =
1
(7x+8y+9z) sin α. Dividing the two equations gives 14 tan α =
2 √
12 5
194
.
39. 41. [18] Suppose that removing the last two digits of n2 leaves
the perfect square s2 . Then 0 < n2 − (10s)2 < 100. Then
n2 −(10s)2 = (n+10s)(n−10s) ≥ n+10s > 10s+10s = 20s, so
20s < 100. Therefore s ≤ 4 and hence n2 < 100+(10s)2 ≤ 1700.
Therefore n ≤ 41, and 412 = 1681, so n = 41 works.
1023
40. 512
[1] Let Q(x) = P (2x) − P (x) − 1. Then Q(2i ) = 0 for
.
0 ≤ i ≤ 9, and hence Q(x) = α(x − 20 )(x − 21 ) · · · (x − 29 )
13

for some number α. The coefficient of x in P (x) equals the


coefficient of x in Q(x), and this equals −Q(0) 210 + 211 + · · · +
1
= 210 + 211 + · · · + 219 = 2 − 219 .

2 9

You might also like